Glory 2019 Final

Download as pdf or txt
Download as pdf or txt
You are on page 1of 275

Glory ..

SMLE 2019 collection

Section1: medicine 3 - 118


2-144

Section2: pediatric
145-186

Section3: obgyn
187-231

Section4: surgery
232-275
Section 1..
Medicine ..
1.Someone with HF on lasix, BB, captopril he has chronic dry cough. What is
the cause ?
A.BB
B.Captopril
C.lasix
D.drug irrelevant

N.B: ACEi side effects are dry cough and hyperkalemia and facial + tongue swelling
(angioedema).
N.B: Thiazide side effects are hypokalemia and hyperuricemia

2. has fever with chills , low back pain , his urine .leukocytes +nitrate how to
manage ?
A.Oral antibiotic
B.ivabx at home
C.admission

N.B: Acute prostatitis = Low back pain.


N.B: Acute pyelonephritis = Flank (costo-vertebral angle) pain.

3. Divorced women has a baby with sickle cell anemia , she want to get
married again but scared of having baby with SCA , what she can do ?
A.karyotype for the husband
B.karyotype for herself
C.HB electrophoresis for her
D. HB electrophoresis for husband

4. diabetic pt on insulin has extremities paresthesia and forget most of the


time , Hb 10 , MCV high, how to manage ?
A.oral iron
B.oral B12
C.Injectable vit B 12
D.packed RBCs transfusion

Vit B12 preferred to be given IM not orally.


Causes of high MCV (macrocytic anemia) are either megaloblastic or non. With
megaloblasts (pigmented neutrophils): Vit B12 and Folate deficiency
Non-megaloblastic: Liver disease, Alcoholism, Methotrexate, Hypothyroidism,
Zidovudine.

5. Case Scenario for 40 years old female with history of travel/ trauma
presented with swelling in one leg , non-pitting edema , skin color is blue,
pulse is intact, How to manage ?
A- lymphpain
B- massage and stocking
C- Anticoagulant

N.B: This is typical DVT.

6. man with HTN Lab investigation normal but CT scan shows small mass - - -
lipid change?
A.adrenalectomy
B.MRI adrenal
C.observation

N.B: This is chusing by adrenal hyperplasia which cause HTN and dyslipidemia –

7. Patient with necrotizing pancreatitis drain done and patient improved what
is the metabolic effect
A.insulin resistance
B. hypoglycaemia
C.inhibit gluconeogenesis
D. lipidolysis

8.Screen test for Diabetic found, 90% sensitive, 80% specific. WOTF is
correct?
A.80 % of patient was non diabetic or 20% have DM.
B.90 % was diabetic

9. Case Scenario For man present with mass in the abdomen, US show
calcified lesion, your Dx ?
A- Carcinoid
B- lymphoma

10. Most important prognostic and preventing factor post MI :


A- Decrease lipid
B - stop smoking
C- decrease weight
D - exercise

11. fever, abdominal pain, low back pain , Brucella, ttt 6 weeks ??
triads of brucellosis is fever which increase afternoon, back pain , sweating,
The recommended treatment for brucellosis is given combination of rifampin and
doxycycline for 6-8 weeks

12. Hepatitis c needle stick , how much the risk to get infection?
A- 0.03 %
B- 3 %
C- 30 %

13. old pt had bilateral shoulder pain now bilateral hip pain?
polymyalgia rheumatics

it is a chronic inflammatory condition affect the elderly, pt will suffer from sever pain
and stiffness in the shoulder and hip but may affect all the body

14. Celiac disease biopsy done , showing villous atrophy , What else could you
finding?
B- subtle crypt lengthening
C- high lymphocyte

15. sickle cel anemia, splenomegaly, hypersplenism CBC ‫ﻠﻛﺤﺎﺟﺔﻗﻠﯿﻞ‬


A- exchange transfusion
B- hydroxyurea

16. meningitis -> LP G+ve, catalase +ve , ttt?


A- Ampicillin (listeria)
B- cefapime

17. pulmonary edema CHF with poor prognosis best ttt


diuretic

18. Parkinson disease Diagnosis?


A- Clinical diagnosis
B- CT

Parkinson disease is a clinical diagnosis. No laboratory biomarkers exist for the


condition, and findings on routine magnetic resonance imaging (MRI) and computed
tomography (CT) scan are unremarkable. Positron emission tomography (PET) and
single-photon emission CT (SPECT) may show findings consistent with Parkinson
disease, and olfactory testing may provide evidence pointing toward Parkinson
disease, but these studies are not routinely needed. (Olfactory testing can reveal
hyposmia, which may precede the motor signs of Parkinson disease by several
years.
Medscape

19. cystic fibrosis or celiac


prognosis +‫ﺟاﺘﻤﻌﻮاﻣﻌﺎﻟﺪﻛﺘﻮرﻋﺸﺎﻧﯿﻌﺮﻓﻮاﻛﯿﻔﯿﺔاﻟﺘﻌﺎﻣﻞ‬
‫ﻟاﺠﻠﺴﺔ ﺳاﻤﮭﺎ ﯾاﺶ؟‬
A- summarisation
B- verbal communication

20. pic of HSP. Ttt ?


Suppurative treatment
Immunoglobulin associated with it is IgA, Most complication is nephropathy
H ( hematuria) S ( skin purpura)
P ( pain in abdomen ,pain in joint

21. case of hyperkalemia ..ttt ?


Ca gluconate

Yes initial ttt of hyperkalemia is Ca gluconate to avoid cardiac complication then give
the pt insulin

22. Physical activity in adult


A- 1/2 hour in 5 days
B- 1/3 hour in 6 days

N.B: 150 hours per weak.

23. hematemesis ?
chronic liver disease Esophageal varices
Most common cause of mild hematmesis is peptic ulcer then esophageal varices
Most common cause of severe hematmesis is esophageal varices

24. egg allergy vaccine ->yellow fever

25. case of shock ( cardiogenic ) 2 different question


One with increase capillary refill? septic shock
One with fever ? bacteremia sepsis

26. Asthmatic patient not controlled on SABA. What to do next ?


A- Add ICS.

N.B: BA management started by SABA, then add ICS, then LABA if not controlled.

27.WOTF is a sign of acute severe asthma?


A-RR >25.
B-HR > 100.
C-O2 saturation < 95%.
D-Peakexpiratoryflow < 300.

28. Pt known COPD , come to ER drowsy , o2 sat 84 % , ABGS shows I think


respiratory acidosis. TTT?
A-o2 by mask
B-mechanical ventilation(Not sure)
C-Nubelizer

29. Type of ABG abnormality in excessive vomiting ?


A-Metabolic alkalosis with hypokalemia

30. Pt known case of PUD diagnosed 8 years ago come with vomiting 2 weeks
ago. On exam dehydrated and abdominal Splash test +ve. What expected on
ABG?
A-metabolicacidosis
B-Metabolic alkalosis.
C-compensated

N.B: This is about Gastric outlet syndrome due to pyloric hypertrophy obstruction
complicated a long standing PUD. Suction Splash test is positive in gastric outlet
obstruction.

31.Pt want to stop smoking, he has HTN and epilepsy. what is


contraindicated?
A-bupropion

N.B: U might be asked about S/E of bupropion? Headache.

32. 20 years old male with meningitis and seizure. TTT?


A- Vancomycin and Ceftriaxone.
B- vancomycin + ceftriaxone + steroid.

33. 50 years old male with meningitis, most common pathogen?


A-Streptococcus pneumonia.
B-listeria.

N.B: Listeria in 60 and above.

34. what’s hemosiderin disposition in macrophages in the lung is indicative


of? A-CMV
B-chronic lung infection
C. Heart failure.
D-pneumocystis jiroveci

35. which group of drugs is contraindicated in diabetic patients with heart


failure? A-biguanides
B- dipeptidyl peptides IV inhibitors
C- GLP-1 analogous
D- SGLT-2 inhibitors
E- Thiazolidinediones.

36. Pt has abdpain , father has hx of h pylori , her urea breath test +ve , dx ?
A-h pylori infection

37. Dermatology in celiac disease ?


A- dermatitis herpetiformis

38. Female with hypotention ,hypok+ , hyponNa , dark line in hand scar what
next?
A-dexasupprestion test
B-cortisollevel
C-syntchan test

39.Turner syndrome ass with ?


A-DM
B-hypothyroidism
C-hyperprolactinemia

40. Pt 660 years ask for cardiac assessment : ASCVD RISK 6.5 % WHAT ELSE
? A-highly sensitive CRP
B- stress echo

41. meningitisprophylaxis ?
A-oral rifampicin .
depends on age (child oral rifampicin ,If adult Ciprofloxacin )

42. pt had low immunity vaccine not to give ? A-varicella

43. about x linked agammaglobulinemia?


Deficiency in Cd19 and CD 20

Defect in tyrosine kinase that prevent the development of mature B cells from pre B
cells in bone marrow (B lymphocytes= total lymphocytes- T lymphocytes)
Predisposed to recurrent infections with encapsulated organisms like H. influenza, S.
pneumonia due to impaired humoral immunity

44. Question about hemorrhagic shock


45-Question cardiogenic shock

46. scenario pt with resting tremors mask like faces ?


A-Parkinson’s disease
B-tardivedyskinesia
C-hypothyroidism
47. a scenario of pt describing rash on palms and soles and syphyllis features
causative organism ?
A-treponema pallium

48. aggressive treatment of SlE ?


A-prednisolone and mycophenolatemofetil
B-prednisoneandhydroxychloroquine
C-prednisone and cyclophosphamide

49.Another scenario of Slept on hydroxychloroquine developed some


additional symptoms what to do ?
A-add prednisone
B-increase the dose of hydroxychloroquine

50. a scenario of pt with 11 months history of fatigue developed jaundice 1 to 2


wks ago with raised lfts ?
A HCC
B AUTOIMMUNE HEPATITIS

51. a scenario of cardiac temponade muffled heart sound distended neck veins
on ecg specific finding
A- diffuse st elevation
B- prdepression

-The answer is low voltage or sinus alternans

52. ecg showing heart block


A- first degree
B- 2nd
C- 3rd
D- 4th

53. a pic of urine dipstick (coloured) showing +3 proteinuria ?


The answer should be (300-1000 mg/dl)

54. saag less than 1.1g/dL Cause


A-peritonealtb
B liver cirrhosis

55. exercise for Diabetic obese pts (I guess )


A 150 min per week
B 180 min/ek

56. causative organism for acute brochiolitis


A RSV

57.a case of GB
syndrome with features showing ascending paralysis and hx of URTI

58.treatment of GB syndrome
A plasmapheresis
B IV immunoglobulins

59. a pt with all cell lines depressed


A aplastic anemia
B hemolytic
C Sca
D IDA

60. a male pt with normal T cell count but B cells almost diminished
A brutonagammaglobulinemia

61. a baby with abdominal distension and palpable bladder


A prune belly syndrome

62. a 60 yr old pt fights and argues with every 1 sth related to his cousin was
also mentioned which I hv forgotten
A Alzheimer’s dementia
B vascular dementia
C Huntington disease
D frontotemporaldementia

63. a scenario showing features of celiac disease


A celiac disease

64. in psoriasis
A clinical hx and examination is adequate to reach the diagnosis
B skin biopsy is confirmatory

65. fixed splitting of S2 bicuspid ventricular hyperteophy


A AVSD
B ASD
C VSd

66. treatment of RHF in which pulmonary edema was developing


A IV frusemide

67. a pt after carpel tunnel release presented with pain what to give for pain
relief A triptans
B opioids
C NSAIDs

68. H pylori pt treated with clarithro amoxicillin and omeprazole for 10 days not
improved what to do?
A same regimen for 14 days again
B switch to doxycycline metronidazole and PPI.

Look for other options


10 days ok need to switch but not doxy. Switch to levofloxacin amoxcillinomperazole
regime

69. a pt with hx of H. Pylori infection along with maltoma( 2 queries related to


this) A eradicate H.pylori first
B treat maltoma first

70. a 60 yr old lady with features of osteoarthritis and varus force was
mentioned in the scenario but there were no options of medial or lateral
meniscal tears

71. treatment of hyperkalemia( 2 queries related to this)


A cagluconate
B insulin

72. a pt developed fever after 17 days of chemo


A send CBC blood culture urine culture and give antipyretics
B send CBC blood culture urine culture and start broad spectrum antibiotics

73. pt working in some factory develops symptoms of asthma wat to do ist?


A CXR
B PFTS
C ABGS
D CBC

74. a young child newly diagnosed asthmatic mother also asthmatic father is a
chain smoker
A stop smoking in the house
B give short acting beta agonist to the child

75. Pt in COPD or Asthma


A increased TLC decreased Fvc and decreased fev1/fvc
B decreased TLC INCREASED FVC AND DECREASED FEV1 to Fvc
C increased tlc normal Fvc and decreased fev1/Fvc

76. a pt started taking enoxaprin bd for pulmonary embolism worsening of


symptoms in the evening
A- switch to heparin
B- increase the dose of enoaxprin
C- continue the same treatment

77. a patient is diagnosed with HIV


A- tell the pt
B- tell his wife
C- tell both of them

78. Thromolytic only in persistent hypotension


PE with hypotension give thrompolytic

79. Aspirin toxicity level in the blood , Starts from 50 mg/dl?


A- alkalization of urine
B- dialysis

80. anterior lateral MI has ,heart failure


A- thrombolytic, NG,
B- heparin, NG, thrombolytic
C- heparin, NG

81. in the lab show low Tsh and 3×4 ‫ اوﻵﺧﺮ‬٢×٤pt with diffuse goiter in us have 2
nodular once ‫ﻟاﺴﺆال ﻻاﺧﺮ‬highT3 what the best managment:
A- Total thyroectomy
B- Thyroid scan
C- Fna in both nodul
D- Fnainlargestnodul

82. the most important prognostic preventing factor after in post MI


A- Lower lipid
B- Stop smoking
C- Loss weight
D- Exercise

83. 17 yrs old football player present with *area macular hyperpigmentation on
the trunk and arms, the area seem even lighter with sun exposure, what is the
most appropriate treatment regimen for this patient?
A. Topical steroid cream
B. Selenium sulphate
C. Oral antibiotic
D. Topical antibiotic

- pityriasis versicolor-

84. ECG pericarditis. what does the segment show?


A. diffuse st segment elevation
B.pr depression

85. Pericardial tamponade, PT hypotension, management?


Fluids to avoid Cardiac collapse+ pericardiocentesis

86. Cardiac tamponade, diagnosis?


Echo is best

87. OSPE, anterior myocardial infarction?


St elevation in V1, V2 , V3 , V4 is anterior MI

88. effusion analysis, what diagnosis?

89. Pre renal, post renal and renal causes of renal failure. How to diagnose
each one?
Prerenal : hyaline casts
pre renal .. usually manifested by hypotension either due to dehydration or
hemorrhage + renal artery stensois
Renal : brown muddy like casts

90. Perforated ulcer management?


Graham omental patch

91. Pt complaining of black stool before 3 days and came with bright fresh
rectal bleeding, doctor do upper and lower endoscopy and reveal
sigmoidoscopy, was negative and unable to reach diagnosis, what the next
management?
A. Abdomen CT
B. Capsule endoscope
C. NGT

92. complaining of fever, rigor, nausea and vomiting, splenomegaly, blood film
negative, next step?
A.Thick blood film after 72 hours
B.Thin blood film after 48 hours
C.Serology test

Typical its thick film in 24 to 48 hr If not in options chose B

93. HOCM. Diagnosis?


By echo
Hx of sudden death + murmur increase with Valsalva

94. causes of acute tubular necrosis.


Hypotension and nephrotoxic drugs
95. Two medicines, methylphenidate and amphetamines, are the most
commonly used stimulants for the treatment of ADHD. Methylphenidate –
Methylphenidate (sample brand names: Concerta, Focalin, Metadate, Methylin,
Ritalin) is available as a tablet, capsule, and liquid.

96. Case About ADHD , and which treatment is best ?


Less than 6years > behavioral More than 6years > CNS stimulants
Bmethylphenidate is the best

97- calculate the BMI.

98- 45 male came with persistent neck pain with diaphoresis troponin normal.
ECG unremarkable. What will you do ?
A. Repeat troponin after 6 hr
B. Give ibuprofen

99. Aortic stenosis case .


Ejection systolic murmur radiated to neck + syncope + dyspnea

100- What’s increase prognosis of COPD ?


A. Supplement O2
B. Smoking cessation

101- Case of management of DM?

102-How many years of quitting smoking required to decrease the risk of


ischemic heart disease?
A.1year
B. 4 years
C.3 years
D. 2 years
103-Hyperthyroidism what is the treatment?
Propylthiouracil - Methimazole..etc

^ medical treatment
Note: Best drug for hyperthyroidism in pregnancy Propylthiouracil
Radioactive Iodine
^Surgical

104-Clinical feature of hypo/Hyperthyroidism with labs asking for diagnosis

105-pt with distal and proximal hand joint swelling and morning stiffness is
and RF -ve no other symptoms :
A-Reactive arthritis
B-Seronegative arthritis
C-Hand OA

https://www.ama-assn.org/residents-students/usmle/usmle-step-2-which-diagnosis-
hand-pain-stiffness

106-neurohormonal stress release


A-androgen
B-corticotrpion
107-pt smoker DM and HTN has bilateral leg pain and loss of hair what dx:
A-peripheral Venus insufficiency
B- chronic limb ischemia
C- DVT

108- old pt with CHD on Lisinopril and frusemide and statin what you will add:
A-losrtan
B-bisoprolol

We don’t use ACEi and ARBS together


Add BB for long term ttt

109- pt with RA on prendisolon and hydroxyurea and methoterxate and still


have symptoms what you will add:
A-azithrprion
B-infliximab
C-cyclosporin

-If DAMARD not effective we should use next line treatment anti TNF-
Note: In another question (SLE), the answer was cyclo
But we said if infliximab was there , go for it !

110- Elderly pt in ICU, with euthyroid syndrome What you’ll see in his lab ?
A-high T3 ,T4 ,reverse T3
B-high T3, T4 and low reverse T3
C-high T3, T4 and normal reverse T3
D-low T3,T4 and high reverse T3
111- pt with meningitis High protien Low glucose 70% polymorphus
A-cryptoccos
B-TB
C-viral
D-bacterial

112- pt with meningitis High protien Normal glucose Prominat lymphocyte


A-cryptoccos
B-TB
C-viral
D-bacterial

113- pt newly dx with HTN and doctor tell him about disease and complication
and medicationt what doctor did?
A-give informational case
B-build bonding with pt

114-internal medicine resident did diabetic foot examination but she forget to
close curtains when she expose pt abdomen what the ethical principles she is
broke?
A-privacy
B-dignity
C-autonomy
D-confidenenality

115- pt with diarrhea and tensmus what dx ?


A-amebia

116- pt has numbness in lower part of periorbital aspect and upper lip what is
the nerve injured?
A-mandible
B-infraorbital

117- female his son died before 5 days and she is crying and can not sleep
since 2 days what you will give in short course??
A-lorazepam
B- fluxtein

118- what is the antipsychotic most likely side effect is wt gain?


A-olanzepin
B-rospiredon

119- 93 year oldpt take paracetamol with codien and he has symptoms after
take what is the cause?
A-codien
B-paracetamol

120-pt with hemoptysis and hematurea?


A-goodposture disease
B-glumeronephritis

121- young lady with SOB, ..etc (PE) , Unsyable ( CT shows huge emboli)
A- enaxparine
B- unfractionated heparin+ warfarin
D-altepase

122- Heptitis B vaccines


A- toxoid
B- recombinant
C- attenuated
A- Inactivated

123- pt presented with inability to extend wrist, fingers, thumb extention,


thenerms wasting and numbness over the snuff box , radial nerve injury at
which level
A- Olecranon
B- Spiral groove
C- Carpal tunnel

124- pt on parental nutrion Lab was mentioned Only thing was elevated INR
What should you add?
A- vit K

125-pt after inferior MI of some medications sorry I cant remember them


What to add
A-BB
B-CCB

126- pt with CVS disease and impaired lipid profile what should you give him
A-Statin
B-Fibrate

127- which ABX excerbats SLE symptoms


NO choices

128- Symptom confirm diagnosis of eosinophilic esophagitis


A- dysphagia
B- stucking of food in esophagus
C - chest pain centrally located not responded to anti acid
D - confirmation by biopsy and endoscopy

129- pt vet presented with 2 mths HX of severe lower back pain and fatigue and
now agitated , Sacal joint tendrness ,I think physical examination should
splenomegaly ,Some labs were mentioned , What is the Dx
a- Brucellosis
b- Toxoplasma

130- how to confirm the dx in the previous case


NO choice
it depends
Acute less than 1 year serology
Chronic more than 1 year blood culture

131- pt with meningitis LP should lymphocytosis , normal sugar


How to manage it
A-Acyclovir

132- which type of HCV is common in Saudi


NO choices
- 4

133- pt had nephrotic syndrome ,Presented with proteinuria which medication


protect the kidney
A-ACE I
B-Prednisolone

134- lady k/c of BA on short acting b agonist and steroid inhaler in the
previous yr used SABA only once
a- Stop both medications
b- Half the dose of steroid and review in 6 mths
c- Double the dose

135- Case of septic arthritis. First step of management?


A. Fluid aspiration
B. Broad spectrum antibiotic
C. Blood culture
D. Drainage and debridement
Explanation :
Clinically patient will be presented with fever, joint pain and swelling. On joint
aspiration there will be high WBCs with neutrophils predominant without crystals,
Most accurate test: Joint aspiration
Empirically treat with ceftriaxone and vancomycin initially until culture test results;
then modify therapy for specific organisms. Septic joints are treated with joint
drainage or surgical debridement.

136- Case of gout. What is the type of crystal you will find?
Explanation: (Joint aspiration and synovial fluid analysis is the only way to make a
definitive diagnosis of gout; needle-shaped and negatively birefringent monosodium
urate crystals appear in synovial fluid)

137- Case of gout. Which medication shouldn’t be given in acute


exacerbation?
A. Allopurinol.

Explanation :
Treatment of acute gout usually started with NSAID, if no improvement then go to
either Colchicine or intraarticular Steroid if patient has renal failure, never use
Allopurinol in acute attack because it worsens the condition
^ Toronto

138- Symptoms of depression. What is the diagnosis?

Explanation:
SIG E CAPS
Sleep (hypersomnia or insomnia)
Interest (loss of interest or pleasure inactivities)
Guilt (feelings of worthlessness orinappropriate guilt)
Energy (↓) or fatigue
Concentration (↓)
Appetite (↑ or ↓) or weight (↑ or ↓)
Psychomotor agitation or retardation
Suicidal ideation

139- Pt with generalized anxiety disorder. Which medication to give?


A- Lorazepam.
B- Bubropion.
C- Buspirone.
D- Escitalopram.
Explanation
140- Two Cases of about irritable bowel syndrome.

CXR findings in neonatal lung diseases; in RDS there will be ground glass
appearance (homogeneous infiltration) + air bronchogram and decreased lung
volumes , in Meconium aspiration syndrome there will be coarse irregular patchy
infiltrates + flattening of the diaphragm + lung hyper expansion (hyperinflation) and
10-20 % may have pneumothorax)
^ Toronto

141- Clinical scenario of (painless genital ulcer + skin rash). What is the dx?
A- Secondary syphilis.

142- Positive H.pylori what is the treatment?


Metronidazole + PPI + clarithromycin

143- Pt with dyspepsia not responding to the H2 blocker what to do?


A- PPIs.

144- Young lady with presentation looks like DVT and amenorrhea for several
wks , PT -ve What is the important q to ask?
A- History OCP.
B-

145- Parasitic liver lesions how to mange?


A- Deroofing.
B- Aspiration.
C- Abx.

146- after diarrhea what to expect?


Metabolic acidosis
Metabolic alkalosis

147- what does nicotine in cigarette do


- Additiction effect
148- which Abx for TB exacerbate gout

149- Traveler's bloody diarrhea culture campylobacter


- Ciprofloxacin?
-Azithromycin \ erythro

150- in poor area


- Heptits A

151- hemolytic jaundice


- Increase indirect bilirubin

152- 55 yr old male with Afib what to give him prevent complications
- Warfarin

153- traveller advise


- Peel fruit

154- pt with chronic disease came with microcytic hypochromic anemia ,fretin
double the normal dx
- Anemi of chromic disease
- Sidroplasticnemia
- IDA

155- pt with fever ,cercical lymphadenopathy swollen tonsils aithwt patch


Possible complication
- Pharyngitis
- Scarlet fever

156- two cases of pleural tab analysis and Dx

157- microscopic finding of minimal change diseas


- effacement of the epithelial cell (podocyte) foot processes and loss of the
normal charge barrier

158-prophylactic of cluster headache ?


Verapmil and lithium
Step up is drug of choice is verapmil

159- migraine what is the symptoms and signs for dx


160- tension headache what is the dx ,ttt
161- complication of polymyalgia rheumatica ?
- With giant cell arteritis , Blindness & stiffness
- Complications related to GCA and PMR and to therapy for these conditions
include osteoporosis, corticosteroid myopathy, bruising, emotional symptoms
(e.g., insomnia, restlessness, hypomania, depression), hypertension,
diabetes, elevated cholesterol, and fluid retention. Source: AAFP

162- frontal sinusitis not responding to simple analgesia / decongestant what


is next?
- Antibiotic
- First-line therapy is amoxicillin with or without clavulanate
163- after seizur flat shoulder arm adduct and internally rotated
A- Posterior dislocation
B- Anterior dislocation

164- pt with DM annual check up


A- Creatinine/ albumin ratio
B - Urine microalbuminuria

165- case of Cushing syndrome diagnosis ?


166- young with coma after exercise
A- Long qt syndrome
B - HOCM

167-pt with low O2 first step of mx ?


O2

168- PtoncestartvancomycinstarttogetrashWhattodo?
1- Stopandlabel
2- Continueonsmallerdose
3- Replacewithcephalosporins
Explanation:
RapidinfusionofVancomycinmayalsocauseflushingoftheupperbody
)"called "redneck" or "redmansyndrome(
169- LDL levelgoalin a normal diabetic pt
1- <70 mg/dl
2- <100 mg/dl
3- <70 mcg/dl

170- an elderly man with a history of asthma, congestive heart failure, and
peptic ulcer disease is admitted with bronchospasm and rapid atrial
fibrillation. He receives frequent nebuliser salbutamol and IV digoxin loading,
his regular medications are continued. 24 hours after admission his serum
potassium is noted to be 2.8 mmol/l.
Which of his medications is most likely caused his abnormality?
a. Digoxin
b. ACE inhibitor
c. Salbutamol
d. Ranitidine

171-Patient has pleuritic chest pain on the left side, physical examination
reveals a pleuritic fruction rub, what’s the next step
a. CT Thorax
b. Chestxray
c. Echorefertocardio
Explanation:
Usuallywedo ECH to Ruleoutthepericardialeffusion

172- a 40 year old male has been diagnosed with lung cancer, he and his
family have never smoked, has no family history of lung cancer, he works on
electricity generation.
What’s the cause?
a. Asbestos
b. Inhaledcoaldust
c. Passivesmoking
d. Radon gas
Notes:
farmerlung>>abstosis :
Cotton>>bezenosisi
Shipbulid>>asbestosis
Begozosis>>sugerchaey
173-The diagnosis of DVT is?
US duplex.

174- pt with distal and proximal hand joint swelling and morning stiffness is
and RF -ve no other symptoms?
a. Reactivearthritis
b. Seronegativearthritis
c. Hand OA
Explanation:
Weneedtoknowisitunilateralorbilateral
Weneedtoknowtheage
Thedurationofstiffness

175-neurohormanl stress release


a. androgen
b. corticotrpion

176-Ptsmoker DM and HTN has bilateral leg pain and loss of hair what dx
a. peripheralvenusinsuffincy
b. chroniclimbischemia
c. DVT

177-old pt with CHD on lisiopril and fursamide and statin what you will add
a. losrtan
b. bisoprolol

178- ptwirh RA
onprendisolonandhydroxyureaandmethoterxateandstillhavesymptomswhatyo
uwilladd
a. azithrprion
b. infliximab
c. cyclosporin

179-Sick euothyroid syndrome


a. high T3 ,T4 ,reverse T3
b. high T3, T4 and low reverse T3
c. high T3, T4 and normal reverse T3
d. low T3,T4 and high reverse T3

180-pt with meningitis , High protien , Low glucose


polymorphus %70
a. cryptoccos
b. TB
c. viral
d. bacterial

181-pt with meningitis , High protien ,Normal glucose , Prominatlymphocyte


a. cryptoccos
b. TB
c. viral
d. bacterial

182- A 69-year-old man is treated for chest infection. He has been on a stable
dose of warfarin for the last six months as a treatment for atrial fibrillation,
with INR recordings between 2-2.5. However, his most recent INR was 5.
Which one of the following drugs that has recently been started is likely to be
responsible for the increased INR.
1- Clarithromycin
2- Co-dydramol
3- Digoxin
4- Rifampicin
5- Temazepam

183- A 30-year-old man presented with a history of transient loss of


consciousness and palpitation. His ECG showed ventricular tachycardia.
Which one of the following treatments should be avoided?
1- Adenosine
2- Amiodarone
3- DC cardioversion
4- Flecainide
5- Verapamil

184- 28-year-old man who is known to have Hypertrophic Cardiomyopathy has


an out of hospital cardiac arrest and is successfully resuscitated.
What is the most appropriate mode of treatment?
1- Alcohol Septal Ablation
2- Amiodarone
3- Beta Blocker
4- Implantable Defibrillator
5- Myomectomy

185- 30yrs old e painless jaundice no other symptoms physical exam


normal,bilirubin15mainly
indirect liver enzyme normal wh is the d.d
A-Hypothyrodism
B-Cholidecal cyst
C-Gilbert synd

186- 6-7yrs old had clumsy gait and unable to walk or stand,hadchecken box
begore 3wks,muscle
taken no abnormalities also had resistance to neck flexion wh is the d.d
A-GBS
B-Meningoencephalities

187- Elderly pt had pneumonia she became restless and agitated for the nurse
what will do
A-Tell the family to staye her
B-Give oxy and elevate the head bed
D-Call the Dr duty and to give lorazepam 5mg

188- Pt diagnosed ALL invest wbc22 Hb8 K5.7 phosphate low uric acid high
which of the electrolyte cause this condition
A-HypoCa
B-HyperCa
C-HypoNa
D-HyperNs

189- The nearest big-city hospital is a 6-hour drive. Given your attention to
your first priority, you would now:
A. call the ambulance for immediate transport to another hospital that “knows how to
look after this thing”
B. admit the patient to the coronary care unit for observation
C. administer streptokinase or alteplase IV immediately
D. administer heparin IV immediately
E. none of the above
- note:Absolute contraindications to thrombolytic therapy include:
• Any prior intracranial hemorrhage
• Known structural cerebral vascular lesion (e.g., arteriovenous malformation)
• Known malignant intracranial neoplasm (primary or metastatic)
• Ischemic stroke within 3 months EXCEPT acute ischemic stroke within 4.5 hours
• Suspected aortic dissection
• Active bleeding or bleeding diathesis (excluding menses)
• Significant closed-head or facial trauma within 3 months
• Intracranial or intraspinal surgery within 2 months
• Severe uncontrolled hypertension (unresponsive to emergency therapy)
• For streptokinase, prior treatment within the previous 6 months

190- The most correct statement regarding thrombolytic therapy in acute


myocardial infarction (AMI) is:
A. patients younger than 65 years benefit more than elderly victims of MI
B. no benefits have been realized when therapy has been instituted after 6 hours of
onset of chest pain
C. thrombolytic therapy has improved the prognosis of patients with prior coronary
artery bypass grafting (CABG)
D. patients with non–Q-wave MI have benefited as well as patients who sustain Q-
wave MIs with thrombolytic therapy
E. a 50% reduction in mortality has been realized when therapy is administered
within 3 hours of onset of chest pain

191- Which of the following statements regarding the use of heparin in patients
with AMI is (are) true?
a. heparin therapy is used almost routinely with thrombolytic therapy during the acute
phase of MI treatment, provided certain criteria are met
b. heparin is recommended whenever there is echocardiographic evidence of left
ventricular thrombi
c. heparin should be administered (unless contraindicated)
to all patients with acute anterior wall MI
d. heparin is contraindicated in patients with uncontrolled hypertension
e. all of the above

192- Which of the following is (are) a significant feature(s) of the


pathophysiologic process of MI?
a. endothelial cell wall damage
b. coronary atherosclerosis
c. thromboxane A2 production
d. all of the above
e. a and b

193- Which of the following is (are) true concerning aspirin in the treatment of
AMI?
a. aspirin may serve as a substitute for streptokinase
or t-PA
b. aspirin may serve as a substitute for heparin
c. aspirin may serve as a substitute for beta blockade
d. all of the above
e. none of the above

194- Which of the following statements regarding thrombolytic therapy is (are)


false?
a. thrombolytic therapy limits myocardial necrosis
b. thrombolytic therapy preserves left ventricular
function
c. thrombolytic therapy reduces mortality
d. all of the above
e. none of the above

195- Which of the following is true regarding primary angioplasty in the


treatment of acute ST-segment elevated MI?
A. primary angioplasty is not a substitute for thrombolytic therapy
B. its universal adoption is likely to be limited by
geography
C. it can be performed in hospitals that do not perform
cabg surgery
D. stent placement worsens outcomes
E. operator variables are insignificant

196- Coronary reperfusion with thrombolytic agents has been shown to be of


benefit when it is commenced within which of the following maximum number
of hours from the onset of pain?
a. 4
b. 6
c. 12
d. 24
e. 48

197- In this clinical case, there is a key finding on physical examination of the
patient’s abdomen that should be further assessed by:
a. abdominal ultrasonography
b. intravenous pyelography
c. digital subtraction angiography
d. computed tomography (CT)
e. magnetic resonance imaging (MRI)
- missing information

198-Which of the following is the most common cause of community acquired


pneumonia?
A. Haemophilus influenza
B. Pneumocystis pneumonia
C. Streptococcus pneumoniae
D. Streptococcus pyogenes

199- A 80 year old who has been in the ICU for 4 days has developed
pneuomnia. What is the most likely organism?
A. Mycoplasma pneumoniae
B. Neisesria meningitidis
C. Pseudomonas aerogenas
D. Strep pneumoniae
200- A 30 year oldfemaleis admitted for pulmonary embolism. She has a
history of 3 miscarriages. Herlupus anticoagulant test is positive. What is the
diangosis?
A. Antiphospholipid syndrome
B. Factor V Leiden
C. Hemophilia A

201-A 36-year-old woman is diagnosed with tuberculosis three months ago,


now she complains of numbness, tingling of extremities and ataxia. Her
symptoms started after the anti-tubercular therapy.
What is the most appropriate next step in management?
A. Folic acid supplement
B. Iron supplement
C. Niacin supplement
D. Pyridoxine supplement

202- Patient has a pleuritic chest pain on the left side. Physical
examinationreveals a pleuritic fruction rub. What is the next step?
A. 21lead ecg
B. CT Thorax
C. Chest x-ray
D. Echo refer to cardio
- Initially , A
But the best is D

203-A 55 yo man with history of chronic cough associated with production of


daily mucopurulent sputum production came with hemoptysis, foul smelling
sputum, and fever. Examination revealed crackles and rhonchi. What is the
most likely diagnosis?
A. Asthma
B. Bronchiectasis
C. Lung cancer
D. Tuberculosis

204-Which clinical sign is associated with pulmonary hypertension?


A. Central cyanosis
B. Lower limb edema and ascites
C. Pulmonary edema
D. Stroke
- Pulmonary Hypertension associated with Right side heart failure

205-Which of the following is associated with oral candidiasis?


A. Aspirin
B. Decongestants
C. Epinephrine
D. Inhaled corticosteroids
206-What will be high in the lumbar puncture of a patient with mycoplasma
pneumonia?
A. Glucose
B. Protein
C. RBC
D. WBC

207-A 40 year old male has been diagnosed with lung cancer. He and
his family have never smoked, and has no family history of lung cancer. He
works in electricity generation. What is the cause?
A. Asbestos
B. Inhaled coal dust
C. Passive smoking
D. Radon gas

208-Which of the following organisms a common cause of ventilator


associated pneumonia?
A. Legionella
B. Mycoplasma pneumonia
C. Pseudomonas aeruginosa
D. Strep pneumonia

209- A 45-year-old woman presents to the emergency room with pleuritic chest
pain, hemoptysis and shortness of breath for the last three hours. Her
temperature is 37.9 C, blood pressure is 120/70 mmHg, pulse is 137/min and
respirations are 28/min. Her oxygen saturation is 86% on room air.Which of the
following is the most likely diagnosis?
A. Asthma
B. Myocardial infarction
C. Pulmonary edema
D. Pulmonary embolism

210- A middle-aged man present to the doctor with blurred vision and eye pain.
He was recently diagnosed with tuberculosis. What is the cause of his
symptoms?
A. Ethambutol
B. Isoniazid
C. Pyrazinamide
D. Rifampin

211- A middle-aged man present to the doctor with red urine. He was recently
diagnosed with tuberculosis. What is the cause of his symptoms?
A. Ethambutol
B. Isoniazid
C. Pyrazinamide
D. Rifampin

212- A middle-aged man present to the doctor with signs of gout . He was
recently diagnosed with tuberculosis. What is the cause of his symptoms?
A. Ethambutol
B. Isoniazid
C. Pyrazinamide
D. Rifampin

213-20 year old man presenting for a tuberculosis screen had a positive PPD.
What should the next step be?
A. Chest CT
B. Chest XR
C. Repeat PPD
D. Sputum smear

214-A 44 year old male has a chest x-ray done for insurance purposes. His
health has always been ne and he does not take any medications, he does not
have any allergies, nor does he smoke. He last saw a doctor several years ago,
but has never had a chest x-ray before. The radiologist has marked an arrow at
what appears to be a “tear shaped” body of the upper lobe parenchyma. The
work up for a malignancy has proven negative, all the labs are normal and the
physical exam is unremarkable. Based on your anatomical knowledge and
history, this most likely represents:
A. Apical pneumothorax
B. Azygos lobe
C. Consolidated pneumonia
D. Pancoast tumor
E. Tuberculosis

215- What is Cheyne Stokes breathing?


A. Cyclic waxing and waning ventilation with apnoea, gradually giving way to
hyperpneic breathing.
B. Increased rate and depth of breathing over a prolonged period of time
C. Only being able to breathe comfortably while upright
D. Persistent hyperventilation

216-What breathing pattern is observed after a stroke?


A. Cheyne–Stokes respiration
B. Hyperventilation
C. Kussmaul respiration
D. OSA

217-Which of the following isthe most specic test for tuberculosis?


A. AFB culture
B. CT scan
C. Chest radiograph
D. PPD

218-A 19-year- old female presents to the physician with malar rash, arthritis,
proteinuria, thrombocytopenia, positive ANA, and anti- dsDNA. Which of the
follwing is the most likely diagnosis?
A. Behcet's disease
B. Reactive arthritis
C. Sjogren syndrome
D. Systemic lupus erythematosus

219-A50 year old male prsents with chronic retrosternal pain, cough and a
metallic taste in the mouth. What is the most likely diagnosis?
A. Acrodermatitis
B. Angina
C. GERD
D. Gastritis

220- A patient with lung cancer has a low serum PTH and high Calcium .What
is this caused by?
A. Hyperparathyroidism
B. Hypoparathyroidism
C. PTH-related peptide
D. SIADH

221- What cells give rise to lung adenocarcinoma?


A. Bronchial epithelium
B. Neuroendocrine cells
C. Squamous cells
D. WBCs
- Adenocarcinoma of the lung usually evolves from the mucosal glands and
represents

222- A 40 year old female has asthma, on inhaled low-dose corticosteroids.


Despite medication compliance, she is still symptomatic with dyspnea on
exercising and night-time cough. What should the next step be?
A. Add high-dose inhaled corticosteroids
B. Add long-acting beta agonist
C. Add omalizumab
D. Add oral corticosteroids
- Saba > ICS > LABA

223- On a routine CXR, you note a suspicious single hilar mass. The patient
has a 100 pack year history, but is asymptomatic. What is the likely diagnosis?
A. Adenocarcinoma
B. Lymph node
C. Small cell lung cancer
D. Squamous cell cancer

224-What is the treatment of bronchiolitis?


A. Antibiotics
B. Bronchodilators
C. Intranasal glucocorticoids
D. Supportive care and monitoring

225-Anelderlypt presented to ER due to decreased level of consciousness


and lethargy , PCO2 is50 mmhg pH is 7.2 andHCO3 is 40. What does she have?
A. Metabolic acidosis without compensation
B. Metabolicacidosis with compensation
C. Respiratory acidosis with compensation
D. Respiratory acidosis without compensation
226-25-year-old man presents to his primary care physician complaining
of wheezing and breathlessness during exercise.
Which of the following is the most appropriate next step in management?
A. Beta-adrenergic agonists before exercise
B. Montelukast
C. Oxygen
D. Theophylline

227-45 year old female andnon-smokeris found to have a lung nodule on


CT. It appears to be composed of calcium and fat.
A. Adenocarcinoma
B. Hamartoma
C. Mystheoma
D. Squamous cell carcinoma

-Hamartoma may be confidently diagnosed when a sharply marginated, smooth


lesion
containing calcification and fat is identified on a CT scan.-
Calcified nodule > hamartoma

228-45-year-old woman presents to the emergency room with pleuritic


chest pain, hemoptysis and shortness of breath for the last three hours.
Her temperature is 37.9 C, blood pressure is 120/70 mmHg, pulse is
137/min and respirations are 28/min. Her oxygen saturation is 86% on room
Air, Which of the following is the most likely diagnosis?
A. Ashtma
B. Myocardial infarction
C. Pulmonary edema
D. Pulmonary embolism

229-What is most common manifestation of pulmonary disease in HIV


infected patients ?
A. Asthma
B. Bronchiectasis
C. Carcinoma
D. Pneumonia

230- Which of the followingdrugs can reduce the production of mucous in


COPD?
A. Albuterol
B. Formeterol
C. Ipratropium Br
D. Montelukast

C is antichonilergic

231-40 year old factory worker inhales 3 nanogram of cotton in a factory.


Where will the cotton eventually end up?
A. Engulfed by alveolar macrophages
B. Pass without any damage
C. Trapped in mucocilliary system without reaching alveoli
D. Trappedin distal airway leading to fbrosis

Byssinosis” brown lung disease” leading to IPF

232-What is th most common cause of acute bronchiolitis?


A. Adenovirus
B. Influenza
C. Parainfluenza
D. RSV

233-A 62-year-old man presents to the physician with cough for the last 7
months. He also complained of hemoptysis, weight loss and constipation.
He is a chronic smoker with a 40-year history. Laboratory tests reveal
hypercalcemia. Chest x-ray shows hilar mass in the right lung.
Which of the following is the most likely diagnosis?
A. Adenocarcinoma
B. Small cell carcinoma
C. Squamous cell carcinoma
D. Tuberculosis

Hypercalcemia and hailar location is squamous cell ca

234-A 50 year old adult man has a postive PPD skin test. He is
asymptomatic. What should the next step be?
A. Chest XR
B. Isoniazid for 9 months
C. Isonizid and rifampin for 3 months
D. Lung biopsy

235-20 year old presents with asthma exacerbation. Which of the


following drugsacts more by decreasing the mucous secretion than
bronchodilation?
A. Beta agonist
B. Ipratropium Br
C. Leukotriene antagonist
D. Oral corticosteroids

In COPD >>ibra
In asthma >> steroid

236-A 40 year old patient presents with cough during exercise. What
medication could you give her before exercise?
A. Formeterol
B. Inhaled oxygen
C. Inhaled salbutamol

237-Which of the following is the most common cause of pulmonary


embolism?
A. Antiphospholipid syndrome
B. Deep vein thrombosis
C. Nephrotic syndrome
D. Obesity

238-3 year old boy presents for TB screening. His father has pulmonary
TB. His PPD is 10mm. What does this indicate?
A. Strong postive
B. Strong negative
C. Weak positive
D. Weaknegative

239. A 30 year old female has recently given birth. She presents with a
sudden onset of shortness of breath.What would you expect to see in the xray
A. Cardiomegaly
B. Increase in mediastinal width
C. Lobar infltrate
D. Pleural effusion

240-According to the new classifcation of lung cancer, which of the


following is considered a carcinoma in-situ?
A. Adenocarcinoma less than 10mm
B. Adenocarcinoma less than 2 cm.
C. Atypical hyperplasia
D. Hamartoma

241-68 year old male with adenocarcioma of the lung presents with
blurring of the vision. On examination, you note ptosis and miosis. What is
the diagnosis?
A. Gustave's syndrome
B. Horner's syndrome
C. Hyponatraemia
D. Invasionof CNIII

-Ptosis , miosis , anhidrosis apical lung caPancost tumor-

242-Which of the following is the best treatment for a lung abscess?


A. Ciprooxacin
B. Clindamycin
C. Macrolides
D. Piperacillin

For 6 weeks

243- A 69-year-old man is treated for chest infection. He has been on a stable
dose of warfarin for the last six months as a treatment for atrial fibrillation,
with INR recordings between 2-2.5. However, his most recent INR was 5.
Which one of the following drugs that has recently been started is likely to be
responsible for the increased INR.
1- Clarithromycin
2- Co-dydramol
3- Digoxin
4- Rifampicin
5- Temazepam

Clarithromycin CLEARS the clotting factors


-cephalosporins may increase INR by inhibiting production of vitamin K-dependent
clotting factors.
-Macrolides and metronidazole may increase INR by inhibiting warfarin's metabolism

244-A 30-year-old man presented with a history of transient loss of


consciousness and palpitation. His ECG showed ventricular tachycardia.
Which one of the following treatments should be avoided?
1- Adenosine
2- Amiodarone
3- DC cardioversion
4- Flecainide
5- Verapamil
245-28-year-old man who is known to have Hypertrophic Cardiomyopathy has
an out of hospital cardiac arrest and is successfully resuscitated.
What is the most appropriate mode of treatment?
1- Alcohol Septal Ablation
2- Amiodarone
3- Beta Blocker
4- Implantable Defibrillator
5- Myomectomy

246-1-2wks Neonate has non bilious vomiting on US there is olive what you
find?
A-Hypercholaremic metabolic acidosis
B-Hyopchl Met alkalosis
C-Hypochl resp alkalosis

247-Women diabetic controlled developed dysuria and frequency urine


analysis nitrate, creatinine ,high , Which drug is contraindicated?
A-Amoxicillin
B-Septrin
C-Ciprofloxacin
D-Pipracilintazopactam

248-30yrs old e painless jaundice no other symptoms physical exam normal,


bilirubin 15mainly , indirect liver enzyme normal wh is the d.d
A-Hypothyrodism
B-Cholidecal cyst
C-Gilbert synd

249-6-7yrs old had clumsy gait and unable to walk or stand,hadchecken box
before 3wks,muscle
taken no abnormalities also had resistance to neck flexion wh is the d.d
A-GBS
B-Meningoencephalities

250 -Elderly pt had pneumonia she became restless and agitated for the nurse
what will do
A-Tell the family to staye her
B-Give oxy and elevate the head bed
D-Call the Dr duty and to give lorazepam 5mg

251-Pt diagnosed ALL invest wbc22 Hb8 K5.7 phosphate low uric acid high
which of the electrolyte cause this condition
A-HypoCa
B-HyperCa
C-HypoNa
D-HyperNs
252- rheumatoid arthritis pt with swelling in hands..(not mentioned about pain)
what to advice..
A. NSAIDS
b. methotrexate will help after 3rd stage
c. intra articular corticosteroid injections
d. symptomatic tx

253- pt diabetes type 2, with no pedal pulses in left leg, cold, raised red color
swollen area over the medial side of calf…(no pic given)
a. cellulitis
b. peripheral arterial insufficiency
c. polymyositis
d. pretibial myxedema

254- Long scenario of a man with twitching of facial muscle upon tapping of
the face, there is elevation in creatinine level diagnosis:
a) Pseudo hypoparathyroidism
b) di george s/d
c) chronic renal failure
d) hypoaldosteronism

255- long scenario given about pt having hepatomegaly lab values given alt 40
ast 200 bil 160 asked about diagnosis
a. liver cirrhosis
b. hepatic angiopathy
c. portal hypertension
d. biliary cirrhosis.
256- daily requirement of vit D:
400 IU/ml
400 > less than 1 y
600 > from 1-70
800 > more than 70 y

257- pt with pruritic folluculitis in extremitis treatment?


A. benzyl peroxide with hydrocortisone
B. topical antibiotics
C. oral antibiotics
D. systemic antibiotics.

258- normal bereavement case?


Sadness , 2-3 months .. not exceeding 6 months

259- 14 yr old child diagnosed with DM type 1, when to refer for ophthalmic
examination…
a. now and repeat after every 5 years
b. after every 3 years
c. each year after the age of 40.
d. yearly, after 5 years from now
DM1 > 5 y from dx then annually
DM2 > at time of dx then annually

260- pt with URTI, since 2 weeks, red congested throat pic was given…
(streptococcal pharyngitis) treatment…
a. ceftriaxone
b. vancomycin
c. doxycycline
d. cephalosporin.

Cephalosporins has good eradication more than penicillin


261- pt with meningitis, his sibling allergic to the primary treatment given for
prophylaxis….what to give him as prophylaxis now??
a. erythromycin
b. vancomycin
c. ceftriaxone
d. cefuroxime

262- LDL level goal in a normal diabetic pt.


a. <70mg/dl
b. <100mg/dl
c. <110mg/dl
d. <70mcg/dl

263- why doctor ask about site of pain in backache..


a. location and duration help to know about biological origin
b. location and duration help to know about biological outcome

264- common cause of clinic visit in pt with chronic kidney disease?


a. Annually routine blood test
b. oliguria

265- pre auricular tenderness with fever and headache..


a. chlamydia inf.
b. gonorrheal inf.
c. influenza inf.
d. coxsackie virus inf.

266-40 yr old man diagnosed with htn….advise


a. restrict salt intake to 4g
b. do heavy exercises
c. avoid fruits and vegetables
d. avoid organ meat..

267- Glipizide mechanism


a. Increase secretion of insulin from pancreas
b. Increase sensitivty of cells to insulin
c. Others choices

268- pt with diabetic nephropathy , u have to assess the severity of disease…


a. 24 hour urine protein
b. albumin creatine ratio
c. urine albumin
d. ketones in blood

269- corneal scarring….what to advise…


a. topical antibiotic
b. systemic antibiotic
c. topical steroid
d. systemic steroid

270- pt exposed to mosquitoes, have high grade fever, rash appears over the
body , splenomegaly , platelet count 40,000. causative organism spread by
a. aedes egyptii
b. female anopheles
c. sand fly
d. tick

271- a child presented with fatigueability,...., IX showed low HB (picture of


peripheral blood film showing target cells), the phycisian ordered blood
transfusion, what else he should get:
a- methotrexate.
b- desferroxamine
c. penicillin
d. pneumococcal vaccine.
272- pt developed neurological symptoms after taking TB medication.. what to
give for decreasing these symptoms..
a. tobramycin
b. tenefovir
c. gabapentin
d. pyridoxine

273- scenario about pt with previous infective endocarditis, going for


urological surgery, it stated in the question high risk for IE and penicillin
allergic:
a. iv vancomycin and gentamycin
b. oral vancomycin and tetracycline
c. oral ampicillin and ceftriaxone
d. im gentamycin and oral erythromycin
^ All answers are wronge
- https://www.sahealth.sa.gov.au/wps/wcm/connect/a39de780436f24c2b95cbff
2cadc00ab/Surg-Ab-Prophylaxis-guideline-Appendix4%2BEndocarditis_v2.0-
ics-cdcb-20171120.pdf?MOD=AJPERES&CACHEID=ROOTWORKSPACE-
a39de780436f24c2b95cbff2cadc00ab-mHF1GNc
^Surgical prophylaxis for invective endocarditis

274- Lady with migraine,during pregnancy it improved.trying to reduce work


stress. whice is best accomplishment of preventing migraine attack.
a. biofeedback
b. beta blocker.
c. sumatriptan
d. 100% oxygen.
275- old pt with macule and papule around nose and cheeks, one small pustule
over nasolabial fold, skin red, flushy and telangectasias..
a. actinic keratosis
b. rosacea
c. tinea versicolor
d. seborrhea dermatitis.

276- which of the statement is true about exercise


A. Increase basal metabolic rate
B. Does not effect the waist fat
C. Not recommended for cardiac patient

277- Younger diabetic patient came with abdominal pain, vomiting and ketones
smelled from his mouth. What is frequent cause:
a) Insulin mismanagement
b) Diet mismanagement

278- x ray..…child with no cough, toxic looking, initial treatment…


a. antibiotics
b. refer to orthopedic dept.
c. intubation
d. reassure
279- Patient with Celiac disease doesn`t follow diet. Came to the physician
regarding this condition. If he doesn`t follow diet which part of intestine much
more influence?
a) Proximal small bowel
b) distal small bowel
c) proximal large bowel
d) distal large bowel

280- Child ate overdose of iron several hours ago, Iron level 700, best
immediate management:
a) Gastric lavage
b) Induce vomiting manually
c) Emetic drugs
d) IV Deferoxamine

281- sickle cell patient…mechanism behind pain and lethargy….long


scenario…
a. fatigability
b. vaso-occlusive crisis
c. decreased erythropoietin
d. hemorrhage

282- A boy who was bitten by his brother and received tetanus shot 6 month
ago and his laceration was 1cm and you cleaned his wound next you will:
a) Give Augmentin.
b) Suture the wound.
c) Give tetanus shot
d) Send home with close observation and return in 48 hours.

283- Drug used for bipolar, schizophrenia and schizoaffective disorder


a- olanzapine
b- clozapine
c- quitapine
d- risperidone

284- pt sees diplopia while viewing right or left side…..where is the leision…..
A. CN 2
B. CN 3
C. CN 4
D. CN 6

285- tension headache…band like…

286- An old man using 6 hour computer/day presented with red eye itching
foriegn body sensation reason in his eyes, came to u, ur reason for this
condition….
a- reduce tear film
b- viral keratitis
c- cataract d uveitis

287- case of bronchiectasis…..lung fibrosed and the patient very well knows
that some part of his lung is permanently damaged…what modality of
treatment r u going to offer him…
a. lung resection
b. cpap
c. steroids
d. tracheostomy

288- clear scenario about HZV and asked about Tx:


Antiviral therapy – acyclovir

289-Q about effect of HTN OR DM on kidney ( not sure)


290-Q about about investigation in syphlis

291-Q Tx of septic arthritis


First step aspiration then blood culture and broad spectrum Abx
Staph aureus MC
Tx: empiric IV abx ( vancomycin and ceftriaxone)

292-Q about osteoporosis


293-Q about prevention of traveler diarrhea

294-Q about hallucination Ask u about Dx (Clear scenario)

295-Q about cervical spondylosis

296-Patient k/c of HF & schizophrenia on medication, came with brownish


discoloration in retina & brown vision.. what's the drug?
Thioridazine

297-A patient with eye movement abnormalities and ptosis. What is the nerve
is involved?
a-3rd
b-4th

298-increase the effect of analgesia?


A- Metoclopramid

299-Pt with shock & Cherry red skin;


A. Bacteremia
B. Septicemia
C. Carbon monoxide toxicity

300-patient ate from a restaurant. 2days after that started to complain of


diarrhea, vomiting , and urticaria.
a. Food poisoning
b. Food allergy

301. A patient complaining of severe uncontrolled HTN, “Renography” not


sure”, showed Lt renal artery stenosis, the next step is to?
A Venography
B- IVP
C- CT angiography
D- Renography
302. -patient presented with hemoptysis and signs of nephropathy. Biopsy of
the lung showed presence of anti-GBM antibodies. What is the most likely
diagnosis?
A. Rheumatoid arthritis
B. Systemic lupus erythematosus
C. Goodpasture syndrome
D. Acute glomerulonephritis

303-Which in lipid profile is the most important risk for coronary heart
disease?
A. LDL,
B. HDL,
C. Triglyceride
D. total cholesterol

304-4-the most effective non pharmacological treatment for hypertension??


a-Low sodium diet
b-Decrease Wight

305-post MI patient within 6 hours what is the most expected complication :


A.PE
B.arrhythmia

306-Patient came with ECG of narrow complex tachycardia with no P-wave and
regular rhythm what most important to test?
A) TSH
B) EPS
C) Coronary angiogram

307-The best way for lifestyle management for Hypertensive patient:


A) Na restriction less than 6
B) Weight loss
C)Exercise
D) Swimming every day

308-pt has HTN control on his medications, developed albuminuria, what


should you add to his HTN medications :
A) ACEI
B) BB
C) Lasix
D) digoxin

309-MI patient has big thrombus in left coronary artery what is drug that cause
throm- bolytic action?
A) TPA
B) Heparin
C) Clexane
D) Dabigatran
310-with MI he is on nitroglycerin and morphine . After 20 minutes he
got worse with raised JVP and no lung finding. BP drop.
A) Ruptured
B) Arrhythmia
C) RV infarction

311-A 50-year-old male diabetic and hypertensive, unable to maintain an


erection, start- ed on Phosphodiesterase type 5 inhibitors. Which one of the
following drug classes should be avoided?
A) Steroids
B) Antibiotics
C) Narcotics
D) Nitrates

312-Elderly Patient presented with chest painhe is a smokerwithpositive family


history of coronary artery disease and he is dyslipidemic he is also overweight
and can not tolerate exercise his ECG is normal what will do:
A. Stress echo
B. Stress test
C. Resting reperfusion scan

313-Which of the following is side effect of atropine:


A. Vasoconstriction
B. Decrease iop
C. Decrease urine output
D. Dry of mouth

314-Reflux esophagitis , which ding may aggravating this ?


Theophylline (anticholinergic)

315-Q about gastroenteritis.... Widal test + ve, which of the following cell will
be effected?
payers Cell

316 --Pt on ttt for meningitis with antibiotics, came later with lab result [all CBC
decreased ] ( this is a plastic anemia ?
Chloramphenicol

317-In window of hepatitis B which test is (+ve) ?


Hep B c, antibody ve Igm

318-Elderly pt with pelvic pain + normal ROM pain with abdication (AVN) , your
next step ?
Radiological image

319-Pt recurrent abnormal position or movement and no one can change it


what is Dx?
Catatonia

320 - 22-year-old male drug addict, visits the hospital several times with
multiple
complaints that are exaggerated. When he is not focused he seems
asymptomatic?
Malingering

321. Pt on medication for HTN that , co execration ?


Hydrochlorothiazide

322-Pt with high hypertension lab [metabolic alkalons + hypokalemic +normal


na] ur next step ?
Kidney us

323-Pt with recurrent hemartharosis , lab [factor Vlll =0.2 normal range (50-200)
what is ttt?
Factor Vlll , this sever hemophilia A (1% -5%)
Mild hemophilia, > 5% ttt davab

324-Pt with headache lab [ESR] Dx?


Temporal arteritis

325-Pt pain in eye + redness + lymphocyte , Dx ?


Orbital cellulitis

326-Elderly eat fibers + change in bowl habit , came with anemia microcytic
hypochromic , what is the cause ?
Change in dite [ high fibers cause iron defainanemia ]

327-60 yr pt come with anemia normocytic normochromic anemia [MCV ,NL


(80-100) , LDH ], what is the type of anemia?
Hemolytic anemia

328-Elderly with symptoms(renal cell carcinoma ) Fremantle cell carcinoma (


hematuria , abdominal mess , pelvic or lumber pain) what is the best test ?
CT

329-Pt on antilipid drug (hyperlipemia drug) come with flushing face , with
aspirin , lab [HDL , LDL - CK] Witch drug cause this ?
Niacin

330-Pt with symptom + Alzheimer + sometimes be agitation, what is the


cause?
Alzheimer

331-Fixed belief against culture and intelligence, definition of what?


Delusion
332-Man want to do study on group of things about lung cancer what is the
type of the study ?
Meta analysis

333-Case control study what is its bias ?


Recall bias

334-Relative Risk ?
A/(A+B) / C / (C+D)

335-Pt sudden fell down no breathing his pulse is raped and weak , next step?
Intapation

336-Pt getting better after naloxone , toxicity from ?


Morphine

337-absolute eosinophilia, what is the cause?


Parasite / Schistosomiasis

338-Case of TTP what is the response for it , 27 girl came with less of
consonants of seizure + hematuria + petechial rush + lab [throbocytenia] ?
ADAMTS13

339-14 yrs before one week has URTI now come with non-blanchable rash ,
what is the cause ?
Immune thrombocytopenic purpura

340- DM patient with bilateral lower limbs pain with crusts?


stasis dermatitis

341-Elderly has swelling in metaphalogeal and proximal interphalngal at


morning and with walking. Dx?
Rheumatoid arthritis

342-Sulfa drug come with blaster

343-Pt on short acting B2 against (salbutamol) + in healed steroid also have


recurrent attacks asthma on clinical give broncho inhaler (bronchodilator)
penk expiratory flow rate (PEF1) increase from 60% to 90% , what is ur next
step ?
Observe how to use inhaler

344-Pt with back pain CT show spondylitis change (spondylitis) ttt?


physical treatment
345-Pt with DM on metformin controlled everything is ok , his BP measured 3
time was 138/89 what's RX ?
Add ACEI
Gold of BP in DM according to AHA Below 130/90

346-Apoptosis witch gene ?


A.P53

347-Pt work on cotton comeback with shortness of breath + lymphadenopathy


:
A.Byssinosis

348-When to repeat -ve HIV test ?


After 3 months

349-Pt with dehydration +BP 80/80 what is the first step?


Fluid

350-Mom just eat polished rice , what is the deficiency?


Vit . B1

351-Bacterial meningitis lab ?


high Neutrophils, high protein , low glucose

352-Pt with migraine she doesn't want daily drug , RX ?


Triptans

353-Female has painful period she use NSAID she want stronger medication
what to advice her ?
Exercise and relaxation.

354-Pt with headache same as tension headache band like + stress for one
month and use
paracetamol day after day . What is type of headache?
over use drug headache

355-Which of the following potent antipsychotic drug?


Haloperidol

356-Witch of the following cause metabolic syndrome +weigh gain ?


Olanzapine

357-Doctor diagnosis pt with schizophrenia to Dx


it must duration be 6 months

358-Pt with symptom of schizophrenia (hallosenation) what to give ?


Risperidone (antipsychotic)
359.year old pt with vasomotorrihinitis and chinorrhea, ttt?
A. Ipratropium
B. Fexofenadine

360-Pt with symptom of otitis media redness bulging tympanic membrain, ttt?
A. Amoxicillin + calvulanic acid

361-Complication obstructive sleep apnea ?


A. DM
B. HTN

362-Case about adjustment disorder what is ttt?


A. Supportive therapy

363- OCD , what ttt?


A. Exposure and prevention therapy

364-Young pt with discharge from unilateral nose + foul odor , ur next step ?
A. X-Ray for head and chest

365-Pt with periorbital and on nose rash and painful (Dx, herpes zoster
ophthalmicus , ttt?
A. Acyclovir and refer ophtha

366-Pt with RA treated by INFH witch disense can recurrent to him when use
this drug ?
A.TB

367-Osteoporosis, best exercise?


Weight beaning exercise

368-MG come to ER , he is on pyriodostigmine what to give him in ER ?


Plasmapheresis

369-Female over thinking with tachycardia when she relaxed , tachycardia and
anexiaty, what
cause ?
Norepinephrine

370-Old pt asymptomat + fatigue lab [CLL] what is ttt?


No treatment

371-Pain and discharge from eye when pt cough , why?


Mechanical pressure in retina

372-Pt DM came with hemoptysis and chest pain (infection) found to have non
septette hyphae
fungal, witch fungal non septite ?
Zygomycosis
373-Pic of swelling redness painful in upper eyelid .Dx?

Styes

374 -Atopic dermatitis best thing to Dx?


Clinical

375-21 year with asthma with ulcers on knee or other site Dx?
Atopic dermatitis

376-Parasite and TB witch cytokines response?


IFY

377-Reservar for HIv ?


Macrophage

378-Pt on NSAID for 2 weeks come with medium and epigastric pain or upper
GI bleeding what is the cause?
Peptic ulcer

379-Hemeplagia and straipsnis or esotrapia , cause?


Pons

380-Pt with DM witch medication to give before sleeping?


MPH ( milk protein hydrolysate )

381-End stay liver disease + fungal infection


capsofungin

382-Energy in and out of cell


GTP cytoplasmic

383-Poor healing in DM due to


in decreased phagocytosis

384-Pt on ACEI not control what to add ?


CCB

385-Mechanism of action metformin?


Phosphatase kinase
386-Pt on metformin and sulfonylureas but not control, what to add?
A. acarbose
B. Insulin

387-Pt allergy to sulfa what to add ?


DPP 4 inhpotor (-tide)

388-Pic of corneal abrasion , asking about the dye


Fluorescent

389-Complication. of strabismus?
Amblyopia

390-Pt with problem in mandible?


great auricular nerve

391 - loss of sensation in out 2/3 of tongue


Trigeminal nerve
Facial is for taste anterior 2/3

392-Asthmatic. pt present in the ER witch investigation indicate security of


asthma?
O2 pressure

393-High myopic ttt?


A. Keratotomy
B. Phakic intra oculor lens

394-Case give lab and ask about acid base balance ?


Normal aniongab metabolic acidosis ^^^ more details

395- Myasthenia Gravis mechanism?


Antiacetylcholine receptor antibodies.

396- Pt with history of gastroenteritis took medication. Now came with


abnormal movements in head and eyes. Cause?
Metoclopramide.

Metoclopramide may be the most common cause of drug-induced movement


disorders specially dystonia and oculogyric crisis. Treated by Benzotropine or
Diphenhydramine.

397- Elderly female came with weight loss + constipation + pelvic mass. best
initial screening?
US.
Suspected Ovarian carcinoma so the first step in any abdominal mass is US.

398- oll lady known case of hypothyroidism present with painful movement of
the right shoulder and can't raise the shoulder due to this pain. What's the
most likely diagnosis?
Adhesive capsulitis.

399- Which of the following primary cause of osteoporosis?


Aging.

400- Case of osteoporosis, ttt?


Alendronate (Bisphosphonate).

401- Type of lung cancer that is related to keratin?


SCC.

402- Pt with hypothyroidism and vitiligo, lab showed hypocalcemia , normal


po2. will cause?
•Panhypopituitarism.

403- Pt with COPD and CO2 9. What u will do? ❓


A. Mechanical ventilation.
B. Reduce O2.
C. Antibiotics.

404- 55 Years old with AF, what u wll do to reduce the complications?
Warfarin.

405- Pt with atrial fibrilation on warfarin develop inracranial heamorrage what u


wll do?
A. Vitamin k.
B. FFP.
C. HEPARIN.

406- Pt developed orthopnia PND, chest auscultation basal crepitations, she


deliverd one week ago, what is your diagnosis?
A. Peripartum heart failure.
B. IHD.
C. Asthma

407- Asthmatic c pt have no exacerepaon in last mounth, his dialy PEF is 600,
what is indicator of sevirity in this pt?
A. RR 25.
B. PFR 240.

408- Old pte admited becouse of pneumonea at night she develop restlessness
she is disoriented what u wll do physicaly to but her in her bed?
A. Call her family to set besid her.
B. Lorazepam 4 mg stat.

409- Pr presented with some coplaint, potassium wass 7.9. What u will do
next?
A. Dialysis
B. Insulin.
C. Ca gluconate.

410- Pt on dialysis developed shivering he mentioned history of fever 3 day


ago, you sow pus drainag from the catheter, what u do?
A. Take blood for culur and give AB.
B. Blood culure ABand stop dialysis 3 days.
C. Change the catheter.

411- Pt with URTI 3 days ago, develope shortness of breath, PND, LLedema,
troponin is high, what is apropriat step to reach the doagnosis?
A. ECHO.
B. CXR.
C. MRI.

412- Hydatid cyst treatment?


A. Albendazole.
B. Metronidazole.

413- ameobic liver abcess treatment?


Metronidazole.

414- Pt has cafe alue spot in his back what you will do next?
A. Council the pt about neurofibrmatosis.
B. send the patient to genteics to do genetic.

415- pt has subcutanuoes nodule and arthrlagia what one of follwoong confirm
diagnosis of Rhematic fever?
A. High crp
B. high esr
C. postive blood culture

416- Pt has st elevaon in v1 v 2 v 3 what is your diagnosis?


A. mi.
B. pericardies.
C. cardiac tamponade.

417- Pt age of 4 years convulsing at home for 3 minute he has tonsillitis, temp
is 38 what you will do next?
A. Diazepam.
B. lorazepam.
C. amoxicillin.
D. paracetamol.

418- Copd excarebaon and co2 retenon and consious.


NIMV = Noninvasive motion ventilation

419- Unconscious COPD exacerbation


A.mechanical ventilation

420- Pericarditis first drug to give....


A. non steroidal anti-inflammatory drug like ibuprofen

421- Celiac disease should avoid .


A. gluten contain diet like u should know the type of food contain gluten

422- Hypernatremia with normal potasium .....


SIADAH XXXXXDM Water retention

423- a myocardial biopsy was taken from woman because of unexplained


congestive HF: biopsy showed Aschkow cells:
A. rheumatic fever
B. SLE
C. RHD

424- Sever headache ever...


A. subarachnoid hemorrhage

425- Patient with fatigable weakness( sx of MG), what’s the pathophysiology of


this condition:
A.Antibodies against Ach receptor

426- Patient with DLP on statin, lab results for lipid profile all within normal
range except high alanine
aminotransferee (48) ‘normal up to 40’
What you’re going to give this patient:
A- Fibrate
B- Cholestyramine
C- Ezetimibe
D- Omega 3
427-Patient with sx of gastroenteritis:
A-Rota
B-Norovirus
C-Adenovirus
Adult = nora virus
Pedia = Rota virus

428- Qs about staph aureus food poisoning


Within 6 hours

429- worse prognostic gene with HCM


A-Myocin binding protein C
B-Beta myocin

430- Case presentation of Addison, what u will find:


A. Hirsutism
B. Hyper pigmentation
C. Buffalo hump

431- Patient with MG crisis:


A. plasmapheresis

432-Case presentation of hyper aldosteronism, what u will find:


A.Polyuria
It is typically due to hyperplasia or adrenal adenoma ( MC )
433- Patient with back and buttocks pain that improves with exercise:
A. Riter syndrome
B. Reactive arthritis
C. Ankylosing spondylitis

ankylosing spondylitis: pain in low back and gluteal region, insidious pain with
increased pain and stiffness in the morning, pain decrease with exercise and
increase with rest.

434- End distention of protien


A-ER
B-cytosome
C-golgi,
D-lysosome

435-Patient coming for general check-up his BP is normal and has family Hx of
cardiac diseases, what u will do for him
A - Lipid profile

436- patient coming to see the results of INR, what the drug he is taking:
A - Warfarin
437- yrs old male Patient with DM and HTN and hx of 2 stroke events in the
past. Coming with Afib. What u will do for him:
A - Warfarin to prevent any stroke event

438- yrs old male came for general check up, BP and Fast blood glucose are
within normal ranges his BMI is 28 and he is asking about ur advice:
A -Wt reduction and exercise
B - Exercise
C-Low Salt diet

439-Patient with bph and HTN, what u will do for him:


A- Alpha blocker
B- TURP
C- Open prostatectomy

440- patient on methtroxate, what would be deficient in him:


A-Folate
B-Niacin
C-Vit E

441-Case of rheumatic fever (A 7yrs old boy presented with knee pain. On
examination, he has heart murmur. His parents told you that he had tonsillitis
3 wks ago and treated with paracetamol. His ASO titer is high. How u will treat:
A- Penicillin
B- steroid->-> sever carditis
C- aspirin
442- patient presented with SOB on exertion and pallor. His CBC showed low
Hb, normal MCV and high LDH
A- Hemolytic anemia
B- Microcytic

443- patient with jaundice after quinine treatment. What the deficient enzyme:
A- Glucose-6 phosphate dehydrogenase
B- Glucose dehydrogenase
C- Glucose 6 kinase

444- patient his bone marrow bx showed 80% blast cells and Auro rods:
A- AML
B- ALL
C- CML
D- CLL
AML = Auer bodies + adult + presence is myelperoxidase and esterase
ALL = in children ( Down syndrome MC )
CML = Philadelphia chromosome + LAP
CLL = asymptotic , elderly , smudge cells , CD19 , CD20

445- most oxygenated zone of the liver is ->


A- Zone 1 - periportal (centroacinar), around the portal areas
B- Zone2 - midzonal
C- Zone 3 - centrilobular (periacinar), bordering the hepatic venules
Zone 1 oxygenated
Zone 3 hypoxg

446- patient with presentation of generalized anxiety disorder, what is the best
drug for her:
A- Lorazepam
B- Bupropion
C- Buspirone
D- Citalopram > SSRI

447-Fibroid comments in
A. african race

448- Increase total cholesterol + high LDL + high triglyceride + low HDL ... ask
about the follow up after 1 year
A- Blood pressure
B- Lipid profile

449- On antidepressant suicide bottle beside her has dilated pupil:


A- amitriptyline
B- Baro receptor activation will
C- decrease venous capictance

450- Monitor of asthma in ER


A- FEV1
B- Heart rate
C- partial pressure of oxygen

451- After stopping the OCP, pt sll have amenorrhea for 6 months, gain 5 kg,
feel tired. labs show elevated glucose
A- Hypothyroidism
B- Cushing syndrome
Estrogen increases cortisol binding globulin,

452-Pt with COPD and CO2 90 what you do?


1-mechanical ventilation
2-reduce O2
3- antibiotic

453- 55 years old with atrial fibrillation, what you do to reduce complication?
Warfarin

454- Pt with atrial fibrillation and on warfarin develop intracranial hemorrhage,


what you do?
1-vitamin K
2- FFP
3- heparin

455- Pt develop orthopnia PND, on chest auscultation basal crepitation, she


delivered one week ago, what is your diagnosis?
1- Postpartum HF
2- Asthma
3- IHD

456- Asthmatic pt with no exacerbation last month, daily PEF is 600, what is
indicator of severity in this pt?
1- RR 25
2- PFR 240

457- Old pt admission because pneumonia, at night develop restlessness and


disoriented, what you do?
1- Physically but her in her bed!!!!!!!!!
2- Call her family
3- Lorazepam 4 my

458- Pt presented with some complain potassium was 7.9 what u will do
• dialysis
• insulin
• Ca gluconate

459- Pt on dialysis develop shivering he mention history off ever 3 day ,you
sow pus drainage from the catheter what u do
• Take blood for culture and give AB
• Blood culture AB and stop dialysis 3 days
• Change the catheter

460- Pt with URT 3 days develop shortness of breath PND LL edema troponin
is high what is appropriate step to reach the diagnosis
• echo
• ecf cxr
• MRI

461-2questions about diabetic keto acidosis management?


Fluids >>Then insulin >>Electrolytes correction (k+)

462-case about syphilis, ask about diagnosis


primary >> Dark field If secondary >> treponomal (MHA, FTA)

463-ECG,types of heart block


464-Crecendo acrecendo murmur cause
Raduated to neck = AS

465-Pleural effusion analysis and management


By light’ criteria Ttt by drainage +Abx

466- Diabetic and heavy smoker. With 100 meter . Pain when go to mosque in
left leg. During investigation. Found the right poplitibia artery insuficiant . Best
to do?
Smoking cessation

467-Patient with vitiligo . Low HB. MCV high. ?


Pernicious anemia /leukemia

468-Antibody to intrinsic factor?


b12 level

469-RA Patient with right and left knee arthritis. What the type of RA ?
If less than 5 joints so oligo (pauci) JRA

470-Empirical treatment of invective endocarditis. >


Vancomycin and Gentamycin
471-RTA. With Low urin osmolality . High plasma osmolality.
central DI

472 -Phentolamin in the treatment of pheochromocytoma. Act by?


Alpha

473-Treatment of HB is example for?


A-Pimary prevention
B-Secondary
C-Tertiary
D-Primou
Primary = vaccination
Secondary = screen
Tertiary = treat to prevent complications

474-Adrenal hyperplasia. Best investigation


CT

475-IBS treatment
Symptomatic If no effective give TCA antidepressants

476-Patient. With eye symptoms. Lower limb spasticity.what is the best inv to
reach diagnosis
MRI

477- Patient on anti TV medication and has prepared sensation of needle


.drug? Pyrodoxine vit b6

478-patient with abdominal pain release with defecation. diarrhea 6month. No


wt loss . No anemia.diagnosis? Microscopic appearance of celiac disease
IBS
479-Severe Mortal stenosis. ? Valve gradient.
Less than 1 cm2
3
4

480-Status epileptic, ttt?


IV diazepam, if no response phenytoin Then phenobarbital then GA

481-Fever Lowe eye lid swelling redness .


A-Low protin
B-Abscess

482-History of bloody diarrhea .

483-Counselling to TB patient. ?
A-Contact
B-Air born

484-Meningitis case with skin rash. ?


Airborne
Contact

485-Pulsating abdominal aorta,DIAGNOSIS?


CT

486-Chang between tow arm pulsation . Best diagnostic?


Coarctation of the Aorta

487-Right heart failure. What medication show to be effective?


Betablocker

488-Heart failure. Best medication .?


1.Betabloker and hydralizine
2.acei and diuretic
3.beta blocker and diuretic
4 betabloker and nitrate
489-Lower and upper limb flaccid..and arefleixa after URTI.diagnosis?
GBS

490-Alarming sign in stridor patient?


Cynosiis

491-Low hb and murmur.how to treat this murmur?


A-Treat the underlying cause which is the anemia.

492-Patient with AF has lower limb ischemia .Treatment?


A- LMWH
N.B: This is acute embolic limb ischemia due to emboli cuz patient has AFib
N.B: First step in ttt is Heparinization with 5000 IU bolus then we will decide
according to the neurovascular status;
**If intact pulse so CT angiography and observation.
**If impaired or absent pulse so emergent revascularization by embolectomy and
fasciotomy.

493-What is the leading cause of death in hemodylisis patient?


A-Arrhythmia due to hyperkalemia.

494-Pulmonary symptoms and raised jvp hepatomegaly and lower limb


swelling.diagnostic test?
A-ECHO.
N.B: Right side heart failure case, diagnostic test in heart failure in general = ECHO

495-Case with pancytopenia ANA+ve and arthritis.diagnosis?


N.B: Most likely this is SLE.
N.B: Felty syndrome is a triad of splenomegaly + leukopenia + arthritis + positive
RF and antiCCP antibodies.

496-Rt side heart cathetrization used to diagnose?


A-Pulmonary HTN.
N.B: Initial test for pulmonary HTN is ECHO, confirmatory test is Cardiac cath.

497-Post MI bradycardia treatment?


A-Atropine.
N.B: If the question about heart block so yes Atropine and pacing.

498-Smoker for long time came with couph white sputum.. FEV1 less than 70%
what is the diagnosis?
A-Chronic bronchitis
B-Bronchial asthma
C-Lung fibrosis
N.B: Chronic smoking is the most common risk factor for COPD ''90%''
N.B: If FEV1 and FVC are low so will look for the ratio;
If ratio low so COPD or BA ''obstructive pattern''
If ratio normal or high so Pulmonary fibrosis ''restrictive pattern''

499-ln child what is the symptoms that indicat there is disease?


A.Pain around ambalicus
B.Pain at night when patient sleep
C.Pain when patient awake

500-Patient with history of chest pain since 4 week ago .increasing . In rest and
movement.diagnosis?
A-Prizemental
B-Unstable angina
C-Heberden
D-Stable angina
N.B: Chest pain that is gradually increasing and initially on exertion but now on rest
so this is unstable angina.

501-Patient with systolic murmur, ecg show undetermined(non spacific) T


changes and St.diagnosis?
A-AS
B-Obstructive cardiomyopathy
N.B: Q depends on murmur if radiated to neck = AS.
N.B: If murmur increased with valsalva maneuver + FH Bof sudden death + Dagger
like Q wave on ECG = HCM.

502-Type 2 DM . What medication decrease cardiovascular disease?


A- Atorvastatin
B- Fibrat

503-High cholesterol and High ldl. Treatment?

504-Patient com with convulsion . Lymph node . Rash?


A-HIV serology

505-Picture of Urin dipstick test. +2 proteinurea And scenario about pregnant


female with bp140/90.diagnosis?
A-Mild Preeclampsia
506-Patient taken throxin for hypothyroidism.TSH high, T3 normal, T4
normal.explain?
A-Subclinical hypothyroidism

507-Which group of drugs is contraindicated in diabetic patients with heart


failure?
A. Biguanides
B. Dipeptidyl peptidase IV inhibitors
C. GLP-1 analogues
D. SGLT 2 Inhibitors
E-Thiazolidinediones
N.B: TZD drugs like Pioglitazone will cause fluid retention.

508-child suffer from tonsillitis, you examine the patient and you find whites
patches on it, what is the management?
A- Penicillin

509-man diagnosed with Hep C , the couple ask you what they do:
A-Abstinence
B-Use condom
C-Do intercourse with no restrict
N.B: HCV mainly transmitted by blood but also can be transmitted sexually.

510- Child in Er DX as DKA managed by fluid and insulin ,serum electrolytes


will be imbalanced
A.Ca
B.K
C.Na
D.Mg
N.B: Hypokalemia is one of DKA manifestations.

511- obese has sretch marks in skin ane puffallo hump. Your diagnosis
A.chushing
B.chines
C.pheochromocytoma

512- Pt have GERD, he take PPI for one month, after that his symptoms
returned, investigation?
A- 24 h pH monitoring test
N.B: The gold standard test to confirm diagnosis of GERD is by 24 h pH test.

513- 40years old male, he admitted to ER with convulsion, not response to


diazepam, what the next atep?
A- oral phenytoin
B- IV phenytoin

514- HBsAG -ve, HBsAb +ve, HBc +ve , your diagnosis ?


A- immediately of vaccination (Only positive HBsAb)
B-past infection (positive HBsAb and HB core IgG antibody)
C- acute infection
D- carrier
N.B: Vaccination (Only positive HBsAb)
N.B: Past infection (positive HBsAb and HB core antibody)

515- 32 years old male , non pitting edema , diagnosis ?


A- bilateral DVT
B- lymphangitis
N.B: DVT is unilateral pitting edema.

516- Pt complain of lower mandibular pain with loss of sensation ?


A- great articular
B- ophthalmic

517- Pt with history of asthma on SABA, need to go 2nd step but she refuse
that, what you do?
A- increase does of SABA
B- add LABA
C- tell pt how take steroid and benefit

518- Pt complain from discharge ear purulent?


A-Otitis media with ruptured drum

519- Pt coming from sudan to KSA need to prophylaxis?


1- Choloquine for malaria prophylaxis
2- Fansidar
3- Quinie

520- The first line of P.Falcipram in KSA ?


1-quinie
2-fansidar + artesunate

521- Long sinario Pt c/o with long time of vomiting, development of blood, no
chest pain?
1-esophageal rupture
2-Malary Weis syndrome
3-peptic ulcer

522- Pt diagnosis with leukumia on chemotherapy, what aboute leuctrolyte ?


1-hypocalcemia
2-hyperca
3-hypona
4-hyperna
N.B: Hypocalcemia due tumor lysis syndrome

523-Pt old in ALHAJ, c/o body rash, with neck pain and fever, wbc12 hg11
lymphocyte low , how can transport this disease?
1- Airbourn
2- Droplet

524-Pt c/o of fever and cough lymph node enlargement?


1-RSV
2-influenza
N.B: Q is not complete, might be EBV if mentioned splenomegaly.
N.B: Acute bilateral cervical lymphadenitis is the most common infectious neck
mass, caised by entero, adeno and influenza viruses.

525-Pt asthmatic for longtime, IUGR, and do induce of labor, what the drug
contraindication with her?
1-sintocynone
2-methocholine
3-PGF2 alpha

526-Woman with main cause of nephrotic syndrome?


1-minmal change disease
Nephrotic syndrome
Child = minimal change disease
Black adult = FSGN
Whites adult =MN

527- Long scenario with symptoms of Cushing syndrome, in end of the


scenario say they do renin 24 ratio it's double result, what the diagnostic test?
A. CT of adrenal
B. brain MRI

528- Patient with hypercholesterol what food he shouldn’t eat?


A. Avocado
B. Chicken
C. Brown meat
D. White part of the egg

529- Mushroom inhibit which enzymes?

530- The degree of obesity in woman based on the BMI, height 154cm
weight150kg
Obese grade 3
531- The treatment of leishmaniasis?
Pentosum

532- An elderly complaining of bilateral shoulder pain and now bilateral hip
pain What is the diagnosis?
A. Rheumatoid arthritis
B. Myalgia
Polymylgia rheumatica: bilateral shoulder and hip pain

533- Patient complaining of face swelling when sleeping and it goes after he
wakes ,also he is smoker,diagnosis?
SVC obstruction due to lung cancer

534-pic of ECG (heart block) what to give?


A. pacemaker
B. BB
C. warfarin

535- elderly pt long scenario of heart diseases and BPH, and new diagnosis
colon cancer and mets
A. paliative care
B. need of high care

536- Pt with long senario, severe fever and neck pain and rash, blood film
wbc13 high lympocyte, what transmission of this diseases?
A. airborne
B. droplet
C. contact

537- Pt, came with fever and blood of sputum, lymph node enlargement, how
to protect another?
A. by safety clothes
B. by face mask
C. control of airdrop
D. control of contact

538- A runner who feels pain that decreases in the day?


A. Plantar fascitis

539- Patient with posterior nose bleed management ?


A. Posterior packing
B. Vasoconstriction

540-Pt came from Sudan ( with all symptoms of malaria ) blood film done and it
was -ve
What you’ll do ?
A- thin blood film
B- thick blood film again
C- thin blood film every 8 hours until 2 day

541-Case of foot hand mouth (pic) And clear symptoms . What else you’ll see
?
A- rash of foot

542-Elderly pt with Alzheimer came with agitation, what you’ll give ?


A- Diazepam
B- haloperidol

Resperidone is treatment of agitation in elderly

543- when you cover the right eye (cover test) , there is movement in the left
eye What’s the worst complication
A- nystagmus
B- strabismus
C- amblyopia

544-Female 41 yrs want to get pregnant, lab done (high LH, high FSH) More
prone to what ?
A- endometrial cancer
B- osteoporosis
C- ovarian cancer
High FSH indicates ovarian failure > menopause > low estrogen > osteoporosis

545-Case of H. Pylori (adult pt) urea breath test +ve and he on PPI +
metronidazole
What you’ll add ?
A- clarithromycin

546. Case of H. Pylori (child pt) urea breath test +ve


What’s the organism !

547-Case of CP with lower limb spastic and upper limb spastic (less than
lower)
A- quadriplegic
B- hemiplegic
C- diplegia

548-71 yrs c/o urine retention, stomach distention and severe pain with
urination
A- foly catheter + culture

549- Cortisol at 8am norma level (after given oral dexamethasone1mgatnight)


wt investigation to do?
A-Brain MRI
B-CT for adrenal gland

550-Pt diagnosed acromegaly and started ttt with octreotide what the
investigation of choice in the future?
A- Echo
B- CT abdomen
most common cause of death in such patient cardiac disease after that cance r

551-Symptom of cough and shortness of breath after exercise whats high


diagnostic value to reach diagnosis?
A- CXR
B- PEFR
C- Methacoline test FEV1 decrease to more than20%
D- Increase to 5% after albuterol inhalor

552-Asthmatic pt take SABA and inhaled steroid but there is no response what
is the next step
A/LABA
B/ systemic steroid
553-There is left parasternal heave and early diastolic murmur and pistol shot
at femoral artery what is the diagnosis ?
A/ Aortic regurgitation
B/ HOCM

554--Case about HBV serology and give you HBsAg_ve, AntiHBs+ve ,


HBcAg+ve
IgM _ve, what is the diagnosis
A/Acute HBV
B/previous infection of HBV
C/chronic HBV
D/previous immunization

555-Long scenario about old female see cars and other things in her plate
what is the diagnosis
A/ dilusions
B/ halucination
C/ ilusions
A visual hallucination is a perception of an external visual stimulus where none
exists. By contrast, a visual illusion is a distortion or modification of real external
visual stimuli

556-Case of meningitis and ask about the of organism Protein increase,


Glucose decrease
A/ bacterial meningitis
B/viral meningitis
557/ pt complained of Rt upper quadrant pain and the direct bilirubin is high
what is th diagnosis
A/ cholengitis
B/ Gilbert syndrome
C/Hepatitis
D/ cholecystitis
558- Pt c/o of anal pain and bleeding and she had history of constipation , on
Ex there is boggy appear at anus what is the diagnosis
A/ fissure
B/ hemorrhoids
C/ thrombotic piles
■ thromboses hemorrhoids are very painful
resolve within 2 wk, may leave excess skin = perianal skin tag
treatment: consider surgical decompression within frst 48 h of thrombosis, otherwise
medical treatment

559-Population screening
A. Oral cancer
B. Pancreatic cancer
C. Colorectal cancer
three national population based screening programs
Breast Screen
Bowel Cancer Screening
Cervical Screening

560-23 yrs old with forearm lesion since chilhood 1×2 cm what to do
A. Laser
B. Excision
C. Radiation
D. Followup

561-Age of screening of colon CA



A- 50
B- 40
C- 60
D- 70

562-Case of Sarcoidosis in xray there is filtration and lymph node?


Bilateral hilar

563-Erythema nodasum located in fabila and tibia

564-Congestive heart failure increase survival by



A- ACEI


565-pt came to ER with rash and blood in urine found sign of HSP but not
found it in investigation
(hypersensitive evasculitis)

566-pt of SLE came with severe joint pain?


A-methotrexate

B- cyclophsomide

567-pt complain of erection, Thyroid hormone normal


A- hyper pitutary
B- hypopituitary
C- macr prolactinoma
D- micro prolactinoma

568-GERD

A- endoscopy
B- PH depended monitor

569-(long scenario ) esophagus with metaplasia with high grade what you
give?
A- pendazole and randitadine

B- PPI
 if low grade + screen 3 mo
C- Sclerotherapy, resection


570- Hot potato voice, what is the condition?


Peritonsillar abscess

571- Corneal ulcer treatment. ?


Antibiotics cycloplagia and areferral

572- Treatment of patient presented with herpetic lesion?


Acyclovir

573- A30 year old female,pregnant,complaining of hearing loss,what is the


condition.
otosclerosis

574-Otitis media complicated with effusion-> expectant 90% resolve by 3 mo


Myringotomy
575-Skin lesion with spindle cells and vascular diagnosis?
Kaposi sarcoma

576- 3rd layer of scalp

577- What layer has nerves and blood vessels in the scalp
Loose Areolar CT

578- Retinoblastoma=
absence of red reflex unilateral

579- What anti-depressant used for both kids and adults?


Fluoxetine

580- The most effective modality in bed wetting?


alarms,
desmopressin,
toiled before sleep,
mid night void
581- What do you call ice craving.
Pagophagia

582- ethylene glycol ingestion And kidney


ATN

583- Aspirin->
Respiratory alkalosis the metabolic acidosis

584- What goes with CLL diagnosis?


Reticulocytes,left sheft
Smudge cell in blood film

585- Sickler with pancytopenia, what is the infection ,


para19

586- Sickler with recurrent admissions presented with chest pain and bone
pain,management?
Reticulocytes
hydration and analgesia,
admission and analgesics

587- Ace inhibitors->angioedema

588- 6hrs post stroke management?


Thrombolysis
clopidogrel
aspirin

589- TTP case schistocytes and fragmented RBC

590- Meningitis after swimming organism


Naeglria (naugeria fowleri)

591- Cluster headache acute management?


100%O2
sumatriptan

592- Cluster headache prevention?


Verapamil

593- amitriptyline side effects


Cough and blurring vision

594- The most specific for rheumatoid arthritis?


Anti ccp

595- Decrease knee reflex what level?


L3-4
596- Transmission of pain from tonsillitis?
Glossopharyngeal
vagus
sphenopatine
nasopalatine
1- Parotitis + referred ear pain -> auricoltemporal N ( part of 5thCN)
2- parotitis or no parotitis + referred ear pain WITH eating or chewing -> posterior
auricular pain ( part of 7th CN )
3- tonsillitis + referred ear pain -> 9th CN “ glossopharyngeal N”
4- sensation posterior to ear or at the mandibular bone -> great auricular nerve (
branch of C2-3)

497- Choosing every 10th family?


Systemic selection

598- A harsh systolic murmur in the left sternal border in an elderly?


Aortic stenosis,
aortic sclerotic
pulmo stenosis

599- Which of the following will present with cyanosis?


New born with truncusbarteriosis,
4years withVSD,
12 with ASD

600- Cyanosis after suckling?


Choanal atresia

601- High JVP with basilar crepitation and lower limb edema,diagnosis?
cor pulmonale
liver cirrhosis
nephrotic

602- The reason of increased TP in developed countries?


Un diagnosed TB
untreated TB,
AIDS

603- The leading cause of death in KSA


CAD

604- Anaphylactic after abee sting->


give epi

605- 35mins patient with continuous seizure and benzo failed?


Give Phenytoin
606- Patient 12hrs post femoral cardiac bypass (Ithink this what it is called)
started have disorientation symptoms
Delirium

607- The mode of inheritance in NF?


AD

608- Anti fungal with end stage liver disease?


capufungin

609- Baro recpter activation will decrease venous capictance

610- pt sustained only one posture flexed right arm and left is straight,dx?
Dystonia or catatonia

611- Beta-lactmaise strep pyogne..what will you add to amoxicilli->


Clavulanicacid

612- Hyperthyroidism and now Recurrent sore throat+lowWBC what


medication she took?
Methimazole

613- Monitoring asthma in ER


PEF

614- Bleeding from umbilicus->>


Xlll
615- Treatment of cystic hepatic lope?
Liver cysts are fluid-filled sacs that form in the liver. They're benign growths,
meaning they aren't cancerous. These cysts generally don't require treatment unless
symptoms develop, and they rarely affect liver function

616- had family history of lymphoma in his brother and ....... what is the
important lymph node to be detected to diagnose lymphoma?
A/ suraclavicular lymph node
B/ periauricular lymph node

617- Tb and side effects of anti tb?


Heptitis
- Most hepatotoxic is pyrzinmide-

618- Woman patient with neovascular lesion in her lt leg when intervention
A. Pain
B. Claudication
C. Bad cosmotic
D. Congestive heart failure

619- pt came with fever vomiting &diarrhea rapid fluid replacement done pt
improve for awhile then she developed abnormal movement and
became comatose?
a)complicated with meningitis
b)rapid fluid replacement cause cerebral edema

620- A 73-year-old man presents pain in his right thigh. This has been getting
progressively worse for the past 9 months despite being otherwise well. An x-
ray is reported as follows:
X-ray right femur Radiolucency of subarticular region suggestive of osteolysis.
Some areas of
patchy sclerosis Bloods tests show: Calcium2.38 mmol/lPhosphate0.85
mmol/lAlkaline phosphatase544 u/LProstate specific
antigen4.4 ng/ml
What is the most appropriate action?
A. Vitamin D supplementation
B. Check serum testosterone
C. Referral to an orthopaedic surgeon
D. Referral to a urologist
E. IV bisphosphonates

621- Ttt of spherocytosis ...


folic acid and spleenectmoy

622- Female pt with her husband for fertility counselling .... 31BMI
All labs are good Regular menses Your advice
A. Keep physicaly active
B. Eat less fast food
C. Include 2 portion of vegetables and grains

623- A 60-year-old farmer is brought to the local emergency


department with his wife by the county sheriff. Apparently
he developed a “twinge of chest pain” while shoveling grain
3 hours ago. He insisted on staying home until “he collapsed on the floor.”
Even then, he wanted to stay home and rest, but his wife insisted on calling
911. The call was answered by the sheriff’s department, and a county sheriff
rushed him and his wife to the emergency department at the nearest hospital
in the rural area in which he resides. At his admission, he states that the pain
is “almost gone . . . what a fuss about nothing.” On taking of the history, he
tells the admitting physician that he smokes two packs of cigarettes daily, that
he drinks a “goodly amount of beer,” and that he had been told that his serum
cholesterol level is good; the doctor even told him his was one of the best
values he had
ever seen for a man his size and age. On further
questioning, he admits to a dull, aching, viselike pain
around his chest, with radiation to the left shoulder. He also
discloses that when the pain was at its worst, he experienced nausea and
vomiting; but he adds that because he is already doing better, it must have
been something he ate. His wife adds that she has never seen him in so much
pain, but he is a “stubborn old goat.” He still insists that it is just a little
stomach trouble, but on physical examination, he is sweating and diaphoretic.
He has vomited twice since coming to the emergency department. His blood
pressure is 160/100 mm Hg, and his pulse is 120 and irregular. His abdomen is
obese, and you believe you can detect an
enlarged aorta by deep palpation.
His electrocardiogram (ECG) reveals significant Q waves in V1 to V4, with
significant ST-segment elevation in the same leads. There are reciprocal ST
changes (ST-segment depression) in the inferior leads (II, III, and aVF)

The most likely diagnosis in this patient is:


a. acute inferior wall myocardial infarction (MI)
b. acute anterior wall MI
c. acute myocardial ischemia
d. acute pericarditis e. musculoskeletal chest wall pain

624- Given the history, physical examination, and ECG, your


first priority is to: a. call the ambulance for immediate transport to
another hospital that “knows how to treat this thing” b. admit the patient for
observation
c. administer streptokinase or tissue plasminogen activator (t-PA) intravenously (IV)
d. administer heparin IV

625- Smoker for long time Came with cough, White sputum, efv less than
70%,what is dx?
COPD

626-Female DM1 has numbness and burning sensation in feet she wish if
there's away to help her loss whight?
a-Aerobic exercise
b-Swimming
c- Jogging Traidmil

627- Patient with hyper cholesterol what food he shouldn’t eat?


a- Avocado,
b- chicken,
c- brown meat
d- white part of the egg

628- Patient complaining of face swelling when sleeping and it goes after he
wakes, also he is smoker, ?
bronchial carcinoma
-

-
-
-

-
-

-
-
- -
640- A 47-year-old woman presents to her primary care provider with
Raynaud's phenomenon for 3 years presents, puffy fingers, a fingertip ulcer,
and difficulty swallowing. Past medical history is unremarkable. The patient
does smoke but drinks alcohol socially. Physical examination of her hands is
shown:

Which of the following is most likely to be positive in this patient condition?


Anti-Scl-70 (anti-topoisomerase I antibody)
B. Anti-Smith antibody
C. Myositis-specific antibodies (MSAs)
D. Anticentromere antibodies

641-Patient trying to quit smoking, but he couldn’t, he came to you for help:
what behavior change at this stage?
A-precontemplation
b-contemplation
C-Action
D-preparation

642- Want to stop smoking, went to hospital and asked for treatment::
action

643-What’s minimum volume of blood to do a culture:

644-Which of the following reduce cancers?


A-Vit D
B-Fibers
C-Salt
645-Most common infection in saudi arabia and disable patient from work:
Hepatitis B infection

646-Treat water from entameba Drink boiled water to prevent it

647-Researcher want to measure obesity in children he took BMI what else he


should take?
1-girth measurement
2-HDL/LDL level
3-dietary habits
4-forgot

648-How to prevent plague


A-Killed rodent

649-Ministry of health, prevent some group of ppl to do Hajj & Umrah in 2015,
due to MERSA- Co; whose those population?
A- Lactating women
B. Young kids > 12 years
C. elderly with DM
D. HBV

650- inactive person, overweight, smoker, 44 years old man with heart
problems, What is the most common associated with heart disease:
A-Smoking
B-Obesity
C-Inactivity

651- how to prevent MERSA ?


A-by hand washing
B-vaccine.

652-The daughter of an old age patient complains of progressively decreasing


in memory and change in personality of her father, ttt:
A-Refer to geriatric clinic.

653-Important in Quit Smoking?


A-Patient desiree.

654-What is have more risk for CAD


A-LDL.
B-HDL
C-Triglycerides
D-Total cholesterol

655- best way to prevent stroke is:


HTN
657- commonest virus which make rejection of the expatriates or labors from
working in Saudi?( community :
hbv

658- In a village where the incidence of cretinism and iodine was less than 1
microgram the health promoters want to issue a director for the for the
management of those with cretinism.. what is the best initial management
a) TSH and t4 measurements
b) Start thyroxine medication.
c) Iodine supplementation.

659-Want to stop smoking, but now he wants to stop, which phase he is in?
A. Precontemplation
B. Contemplation
C-action
D-Preparation

660- Community medicine employee want to implement a campaign for a


leading cause of death in Saudi Arabia. What is the appropriate answer:
A-Breast cancer
B-DM
C-Coronary artery disease

661- 30-Smoking withdrawal peak:


3-5 days.

662.pt with inferior leads st segment elevation MI treated with with


thrombolysis 2 days after develop dyspnae and palpitation on examination
there is pan systolic murmur at the apical area vital signs BP90/60 pr normal
Ask about the complication happen ?
-tendinous papilla rupture
-cardiac temponade
-pericarditis

663.pt with acute anterior mi ask about the best proper management ?
-asprin. Heparin . di natrate ,
-asprin . heparin
Asprin . dinatrate . thrombolysis
664-75 year old pt known HTN he developed symptoms and signs of CHF
treated well until the symptoms relieve now he is on lisinopril . frusimide .
Ask about other drug to add with the above plan
His vital normal and BP 150/9What to add
A. -bb
B. -ccb
C. spironolactone
D. -dinatrite

665- long case talk about pt diagnosed as SLE and had acute attack ask about
the management ?
A. azathioprin
B. -steroid and hydroxycloriquine

666-antibiotic that is flare up the SLE ?


A. -amixicillin
B. -sulphamexazole
C. -furantoin
D. -ciprofloxacillin

667- which antibody that confirming SLE?


A. -ANTi DSdna
B. -ANA
C. -antihistone
D. -RF
Antihistone for drug induced Lupus

668-pt known case of asthma come for the clinic he is on inhaled B agonist
and ICS still have attack weekly what the next proper treatment ?
A. -ipratropium promide
B. -Inhaled LABA
C. -leukotriene receptor antagonist
D. -cromolyn

669-what is PFT parameter that is for assessment the severity of COPD ?


A. -FVC
B. -FEV1
C. -SPO2
D. -PEFR

670-pt known COPD 65 year he is was on ICS and inhaled LABA brought by his
son disoriented c/o productive cough and fever for 3 days
ABG done , HCO3 46mmol, PCO2 50 ,Po2 4 Kba ,PH 7.27
What the best way to manage his mental state ?
A. -nasal CPaP
B. -mechnical ventilation
C. -ipratropium promide
D. -antibiotic

671-pt have pancytopenia ask about the cause ?


A. -parvovirus
B. -adenovirus
C. -hiv

672-case have symptoms , signs and lab of polycythemia ask about the
diagnosis?
Polycythemia
Hyperviscosity symptoms —> headache , thrombosis ,Pruritis

673- 2chest x ray of consilidation but for acute history of cough and flue like
symptoms one talking about a SCA pt have recurrent chest infection and limb
pain ask about the prevention of this episodes and the other having empyema
what the management
Hydroxyurea iv Acute chest syndrome

674-pt of non alcohol steatohepatitis what the best forst management ?


A. -wight loss
B. -statin
C. -take omega 3 diet

675- A 69-year-old woman admitted for a surgical procedure is noted to have a


soft systolic murmur at the left sternal edge. Her ECG and chest X-ray were
normal and transthoracic echocardiography revealed a small posterior
pericardial effusion with normal valves. Which of the following would be the
most appropriate next step in this patient's management?
A. diagnostic Pericardial aspiration
B. mammography
C. purified Protein derivative test for tuberculosis
D. reassurance
E. right heart catheterization

676-A 28-year-old man who had had tuberculosis of the mediastinal lymph
nodes diagnosed two weeks previously and who had been started on
chemotherapy with rifampicin, isoniazid and pyrazinamide was admitted
because of the increasing dyspnoea and stridor. Chest X-ray showed
compression of both main bronchi by carinal lymph node enlargement.
What is the next step in management?
1- Start prednisolone
2. Mediastinoscopy and biopsy
3. Refer for stent insertion/tracheostomy
4. Refer for urgent CT scan of the mediastinum
5- The addition of ethambutol

677- A 28-year-old man had been treated for pulmonary tuberculosis with
rifampicin, isoniazid, pyrazinamide and ethambutol for four weeks. Pre-
treatment liver function tests were normal but his most recent investigations
revealed:
serum total bilirubin 98 micromol/l (0-18)
serum alanine aminotransferase 620u/l (5-45)
serum aspartate aminotransferase 450 u/l (5-45)
serum alkaline phosphatase 720 u/l (40-110)
Which one of the following is the most appropriate next step?
1- Stop all treatment
2. Stop ethambutol
3. Stop isoniazid
4. Stop pyrazinamide 5. Stop rifampicin

678-A 48-year-old man presents to the physician with episodic productive


cough of yellow sputum for the last two years. He denied any fever or chest
pain. Examination shows digital clubbing.
Which of the following is the most likely diagnosis?

A. Asthma
B. Bronchiectasis
C. Sarcoidosis
D. Tuberculosis

679-A70 year old male has long-standing bronchiectasis. What else beside
medical treatment can benet this patient?
A. Chest physiotherapy
B. High-dose oxygen therapy
C. Rigorous exercise
D. Smoking cessation

680-Which of the following conditions is associated with clubbing?


A. Asthma
B. Bronchiectasis
C. Pneumonia
D. Pulmonary tuberculosis

681-Which of the following decreases the inammation in asthma?


A. Beclomethasone diproprionate
B. Montelukast
C. Salmeterol
D. Theophylline

682-40 year old female presents with an acute asthma exacerbation in the ER.
Nebulised SABA is provided. What should be given next?
A. IV SABA
B. Nebulised ipratropium
C. Oral corticosteroids
D. Theophylline

683-A healthy 40 year old man presents for a tuberculosis screen. He is


asymptomatic
with a normal Chest XR, and his last PPD 5 years ago was negative. Today, his
PPD is positive. What is the treatment?
A. Isoniazid for 6 months
B. Isoniazid,rifampin,ethambutolandpyrazinamide
C. Reassurance
D. Rifampicin and isoniazid for 3months

684Which of the following is the most common cause of chronic cough?


A. Asthma
B. Gastroesophageal Reux Disease
C. Pneumonia
D. Upper airway cough syndrome

685-A 16-year-old male presents with acute severe asthma. On examination his
peripheral pulse volume fell during inspiration.
Which one of the following is the most likely explanation for this clinical sign?
1- The cardiac effect of high dose beta agonist bronchodilator drugs
2. A falling heart rate on inspiration
3. Myocardial depression due to hypoxia
4- Peripheral vasodilatation
5- Reduced left atrial filling pressure on inspiration

686-Which ONE of the following is a contraindication to thrombolysis?


1- age over 75 years
2- the presence of atrial fibrillation
3- asthma
4-pregnancy
5-background diabetic retinopathy

687-Regarding the genetics of bronchial asthma :


1- Mendelian recessive inheritance
2- Leukotriene concentrations are influenced by genetic factors
3- Similar concordance in monozygotic and dizygotic twins
4- Genetic linkage is to a single chromosome
5- There is a contribution from HLA alleles
688-A 20 year old male presents for a tuberculosis screening test. He has no
risk factors for exposure and is otherwise well. What is the minimum wheal
required for a positive test?
A. 10mm
B. 15mm
C. 20mm
D. 5mm

689-. A 40 year old patient presents for a health screen. He is asymptomatic.


Chest X ray shows a unilateral calcied nodule on the upper zone of his lung.
What is the likely diagnosis?
A. Adenoma
B. Granuloma
C. Hamartoma
D. SCC

690-39. A 45 year old female and non-smokeris found to have a lung nodule on
CT. It appears to be composed of calcium and fat.
A. Adenocarcinoma
B. Hamartoma
C. Mystheoma
D. Squamous cell carcinoma

691-Amebic liver abcess management:


Metronidazole

692-Tb infection control:


1. Administrative measures
2. Environmental controls
3. Use of respiratory protective equipment
4. All of the above

693- Pt with clear symptom of cushing syndrome and give diagnosis on


sinario Investigation showACTH low Cortisol at 8 am normal level ( after given
oral dexamethasone 1 mg at night ) wt investigation to do ?

A. Brain MRI
B. CT for adrenal gland

694-pt diagnosed acromegaly and started ttt with octreotide what the
investigation of choice in the future ?
Echo
Ct abdomen
Echo + colonoscopy
Cuz acromegaly associated with CVD and colon cancer
695- MVA pt was fully conscious but 4 hrs later pt become unconscious and
left pupil was dilated and there was fracture at temporal bone

A. Epiduralhge
B. Subdural
C. Subarachnoid

696- .known case of chronic hepatitis b virus since two weeks he complained
of increasing fatigablity and become jaundice,all liver enzyme abnormal with
inr 1,1
A.liver biopsy
B. Ct
C. Ercp

697.pt know osteoporosis with hx of neurological problem corrected surgically


come with 2 weeks urge and urine incontinence with numbness and weekness
in both lower limb
a-Coda equani syndrome
b-Transverse mylitis

698.pt with maltoma and h. Pylori posative


1. Antibiotics
2. Chemotherapy 3. Surgery
4. Radiation

699.pt with high grade fever and abdominal pain for 5days then become pale
and toxic
1-Single blood culture
2-Multiple blood cultures
3-Bm aspirations
4-Urine and stool cultures

700.pt febrile and palpable lymph node with white caot in tonsil, most common
complicatio
1. Pharyngitis
2. Pneumonia
3. Scarlet fever
4. GN

701.pt wih skin rash and recurrent infection, low plt , family hx of same dz in
paternal uncle
. Wichot Aldrich syndrome
702.pic of hypersegmented neutrophils and symptoms of anemia
B12 deficiency

703.pic of hypochromic microcytic anemia what else will be low besides low h

1. Mcv
2. Plt
3. Retic

704.celiac avoid what


1. Barley
2. Battato
3. Rice

705- Pt was self dependent in eating, drinking and wearing clothes. Then
he developed dementia and some abnormal behaviors and agitations.
How to manage?
A.Haloperidol

Frontotemporaldementia

706- Q about erythema nodusum and pyoderma gangrenosum of CD and


UC, with their pictures.

707- the daughter of an old pt complain of progressively decreasing in memory


and change in personality pf her father, ttt?

A.refer to the geriatric clinic

708- Pt has thigh swelling, diagnosed as sarcoma. How to know metastasis?


A- MRI
B- CT
C- CT angiogram
D- Bone scan
709-Healthy 35 years old male doing routine investigations then accidentally
solitary nodule found on CXR. What to do?
A-Follow up after 6 months!!!
B-CT
C- Takebiopsy
D- Resection

710-The most important precaution in patient with Pulmonary TB is?


A-Airborne infection
B-Droplets infection

711- Elderly male came to ER with SOB and difficulty sleeping at night. He
mentioned Hx of heart surgery few years ago. You did investigations and
ECHO. What is the most important thing you will be afraid of that might kill
patient?
A-Pulmonary HTN
B- Stroke
C-PE
D- Cardiogenic manifestations

712-Elderly male came with weight loss, headache, RUQ tenderness, LFT all
high. Dx?
HCC

713- Pt with Normal sodium, low potassium, and has HTN. Dx?
A-Pheochromocytoma
B- Hyperaldosteronism

714-Pt known case of DM and HTN. He has uncontrolled HTN on multiple


medications. What is the cause of uncontrollable HTN?
A-Taking food salts
B- Hyponatremia

715-A Case scenario of K/C DM pt came complained of fatigue, lab shows low
bicarbonate, normal Na and low k. What to give?
A-Givebucarbinfusion
B- Give IV insulin with dextrose
C- Givenormalfluid
716-A case scenario of patient K/C of non hodgkin lymphoma came with
feathers of tumor lysis syndrome. What electrolytes abnormalities you
will find?
A-HypoCa!!!
B- HyperCa!!!

717- pt with all the viral hepatitis markers negative except positive HB
Surface antibodies. Dx?
A-Previous immunization
B- Acute resolved infection

718-What is the most common organism to cause Post prosthetic valve


(following valve replacement) endocarditis?
A-Staphepidermedis!!!
B-Staphaureus!!!
!!!C-Streptviridans

719-Case scenario of patient complaining of SOB, with Severe MR + mild LVH


TTT?
A-Mitral valve replacement!!!
B- Give medication!!!

720-Case scenario of patient with syncope and SOB. Dx?


Aorticstenosis!!!

721-but not responding, best drug?


Methotrexate!!!

722-Case of abdominal pain, bloody diarrhea, Hb low. Dx?


HUS
723-Low Hb, MCV low, RDW high. Dx?
A-IDA!!!
B-Aplasticanemia!!!
C-Thalassemia!!!

If Hx of methotrexate > folate


If pt on phenytoin > folate

724-Pt with recurrent chest infections and low platelets. Family history
of similar condition. Dx?
Weskot Aldrich Syndrome “X linked thrombocytopenia”

725-Female patient 25 years with muscle weakness increased throughout the


day. What to do?
A. Ach esterase antibodies
:myasthenia graves , patient present with proximal muscle weakness , diplopia ,
ptosis , we give him Ach esterase antibody like pyridostigmine

726-Case of diarrhea + dehydration + loss of deep tendon reflexes + flat T


wave on ECG + muscle weakness. Cause of these muscle and ECG findings?
.A HypoMg
.B HypoNa
.C HypoCa
.D HypoK

727-Case of loin pain, IVU showed non opaque patches on renal pelvis, US
showed hyperdense echoic patches. Dx?
A. Renal stone
.B Tumor
C. Rena papillaeatrophy

filling defect , non opaque is Uric acid renal stone

728-Case of elderly with HCV Hx. US shows hyper-vascular mass on right lobe
of liver. Dx?
.A HCC
B. Cholangiocarcinoma
.C Hamartoma

729- Case of elderly with weight loss and obstructive jaundice (high total and
direct bilirubin) with mild liver enzymes elevation. Dx?
A. Periampullary carcinoma

730-Pt with PE, given IV Heparin for 5 days, INT 1.2, Symptoms controlled.
What to do?
A. Stop Heparin
B. Change to Enoxaparin

shift to warfarin with target 2-3 INR for 3 to 6 months

731-Inferior MI. What is the post emergency management (post MONA and
ECG and liver enzymes)?
A. TPA thromolysis
B. PCI with stent
.C Heparin

1- initial management: ( aspirin 162-325 mg + oxygen and maintaining O2


>90%,nitroglycerin,morphine )
2- then reperfusion therapy :
3-antiplatelet : addition to aspirin ( clopidogrel 600mg if having PCI & 300 mg if
<75yrs and treat with fibrinolytic ) , alternative if having PCI ( prasugrel,ticagrelor ) .
4- anticoagulant : if having PCI ( IV unfractionated heparin or IV bivalirudin ) , if
treated with fibrinolytic (IV unfractionated heparin , enoxaparin , fondaparinux)
. avoidNSAIDsexceptaspirin -5
6- additional therapy : BB, ACE ( reduce mortality , recommended for AMI ,
HF,LVEF<40%) , ARB (if ACE contraindication , statin

732- 55 years old male complained of sudden severe headache while lifting
something. What to do?
A.MRI brain and cervical spine
B.CT brain and cervical spine

High BP can lead to subarachnoid hemorrhage, heavy lifting and straining can cause
pressure rise in brain lead to aneurysm rupture .

733- Female patient diagnosed with retro peritoneal sarcoma. What is true
regarding sarcoma?
A.Transmitted to nearby lymphatics
B.Metastasis

734-Female complaining of vomiting when she goes in airplane. Most


appropriate antiemetic?
A- Ondesterone
B- Scopolamine

735- 16 years old male was complaining of dysuria, after 3 days the
parents said that his urine became foul smelling. Gram negative bacilli
was found in urine. What is the causative?
A-Klebsiella
B-Proteus marbilaris
736- Croup case. Management?
A-O2!!!
B-Antibiotics
C- Mild :steriod

Sever : inhaled epinephrine

737- Elderly female came with dysuria and frequency. What investigation
you will do?
A-Urine analysis
B-CT abdomen
C-Something invasive

738- Asthmatic patient on short acting bronchodilator and low dose


steroid inhalation. Still complaining of daily symptoms about 5 days per
weak and night symptoms about 2 days per weak. What to add? (No
LABA in choices)
A••Leukotriene!!!
B••Theophylline!!!
C••Nothing to add!!!

So either to add LABA


Or increse dose of ICS

739- Best prophylaxis against peripheral atrial disease?


A••Programmed aerobic exercise
B••Aerobic exercise
C••Something exercise

If (smoking sessation) in the choices , choose it

740- New test was introduced to 800 of patients who was proven th have liver
cirrhosis And other 800 who didn't have liver disease
200 of first group tested positive and 100 from 2nd group tested positive. What
is the sensitivity of this test?
A••25%
B••33%
C••57%
741- 78 years old male admitted to ICU for MI that was complicated by
pneumonia in hospital, he received Tazocin for 12 days, now complains of
right upper quadrent pain with tenderness, US was done with report of thick
gallbladder wall with fluid collection and no stones. What is the management?
A••Emergency cholecyctectomy
B••ERCP drainage
C••US guided drainage
D••Continue same medication

This is a case of acalculuscholycystitis and pt is post MI so, unfit for surgery. Pt for
percutaneous drainage. But if pt is fit the best is cholycystectomy.

742- DM, HTN with chronic heart failure, comes to clinic for medication
counseling, he takes insulin l, furosemide, enalapril, digoxin. Some tests
values where provided. K = 5.9, Glucose high, BP normal. What drug should he
stop?
A•• insulin
B•• furosemide
C•• enalapril(ACEI causes hyperkalemia)
D••digoxin
743-Child come with cola urine, pt has hx of tonsilitis, PSGN is suspected,
what test would help you confirm your diagnosis?
A Blood culture
B anti-streptolysin titer
C Complement 3 low

744-Case about septic arthritis and asking about diagnosis?


fluid aspiration

745-Case about some labs with high calcium and normal PTH (1.1), phosphate
was not given, renal functions were elevated slightly and asking about what is
the diagnosis?
A••Renal failure
B••2ry hyperparathyroidism
C••Primary hyperparathyroidism
D••Tertiary hyperparathyroidism

746-Case about teenager had argument with her mother and swallowed 20 pills
of acetaminophen come to emergency the second day having abd.pain and
nausea, vomiting, asking about stage?
A••Stage 1
B••Stage 2
C••Stage 3
D••Stage 4
747-Case about child with lower limb movement loss after URTI, child
describes his problem started from down and assending upward, there was
somthing about his sensation in lower limb but as I remember it wasn't lost
completly, asking about diagnosis?
A••GBS
B••Transverse mylitis

748-Man recently diagnosed with parkinson and he wants to begin treatment,


what is the best drug to give?
••Levedopa

749- smoker with restructive pattern


- Pulmoarry fibrosis
- COPD

750- COPD not responding to SABA next


ICS

751- case about coarctation of aorta


Aortofemoralpuls delay

752- PET pt I think was 32 wks– mltiskills tear


ICU unclear Q

753-celiac – gluten free diet

754- FTT,,vomiting, aspiration pnemounia


GERD

756- VIT D deficiency


Osteoporosis

757-pain relief after drinking milk elevated Ca anf PTH


- Primary parathyrodism
758-pt with generalised edema –
diuretic

759-pt crescendo decresnod diastolic murmur


AS

760-reflux, bitter tasts


- Gastritis
- Esophaits
- Bohovr syndrome

if early the answer is A


If late stage the answer is B

761- anaphylactic shock mx


Epinephrine IM

762- croup dx
763croup mx
764-case about pertusus cough
Whooping couhg>> pertussis
Barking cough>> croup
765- case about glaucoma

766- Elevated bilirubin and normal liver function test


767- Assess kideny in DM pt annually through
1- urine microalbumin urea
2- creatinine/ albumin ratio

768-To avoid airbag injuries in children below 12


- restrain in back seats

769- Q about basal cell carcinoma


Common in face, Edge roll out, Rodent ulcer

770- pt with delirium mx


1. environmental setting and elevate bed realse

771-Case of delirium what is the dx


Delirium In elderly usually post surgery or infection like UTI
Most common cause in elderly is multiple medications

772 -Talkitve pt redirect him?


A. let him to complete his talk

773-Croup what is the PEx finding inspiration


A. Stridor And barking cough

774-Case of pneumonia what is the dx History:


Fever and cough short period
Ex:
Dull on percussionInitial X-ray
Best sputum culture

775-Child with hypotension first step in mx


A. IV fluid

776-Epiglottis mx
A. Oral ABx
B. IV Abx
C. Admission
777-How to confirm TB
A. Sputum culture

778- TB precaution :
A. Contact
B. Droplet
C. Airborne

779-How to confirm pneumonia


A. Sputum serology

780-Pt once start vancomycin start to get rash What to do


Stop and label
Continue on smaller dose
Replace with cephalospor

781-Amebic liver abcess management: -


782 -Eccinococus liver abcess management:

783-Pyloric stenosis management:

784 -Crecendo acrecendo murmur cause


A. AS

785-Meningitis treatment in elderly:


A. Vancomycin + ceftroxine + ampicillin

786-Which of the following association with Trousseau,s sign?


A. Pancreatic cancer
B. SCC
C. breast cancer
D. gastric cancer
787-Patient with past history of hodgikon lymphoma . But cured completely
.Presented with back pain . Examination and evaluation show paraspinus
Edema and uid collection, -veburecella titer and tuberclintest ,what the cause ?
A. breast cancer
B. burecellosis
C. recurrent hodgikon lymphoma
D. typhiod fever

788- Pt with painless ulcer + skin rash. Dx?


A. Secondary syphilis

789- Pt with painless ulcer + skin rash. Causative?


A. Tryponemapalidum

790- Pt with high SVR but all other parameters (right side, PCWP, BP, CVP).
Dx?
A. Hypovolemic shock

791- Pt with ejection systolic murmur. Dx?


A. Aortic stenosis
Crescendo-decrescendo systolic ejection murmur and soft S2 (ejection click maybe
present). LV >> aortic pressure during systole. Loudest at heart base;

792- Pt with both kidney and lung affection. Dx?


A. Burger disease!!
B. Goodpasture

793- Case of hemolysis + impaired kidney function + thrombocytopenia. Dx?


A. HUS

794- Case of hemolysis + impaired kidney function + thrombocytopenia + fever


and confusion. Dx?
A. TTP
795- Pt with chronic HCV came with UGIB. You did resuscitation then what to
give medication?
A. Octreotide
B. Vasopressin

796- Pt with weight loss and epigastric pain (features suggests gastrin cancer i
think). What investigation you will order?
A. Endoscopy

797- Case about meningitis. Prevention?


A. Rifampicin
B. Ceftriaxone

798- Smoking increase risk of ischemia by?


A. Two times
B. Three times
C. Four times
D. Five times

799- Pt has diarrhea and he is smoker. His brother has crohn disease. He is
concerned about having crohn disease also. What things that might decrease
occurrence of crohn disease in this patient?
A. Smoking
B. Diet

800- Case about Rheumatoid arthritis management.


A. Methotrexate

802- Case about SLE flare management.


A. Steroids
803- Pt can't look to medial above and below. What cranial nerve affected?
A. 3rd
804- Pt can look normally to right side. When asked to look to left side, right
eye looks normally while left eye stops at middle. Dx?
A. Left 6th CN palsy!!!

805- Pt with features of ischemia for long time, started now to have ischemia at
rest and episodes increased. Dx?
A. Unstable angina

806- What is the thing that increased the level of BNP falsely?

807- Bacteria that causes pseudomemberaneous colitis is?


Clostridium dificile

808- What (increase or decrease I don't remember) the activity of Clostridium


dificile?
A. Vitamins
B. Ca carbonate
C. Ferrous sulphate

809- ECG of pericarditis


A. Diffuse ST elevation And PR segment depression
810- Adenosine is not given with what medication?
A. Dipyredamone
B. Theophylline

811- Clinical and ECG case of Inferior MI. Pt hypotensive. Next step?
A. Right chest leads
B. ECHO

812- What parameter will tell you BA severity?


A. FEV1/FVC ratio

813- COPD case, best to tell you about prognosis and oxygen need in
PaO2 measurement once or twice and less than how much

814- Case of eczema, thrombocytopenia and recurrent infections. Dx?


A. Weschot Aldrich syndrome

815- Case of X linked agammaglobulinemia.

816- Picture if Impetigo.


817- Picture of genital plaques chondylomata.
818- Picture of CXR.
819- Picture of ECG.
820- Picture of CTG.

821- Case of resistant eczema (atopic dermatitis, not responded to steroid).


TTT?
A. Calcineurin inhibitors (Tacrolimus)

822- Most common infection associated with IUCD is?


A. Actinomyces
823-Pt known congestive heart failure on Lasix spironolactone and digoxin
also Asthmatic salbutamol invest showed K2.2 what is the cause ?
A-Digoxin
B-Salbutaml
C-Spironolactone

824-Female came to clinice her sister which was known adult polycystic
kidney disease,she asking for screening what u will request ?
A-AbdUS
.B-Antibody for polycystic kidney disease

825-Pt came fever and sore throat on exam hyperemic tonsil wh complication
can
occur
A-Glomerulonephrites
B-Pneumonia
826- One yr old presented with high grade fever,has droling of saliva,he looks
ill,toxic,febrile what is the intervention
A-Urgent refer to ENT for tonsilectomy
B-Admit to ICU and prepare if need intubation
C-Give him Oral abs for 7 days

827- Pt HTN on Lisinopril 20 mg, Amlodipine 5 mg and Warfarin 2 mg, he


developed cough, AF bacilli +ve, anti TB started. Next step?
A- Decrease lisinopril.
B- Increase amlodipine.
C- Increase warfarin.
D- Stop rifampicin.
ACEI caused dry cough
Rifampcin
‫يقﻠل عمل‬warfr

829- Pt developed neck pain radiated to shoulders and numbness what is the
diagnosis?
A- Rotator cuff.
B- Cervical disk prolapse.
C- Polymylagia Rheumatica.

830- Pt developed vesicle in mouth with cervical lymph node. On exam spleen
2cm. Dx?
A- EBV.
B- HSV.
C- HPV.
D- HZV.

831- Women with skin rash and malar rash and severe joint pain. CBC and RFT
normal. What to add?
A- Cyclophosphamide.
B- Mesotrexate.
C- Azathioprine.?????

A for kidney involvement


B for sever joint pain

832- Man with severe abdominal pain, central, he had a history of


cardiomyopathy. Dx?
A- AAA.
B- MVO.

Mesenteric vascular occlusion


Any elderly cardiac patient presented with abdominal pain so it is MVO until proven
otherwise
Hx of cardiac disease as AF , valvular disorders with abdominal pain

833- Pt came with cervical laymph node enlargement. The patient ask to take
biopsy. WOTF if present is an indication for biopsy?
A- If LN > 1 cm.
B-If there is fever.
C- Supraclavicular LN.

834- Women developed palpitation, HR 174, vitally stable. What to give?


A- Amidarone.
B- Adenosine.
C- DCshock.
D- BB.

835- Glomerulonephritis and hemoptysis?


A- good posture syndrome.

836- Pt K/C of ALL on chemotherapy developed fever, investigation showed


pancytopenia. What to do?
A- Take blood, urine culture and oral ABx.
B- Take blood,urine culture and inj ABs.

837- Pt underwent a surgical procedure for lung cancer developed polyuria


and polydipsia, the investigation showed hypernatremia. What next?
A- Increase fluid intake.
B- Desmopressin.

838- Specific test for nystagmus?


A- Red reflex.
B-Cover test.
C-Corneal light reflex.

839- Malignancy associated with Hashimoto thyroiditis?


A- Papillary.
B-Nodular.
C-Medulary.
D- Lymphoma.

Hasimoto's 70 times more risk to get thyroid lymphoma


D then A
840- 4 cases of bronchial asthma.
841- Case of IHD.
842- Case of bacterial endocarditis.
843- SLE case.
844- IBD case.
845- DKA case.

846- Man with HTN developed dysuria and interrupted urine stream. PSA is 1.
Next?
A- BB.
B- Alphablocker. Prazosin
C- Transurethral prostatectomy.

Below 9 life sty


9-19 so Alfa
Above 19 sugary

847-An 8 yr old girl with weakness since birth, complaints of upper right
abdominal pain, hepatomegaly, jaundice, and ascites…diagnosis..pic of both
usg and histopathological slide
givena
a. liver cirrhosis
b. fulminant liver failure
c. budd chiarri syndrome
d. hepatic cholangipathy

848. chlamydial infection…what measures taken along with intervention to


public health education.
a. water and sanitation
b. sanitation and kill reservoirs
c. kill reservoirs and improve sanitation
d. eradicate reservoirs and improve sanitation

849. 35 year old smoker , on examination shown white patch on the tongue,
management:
a)abx
b)no ttt
c)Close observation
d) biopsy and excision

850. A television actress is suffering from rosacea. Since she states that the
appearance will affect her career, what is your choice of treatment?
a. Oral antibiotics
b. Antihistamines
c. Topical antibiotics

d. laser

851. initial treatment for major depression:


ssri

852.A case of schistosomia hematobium infection(liver fluke),what hygiene


measure needs to be taken :
A. Build latrines

853. 40yr.old woman asking about ca cx screening, pap smear when to do and
asking for ur advice...
a. no need now
b. can be done 1 year starting from now.
c. can be done every 5 year from now
d. if 3 negative , no need to further test for screening.

854. mechanical irritation in eye for a worker doing job in an industry..what to


apply..
a. terbinafine drops
b. ciprofloxacin drops
c. olopatadine drops
d. penicilline drops

855. patient with severe headache and vertigo and pain during lifting head,
cannot maintain a steady gait while walking.
a. bppv
b. vestibular neuritis
c. optic neuritis
d. snhl

856. known patient of alzheimer’s family asking for treatment?


New methods of daily activity

857. Health worker exposed to hep b . took all vaccines earlier. what to give
now.
a. hbig + vaccine
b. hbig + lamivudine for 2 weeks
c. only lamivudine for 2 weeks
d. no risk as already immunized
858.scarlet fever treatment
A.ivig
B.erythromycin
C.intubation
D. paracetamol high dose

859- 72 years old man diagnosed with gastroenteritis and has been vomiting
for the past 72
hours. What's released in the body?

A. CRP
B. cytokines
C. apoprotien

860- patient presented with Hypotension, his phosphatase in normal level, after one
day his
phosphate level decrease, what organ damage?
A- Liver

B- kidney

C- lung

If phosphate increase = kidney

861- SSRI that mostly causes overdose:

A. Sertraline
B. Paroxetine
C. Citalopram

862- Pt suspected H pylori

A blood
B stool
C urea breath
863- Which muscle would be completely paralyzed by obturator nerve injury?

A. Gluteus Maximus.

B.Adductor magnus

C. Adductor longus

864- Best treatment of trigeminal neuralgia?

A. Carbamazepine
B. Prednisolone
C. Naloxon

865- Metformin act on the muscle by:

a. Increase glucose release


b. Stimulate fat oxidation
c. Stimulate gluconeogenesis
d. Increase glucose uptake
866- TTT of H.pylori infection:
a. Omeprazol 2 weeks, clarithromycin and amoxicillin 1 week
b. Ranitidine , erythromycin, metronidazole for 2week

867- 40yr old male pt with h/o syncope when he exercises and on rest amd
chest pain.on exsmntn:there s ejection systolic murmur grade 2 -4/6on the left
lower sternal border not radiating and increases when lying dwn..there is left
atrial emlargmnt.

1.Aortic stenosis
2.Pulmonic stenosis
3.HCM
4.constructve cardiomyopathy

Question maybe wrong

868- Man travels to sudan 2 weeks ago , now he is presnted with fever , maleas
.... unspecific symbtom ( from 3 days ). How you will conferm diagnosis??
A) blood culture
B) Serology

A) Blood film ...


Malaria

869- Patient came to ER, complaining of hypotension, tachycardia and


hypercapnia, ECG showed arrhythmia, what is your immediate action?

A. Needle decompression .
B. Pericardiocentesis= if muffled heart sound
C. FAST ultrasound
D. Thoracotomy

870- 22-year-old male drug addict, visits the hospital several times with
multiple
complaints that are exaggerated. When he is not focused he seems
asymptomatic?

A. Somatization

B. Malingering...

872- In brief: Randomized control studies between 2 populations with coronary


artery disease. What's the first question you should ask?
A. Was the age standardized?...
B. Are the medical facilities in both cities equal
C. If the price of cigarettes cheaper in any of the cities?

873- patient presented to the ER with cough hemoptysis night sweats and
malaise. what is most appropriate initial step in the management?

A. isolation in negative pressureroom...


B. start anti TB
C. give OPD appointment after 2 weeks

874- other question was there was a patent age 60 he has hyperthyroidism he
wants to know what long term complications he'll have....?

a. Brain CT
b. Liver CT
c. Bone scan...

875- Old male present with mid clavicle mass ( smoker for 20y and I think he
drinks ) What is your initial management

A. -Core biopsy
B. -Fine needle biopsy..
C. -bronchoscopy..

876- Inhalation of cotton leads to :

A. sarcoidosis
B. berilliosis ..
C. histoplasmosis

Byssinosis. Byssinosis, also called "brown lung disease" or "Monday fever", is an


occupational lung disease caused by exposure to cotton dust in inadequately
ventilated working environments. Byssinosis commonly occurs in workers who are
employed in yarn and fabric manufacture industries.

877- Giardia diagnostic test

A. 3stool parasite
B. 3 stool culture
C. concentration test
D. D-immunoassay...
878- Which drug should be stoped before IVP!?

A. Thiazides ...
B. ACEI
C. CCB..

879- dull aching headache increase with straining and coughing in DM patient

A. Viral meningitis
B. Tb meningitis
C. Brain abscess
D. Sinusit...

880- parentral Rx of falciprum malaria with 10%

A. parasitemia..
B. artemether
C. quinine....
D. primaquine

881- . What makes the blood air barrier of the lung.

A. Pneumocyte 1....
B. Pneumocyte 2
C. Reticular Interstiti

882- case of streptococcal pharyngitis ,treatment?

A. Aspirin
B. Penicillin...

883- what move molecule in and out the cell nucleous?

A. cytosolic ATP
B. nuclear ATP
C. cytosolic GTP
D. nuclear GTP

884- Best prognosis in lung cancer?

A. SCC
B. adenosquamous carcinoma
C. adenocarcinoma in situ...

885.Behcet disease is more common in?

. young man
886- Cause of death in marfan -

rupture aortic root aneurysm

887/ -Depression

seratonin defeincy

888/ Posterior vagan nerve supply:

esophagus

889- Post vagus nerve damage effect on

A. esophagus
B. jujenum
C. bladder
D. descending colon

890/ different type of acne ,pustule .. what type of acne ?

Inflammatory

891/ how to prevent MERSA?

by hand washing*

892/ In a study they are selecting every 10th family in the city, what isthe type
of study

Systematic study

893/ about steven jonson syndrome is most likely occur in whichgroup?

Patient with something take anti epileptic

894/ 21 yo female with chorioretinal degeneration, myopic with -8.0D what type
of myopia?

- pathological myopia

895 / MVA anterior injury of the pancreas and anterior leakage ofpancreatic
fluid will collect:

Omentum bursa = Truma to pancreas Protein last destination> Endoplasmic


reticulum

896/ Pediatric case scenario indicate respiratory distress syndrome , ask about
the deficiency:
A. dipalmitoyl
B. phosphotidylcholine*

897/ Patient trauma to obturator nerve , which muscle will be fully paralyzed :

Adductor longus

‫ اطول ﻣرحﻠﺔ ﻓي اﻟﺳل ﺳﺎﯾﻛل‬/Q interphase

898/ case of characteristic rash (side effect of sulfa drug)?

Blister

899/ case of cough , SOB, only in work place

hypersensitivity pneumatis

‫ واحد ﻋﻧده‬COPD ‫ وقﻠوﻛوﻣﺎ ﯾش حتﻌطﯾﻠو دوا؟‬Oral Acetazolamide .C

900/ Clinical dementia vit b12 def

901/ Meckls diverticulitis most common in ?

Terminal ileum

902/ Which anesthetic agent is 100 times stronger t han Morphine?

Fentanyl

903/ An elderly, known case of Alzheimer's disease, developed

hallucinations, bizarre behaviors and became aggressive, what drug

to add? : resperidone

904/ A pt drank an ethylene glycol containing product, what renal damage is


suspected?

ATN

905/ Dm , osteoarthritis on NSAID came for regular check up .. Bp found to be


160/some- thing, Invest:Inccr +_ urea IncbicrbDec k nlna What is the cause of
htn NSAID induced

906/ man with erectile dysfunction with normal morning erections. Where to
refer.
Pshyaiatry

907/ A pt with depression on medications found comatose with empty pill


bottle beside her On invx; ABG showed Metabolic acidosis, anion gab =18 ,
what drug overdosed?

Aspirin

908/ pt with fever, altered LOC for 5 days, his condition deteriorated and
developed body rash, and became jaundiced, lab tests showed low Hb, low
PLT?

TTP

909/ Thereis astudy to the effecton intensive insulin regimen in the reduction
ofneuropathy in the patient,the results are as following: Event in the regular
insulin regimen:0.092 Event in the intensive insulin regimen:0.022 Which of the
following is correct

Odd ratiois 4.2

910/ Patient was presented by back pain relieved by ambulation , what is the
best initial treatment :

physical therapy

911/ Which of the following made in nucleolus ??

rRNA in nucleolus

912/ Case of RLQ pain and mass, what is the treatment:

Antibiotics.

913/ Old pt, diabetic dehydrated, lab high Na low k, high bicarbonate, ketones
in urine, Dx?

A. metabolic syndrome
B. diabetic ketoacidosis
C. lactic acidosis

914/.Pain anterior to the heel Worse in the morning and better along with the
day? Diagnosis ?

Plantar fasciitis

915- loss sensation in thumb + index + ring finger What nerve injured

median
916- / Antiviral taking by inhalation?

A. Oseltamivir (Tamiflu) 75 mg PO q12h for 5 days or


B. Zanamivir

917/ Elderly with vasomotor rhinitis :

Decongested

918- Inferior MI which one of the following arteries blocked?

Right coronary artery


Section 2 ..
Pediatric
1.9 Years old boy brought to clinic with his parents complaining that he is
short. He appears healthy, less than 50% percentile for height on growth chart.
His profile revealed progressive increase in height, his labs was normal, best
indicator of his future height is?
A. His weight vs height
B.Parental height
C.parental weight
D. Can't be expected

N.B: The midparental height is a child's expected adult height based on the heights
of the parents.
N.B: In girls; {(the father's height in cm - 13 cm) + (mother's height in cm)} / 2. N.B: In
boys; {(the father's heigh in cm) + (mother's height in cm + 13 cm)} / 2.
For example; if the father's height is 172.72 cm, mother's height is 157.48 cm, so the
midparental height for son and daughter is:
Son: 172.72 cm + (157.48 + 13 cm) / 2 = 172.72 + 170.48 = 343.2 / 2 = 171.6 cm.
Daughter: (172.72 – 13 cm) + 157.48 / 2 = 159.72 + 157.48 / 2 = 158.6 cm

2. child admitted in the ER, due to DKA severely dehydrated, managed with
fluid and insulin , bad management complication can occur ?
A.hyperkalemia
B.hypoglycemia
C. Brain edema.
D.hyperglycemia

3. Baby (2 months old ) was diagnosed with congenital heart disease came to
the ER with bluish discoloration , the mother said he was crying what you will
do?
A.immediate heart surgery
B.Reexamine after baby calmed
C.go home
D.refer to another doctor

N.B: This is a case of Tetralogy of Fallot.

4. 1 month year old baby has yellowish discoloration( body and eye ) he was
diagnosed with physiological jaundice. Total bilirubin 25, mainly indirect. What
is the dx?
A. Rh incomputable
B.ABO incompatible
C.c.najjar

D.Gilber syndrome. N.B: Indirect high.


N.B: ABO incompatibility in first day, Criggler najjar are 2 types (type 1 died usually,
type 2 manifested in adults).
N.B: Causes of high indirect unconjugated bilirubin (Gilbert, criggler najjar, ABO
incomp, G6PD,
breastfeeding jaundice, breast milk jaundice).
N.B: Causes of high direct conjugated bilirubin (biliary atresia, choledochal cyst,
galactosemia, neonatal hepatitis, TORCH).

5. 15 yrs female not menstruating yet at clinic by her parents ,she is short
stature , short neck , hypertensive. Most appropriate diagnosis?
A.Turner
B.down
C.TFS
D.normal variant

N.B: HTN + short stature + webbed neck + primary amenorrhea = Turner.


N.B: Conditions associated with turner syndrome; hypothyroidism + Coarctation of
aorta.

6. 3 years old baby with single palmar crease - long tongue - straight hair.
What is the dx ?
A.turner
B.Down
C.marfan
D. patue

N.B: Most common cardiac anomaly with Down is AVSD (Atrioventricular canal
malformation)..

7. Child with high TSH ,low T4 what is the next step ?


A.Start thyroxine
B.AbdUs
C.brain MRI
D.barin CT

N.B: High TSH + low T3T4 = Primary hpothyroidism.


N.B: High TSH + high T3T4 = Secondary hyperthyroidism = next step MRI pituitary
N.B: Low TSH + high T3T4 = Primary hyperthyroidism.
N.B: Low TSH + low T3T4 = Primary hypothyroidism = next step MRI pituitary. N.B:
High TSH + normal T3 and T4 = Subclinical hypothyroidism.
N.B: Low TSH + normal T3 and T4 = Subclinical hyperthyroidism.

8. Baby elevate his head when pronated, turn his head left and right coos his
age ?
A.3 months
B.4 months

9. Child less than 1 year he has vomiting and diarrhea, he is oriented aware ,
dry tongue , sunken eye , normal pulse . what is the dehydration degree ?
A.less than 1 %
B.5-9% (moderate dehydration_.
C.more than 10 %

N.B: Dehydration grades are mild, moderate, severe. N.B: Mild 3-5% volume loss =
Asymptomatic.
N.B: Moderate 6-9% volume loss = Eager to drink, dry eyes and mouth, decreased
skin turger, tachycardia, irritability, orthostatic hypotension, decreased tearing,
sunken eyes, depressed fontanelles.
N.B: Severe 10% or more volume loss = Confusion, hypotension, deep respiration,
cold mottled skin, lethargy, inability to drink.

10.Child has headache and can't concentrate on Study , Lap Hb= 7.9 MCV less
than normal , TIBC high , how to manage ?
A.oral iron
B.oral folate
C.orlavit B 12
D.packed RBCs transfusion

N.B: Low Hb + Low MCV + high TIBC = IDA..

11.child with polyuria and polydipsia OGTT normal , Na + Cl = high.Other


electrolytes are normal, what is your dx ?
A.DM
B.DI
C.siadh
D.conns

N.B: SIADH = Hyponatremia + Low urine output + high ADH + overhydration +


polydipsia
+ low serum osmolarity but high urine osmolarity.
N.B: DI = Hypernatremia + High urine output + low ADH + dehydration + polydipsia +
high serum osmolarity but low urine osmolarity.
N.B: DI has 2 types (Central that is related to head trauma or CNS lesion, treated
with
Desmopresin), (Nephrogenic that is not responded to Desmopresine).
N.B: Conn's syndrome and Cushing syndrome = High Na and Low K + metabolic
alkalosis +HTN.
N.B: Adisson's disease ''hypoadrenalism'' = Low Na and high K + low BP specially
orthostatic hypotension + skin hyperpigmentation.
N.B: Pheochromocytoma = Episodes of flushing, headache, sweating and high BP
usually above 180 mmHg. It is diagnosis made by measuring urinary
catecholamines, best radiological imaging is MRI, treated by alpha blocker
(Phenoxybenzamine) followed by BB followed by surgery.

12. 9 year old child , her mother notice that he is having a mass on his
abdomen , she goes to the doctor and did US which shows a mass, what is the
dx?
A.wilms

N.B: Wilms tumor doesn't cross midline, but Neuroblastoma does.

13. Case Scenario for child present with Cyanosis for one hour with crying ,
Physical Examination reveals murmur in left sternal border , X-ray shows
increased pulmonary vascular marking and small hearts, what is the next step
in treatment?
A- give Abs
B- Cardiac Catheterization
C.calm the baby first (TOF case)

14. 7days newborn girl born with ambiguous genitalia with hypotension on
investigation: low Na, how to treat ?
A- Saline + glucose
B-Hydrocortisone with saline

This is CAH.

15.Case Scenario for child with nephrotic syndrome treated by steroid, what is
the first thing to improve?
A. Edema
B. protein in urine
C. urine strip for pt

16. Case Scenario for Child with DM type 1 present in E.R with symptoms and
signs of DKA, what is the next step ?
A- provide management plan
B- wait family and discuss
C- concern child for DM type 1
D- discuss with child only

17. Child with peri oral and central cyanosis , Dx ?


A- TOF

18. child frequent eye blinking, normal all


A- Tic disorder
B- eye dryness

19.6 months baby have oral candida


A- oral anti fungal
B- systemic anti fungal
C-oral and systemic anti fungal

Oral cuz there is no cutaneous involvements

20-3 yo child :
A)draws triangle
B) use stairs

21. child have abdominal pain, testes tender, horizontal testis above the other
one
A-US
B- scrotal exploration
C- angiography

22.child with low immunity, which vaccine we shouldn’t give ?


Varicella

2 ‫ﻠﯿﺎﺧﺬﺳﺘﯿﺮويﺪمﻨﻔﺘﺮةطﻮيﻠﺔوﺻﺎرعﻨﺪه‬3.
‫ﺎيﺰيﻨﻨﻌﻄﯿﮭﺘﻄﻌﯿﻤﺔﻧﻌﻤﻼيﮫ؟‬
A- no problem to give him vaccine B- give immunoglobulins
C- give him vaccine after 1 month

24. Umbilical hernia in a 10 months old baby. Management?


A- observation

N.B: Most of UH don't need any ttt. Usually the hole heals on its own by time your
child is 4 or 5 years
old. If not so do the surgery before the child enter the school.
25. 2 days old baby with seizure. LP done and reported normal. Dx?
A-neonatalsepsis
B-asphyxia

N.B: Hypoxic ischemic encephalopathy ''birth asphyxia'' is the most common cause
of neonatal seizures, with LP r/o meningitis so most likely it is birth asphyxia.

26. Child with pneumonia (in Right middle lobe) with x ray, vital signs stable.
Management?
A-admission for iv abx
B- prescribe amoxicillin for 7 days and discharge

27. Newborn with hypoglycemia. What to give?


A-2ml/kg of 10% dextrose.
2ml/kg of 12.5% dextrose
C-4ml/kg of 10% dextrose
D-4 ml/kg 12.5% dextrose

28. Newborn of diabetic mother. From where take sample?


A. central venosuline.
B- peripheral
C- nasogastric
D- orogastric

29.Child with jaundice and anemia ,blood film , coombs direct and indirect are
positive, reticulocytes high. Dx?
A-autoimmune hemolytic anemia.
B-spherocytosis
C-laprascopic
D-observe

N.B: Best for hernia is open surgery except: female for cosmetic reason + bilateral +
obese so here it is better for lap surgery.

30. Child with viral diarrhea ?


A-supportive rx

31. Child with abd pain postprandial, distention , diarrhea , ?


A-carb intolerance
B-celiac
C-IBD
D-CYSTIC FIB

32. Child with abdpain , distention , diarrhea , failure to thrive , buttock muscle
wasting ?
A-celeic
B-Ibd
33. Asthmatic child mother said cannot be sure of proper technique, what the
cause of uncontrolled asthma?Bneed step up ?
A. improper technique
B. cortisollevel
C. syntchan test

34. milestone of 5 months old child


A. sits without support
B. coos
C. reaches for object
D. pincergrasp

35. baby with absent red eye reflex otherwise healthy


A. reassurance of pt that its normal’
B. consult ophthalmologist

-Uni :retinobalstoma Bi: congenital

36- baby after feeding spills the milk examination and labs normal
A reassurance of mother and position during feeding

37- playtime for the kids


A 30 min
B 60 min
C 120 mins

38- a 10 kg child with severe dehydration calculate maintenance & deficit


A 46L
B 54
C 76
D 90

-If the question only asked about Maintenance >> the answer is 1000 ml/day or 41.6
ml/h
-If the question only asked about Deficit >> the answer *could be the same as above
*coz severe dehydration means weight loss more than 10% >> the equation is
percentage of body weight loss multiply by the total weight multiply by 10 >>
10*10*10= 1000 ml/day or 41.6 ml/h
-If the question asked about maintenance and Deficit >>the answer is 1000 + 1000 =
2000 ml/day or 83.3 ml/h . Or more

39- a child with about 3 yrs hx of steatorrhea abdominal bloating and diarrhea
amylase was normal what to do?
A stool for fat
B ultrasound
C anti endomysialantibodies
40- a baby with abdominal distension and palpable bladder
A prune belly syndrome

41- baby having unformed stools


A ORS
B lactose free diet
C formula feeding

42- a pt with short neck short stature both parents short


A turner
B familial

43- Newborn 7 days with ambitigous organ with hypotension, glucose low,
sodium low. Management?
A.Saline with glucose
B.Hydrocortisone with salin
C.Saline, glucose, hydrocortisone

44- Child with nephrotic syndrome treated by steroid, what is the first indicator
for improvement?
A.Edema
B.Urine strip for protein
C. Protein in urine

45- Child in ER with DKA?


A.Provide management plane
B.Wait family to discussion
C.Concen the child with DM 1
D.Discussion with child

46- 15years old female c/o amonerrheahtn short neck the parents below the
level what the diagnosis?
A.tuner syndrome
B.hypothyroidism
C.familial

47- child with mingitis , eyes showed papilledema Important complication to


inform the parent?
A. Visual loss
B. Hearing loss
C. Renal failure
D. Retinal hemorrhage

48- 3 months infant with vomiting and recurrent chest infection on ex low
weight and failure to thrive other unremarkable. What Dx?
A. Upper esophageal sphincter hypertrophy
B. Lower esophageal sphincter hypertrophy
C. Pyloric hypertrophy
D.Trachea esophageal fistula
It might be TEF ?

49-Newborn his older brother died from immunodeficiency infections what you
do in vaccinations?
A. Give vaccinations as normal
B. Order vaccinations and ask immunology consultation
Don’t give him BCG or Do test and wait for result

50- 1 year boy came for vaccinations his older brother died of
immunodeficiency disease what you will do ?
A. Defer vaccine
B. Change IPV to OPV

51- Adolescent fight with her mother yesterday and take two packs of
acetaminophen each contain 20 tablets, came today with right lower
abdominal pain , in which phase of toxicity she is ?
A. Phase 1
B. Phase 2
C. Phase 3
D. Phase 4

52- Egg allergy what’s the vaccine CI ;


yellow fever

53- Baby came with absent Moro reflex he is born SVD to DM mother with
weight 4.8 what’s the nerve affected?
Brachial plexus C5-C6

54- 15 years old male come to ER, LP should be done to r/o meningitis. What
to do?
A. Consent from parents
B. No consent it is an ER
C. Consent from pt
D.asset the child and take consent from parents

55-Pediatric patient with rash I think it's varicella his brother is


immunocompromised, How to protect the brother?
you should give the immunocompromised child Varicella IG
Note: with other similar q asking about pregnant womennot immunized the answer
will be Avoid exposure.

56- Infant at 6 weeks his/her mother is complaining of him spitting all the milk
his birth weight 3.5now its 5.5
A.Reassure

57-3 mother bring her child female with vaginal bleeding and red congested
edematous painful vulva , during the last week she passed stool without
control but she already toilet trained , what is the cause ?
a- child sexual abuse
b- other irrelevant choices i forgot them

58-Pt with pyloric stenosis have :


A-hypochi met acidosis
B-hyperchromic Met acidosis
C-hypo Met alkalosis
D-hyperch met alkalosis

Persistant vomiting cause :


Hypokalemic met alkalosis
AndHypochloremic met alkalosis

59-child with nephritic syndrome develop vomiting ,diarrhea and generalizes


abd pain with rebound tenderness is
A-peritonitis
B-appendicitis
C-pancreatitis

60- main stay of treatment of Kawasaki disease:


A-aspirin
B-immunoglobulin
C-Abx

61- 6 year girl with vulvuar itching and bleeding what you think :
A-foreign body
B-sexual abuse

62- adolescent with band like throbbing headache and he have stress at
school :
A-tension headache
B-migraine headache

63- child with coca cola urine color since 1 week and when examine him he
has congested throat with ant LN what he have :
A-acute glomeuronephritis
B-igA nephropathy

64- child with cogested throat and white patch his temp 39 c what complication
most likely he develpe?
A-pharyngitis
B-glomerunephritis

65- newborn with acute respiratory distress and he have shifted trachea what
is the management ?
A-intubation
B-bronchoscpe
C-thoractomy
D-thoracstomy

66- component of tetralogy of fallot:


A-aortic stenosis
B-pulomnary stenosis
C-left ventricular hyperatrophy
D-inter septal defect

67-parent come with child not vaccinated because they think it is bad for him
what you will do?
A-consult child support
B-explains to parents about wrong myth and it is good for child

68- 4years with asymptotic hernia what you will do?


A-observation
B- surgical repair
Surgery is done before entering school age

69- 3 years child fall down and vomitt 2 times and have headach but no loss of
consciousness his neurological exam is normal what you will do?
A-ct scan
B-close follow up
C-consult neurosurgery

70- mother came to ER with her child drinks paracetamol syrup she
remembered it was a small amount in bottleand his clothes have some syroup
vitally stable and he has normal appearance he spent 4 hrs in ER what it the
next?
A-observe for another 4 hrs
B-discharge with instruction

71- baby vomit large amount after feed esophaguses PH is low but his growth
chart is normal what the next?
A-observation
B-do esophageal monometry
C- abd us

72- child with cystic fibrosis and his family is normal what you will do?
A-sibling sweat chloride test
B-father sweat chloride test

73- newborn crying and refused feeding and the smell of his urine like burned
sugar what is dx?
A-pheoketonyrea
Answer : Maple syrup disease

74- child with meningitis what is the ttt?


A-vancomycine and ceftrixone and steroids

75- Father bring his child because hisbrotherdieddueto immunodeficiency or


like that what vaccine should be not given for this child?
A-Influenza (nasal)
B-Injectablepolio
C-pneumococcal
76- absolute contraindications of Dtap ?
A-seizure. B-child with severe crying for 3 hrs
C-encephalopathy after 7 day of previous vaccination

77- 2 month baby born at 35 week his wt is 1.7 kg about his vaccinations?
A-delay for 2 month
B-give usual vaccines with usual dose
C- give usual vaccines with half dose

78- absolute contraindications of Dtap ?


A-seizure
B-child with severe crying for 3 hrs
C-encephalopathy after 7 day of previous vaccination

79- 2 month baby born at 35 week his wt is 1.7 kg about his vaccinations?
A-delay for 2 month
B-give usual vaccines with usual dose
C- give usual vaccines with half dose

80- 9 yr old with acute severe abdominal pain what is the most common cause
at this age
A- Appendicitis
E- Cholecystic

81- child with meningitis


A-Vancomycin+ Ceftriaxone
82- 8 yrs old ingested high dose of paracetamol presented with N,V, URQ pain
Which stage of toxicity
NO choices
- Stage 2

83- pt presented with hematuria and was sick and hypertensive and his
parents mentioned that he has petechia before
A-HSP
B-Post streptococcus glommuronephritis

84- Newborn delivered with C-S because of deceleration. He is in respiratory


distress, and CXR showed fluid in horizontal fissure. What is the diagnosis?
A- RDS.
B- Meconium aspiration.

CXR findings in neonatal lung diseases; in RDS there will be ground glass
appearance (homogeneous infiltration) + air bronchogram and decreased lung
volumes , in Meconium aspiration syndrome there will be coarse irregular patchy
infiltrates + flattening of the diaphragm + lung hyper expansion (hyperinflation) and
10-20 % may have pneumothorax)
^ Toronto

85- 12 Old months baby with gastroenteritis on oral rehydration after recovery
and recommence on normal diet start to have diarrhea. Active with mild
dehydration. TTT?
A- Milk cow free formula.
B- B- After recover use oral rehydration for 24 hrs.
C- C- For 48 hrs.

86- Severe dehydration in pedia fluid deficit replacement over?


A- 48 hrs.
B- 24 hrs.
C- 12hrs.
D- 6 hrs.

87- 9 Years old boy short, both parents below average. Dx?
A- Constitutional.
B- Familial.

88- Another q of 9 years old boy short, bone age is 6 years, both parents below
average. Dx?
A. Constitutional.
B. Familiar.
89- Delayed puberty 15 yrsshort , some problem with hair line, elevated blood
pressure, short and I think constipation. Dx?
A. Turner syndrome
B. Hypothyroidism.

90- Vegetarian parent , pale child mx?


A- Course of oral ferrous sulfate.
B- B12
C-
Explanation :
Vitamins that will be deficient in vegetarian are (B12 , B6 , iron , zinc )

91- child in DKA mum stop insulin she does not belive that he has DM which
stage she had?
Deniale stage

92- scrotal swelling non trnsillumination What to do


Heriorrhaphy
Herniotomy

93- adlucence with groin pain aggravating by excersis, examination everything


was normal
- Re evaluate in 3 mths?????????/
- Us ????????

94-case of EBV

95- pt with Kawasaki will start him on IVIG what is the indication of poor
response to the medication?
- CRP , direct bilirubin , ALT , AST, albumin and platelet
96- child with meningitis , eyes showed papilledema
Important complication to inform the parent:
A- Visual loss
B- Hearing loss
C- Renal failure
D-Retinal hemorrhage

97- mx of gastroenteritis in peds got it from child care


- Supporative

98- 3 yrs old fell on his head two times vomiting and c/o headache
A- Close observation
B- Head CT

..

99- treatment of febrile seizure


A- Paracetamol

- Controls the fever is most important mx


100- 5 yrs seizure for 5 min IV access present
A- IV lorazepam
B- rectal
C- IM
D- Some options about diazepam and others bout lorazepam

101- child coo smile turn face follow obj when in pron position raise head
which age ?
- 4 months

102- pincer grip , pull to stand, cru, say dada ?


- 10 months

103- 5 months old how to assess he is normal?


- reach objects

104- 8 yrswt for ht 95% , high BMI how to manage ?


- Lifestyle modifications

105-3 mths with noisy breathing win supine, goes away when prone ??
A- Will get wors
B- Will grow out of it when reach one
C- Need surgical repair

106- friendly with strangers, can’t sit without support, play peekaboo, what’s
the developmental age of this child?
1- 6-7 months
2- 7-8 months
3- 8-9 months
4- 9-10 months

107- child with nephritic syndrome develop vomiting


diarrhea and generlizes abd pain with rebound tendernessis
a. peritonitis
b. appendicitis
c. pancreatitis

108- main stay of treatment of kawasaki disease


a. aspirin
b. immunoglobulin
c. Abx
109- 1-2wks Neonate has non bilious vomiting on US there is olive what you
find?
A-Hypercholaremic metabolic acidosis
B-Hyopchl Met alkalosis
C-Hypochl resp alkalosis

110- Infantcrawel ,transfere object from hand to hand,had palmer grasp no


pincelgrisp,age by month
A-5
B-7
C-9

111- Child nephrotic synd developed abd pain whyouexpect


A-Uti
B-Peritonitis

112- 8-11yrs old had night bed e out to go to toilet exam normal he feel
shamed and asked help
A-Avoid punishment
B-Alarm bed Reinforcement
C-Desmopressin intranasal

113-What skin manifestation is associated with cystic fibrosis?


A. Acrodermatitis enteropathica

114-Pt with pyloric stenosis have


a. hypochimetacidosis
b. hyperchromicMetacidosis
c. hypoMetalkalosis
d. hyperchmetalkalosis

115-Infantcrawel ,transfere object from hand to hand,had palmer grasp no


pincelgrisp,age by month
A-5
B-7
C-9

116-Child nephrotic synd developed abd pain whyouexpect


A-Uti
B-Peritonitis

117-11yrs old had night bed e out to go to toilet exam normal he feel shamed
and asked help
A-Avoid punishment
B-Alarm bed Reinforcement
C-Desmopressin intranasal

118- All deciduous teeth fully erupt at age??


a. 2-3 yrs
b. 3-4 yrs
c. 4-5 yrs
d. 5-6 yrs

119- 8 yr old girl with pain abdomen. she is weak since birth, h/o
hepatomegaly, with jaundice… AST 1024 ALT 600 bilirubin 370
a. biliary stricture
b. liver cirrhosis
c. hepatic cholangiopathy
d. congenital liver failure

120- iv fluid management a 22kg child, hourly fluid to be given…


a. 25ml/hr
b. 65ml/hr
c. 110ml/hr
d. 600ml/hr

121- Long scenario Child with Duchenne muscular Dystrophy only given
symptoms of diseases, his mother pregnant and probably fetus is a boy. How
is chance this baby to born with this diseases?
a) 12,5%
b) 25%
c) 50%
d) no any chance
122- bmi chart was given of a 14 yr old boy…asking for conclusion…bmi 32.5
a. normal wt
b. over weight
c. obese
d. morbid obese

123-coarctation of aorta with ?


A) down syndrome
B) Turner syndrome
C) Edouard syndrome
D) Patau syndrome

124-Witch of the following cause newborn meningitis or sepsis ?


A. GBS
B. E-Cali

125-Newborn baby has SOB + no heart sound + usual site ... ?


Diaphragmatic

126-Child with gingival bleeding + hypertrophy in gingiva ,Dx ?


Vit-c deficiency

127-Children BMI is high what the next step ?


Full skin folded thickness

128-Q, redness color in the shoulders since breath ,Dx ?


Strawberry nevus
129-Child drink juice then diarrhea?
Fructosmia

130-Baby drowning his brother undressing clothes witch year?


5 years

131-Baby retate 180 and can't set , coming smile for doctor?
4-5 months

132-Best image for Dx intussusception :


Us (test of choice)

133-Baby with recurrent diarrhea + his capillary refill more than 3 sec + his BP
80/40 (sever dehydration) next step?
20ml/hr/kg bullous

134-10 year baby came with diarrhea sometime bloody + joint pain + IDA , what
is ttt Dx :
crohn's disease , RX : Aminosulicylates

135- Pic of baby with umbilical mass?


Umbilical granuloma

136- 2 Weeks old baby with violent vomiting + epigastric mass. Next step?
Us (pyloric stenosis).

Pyloric stenosis is an acquired condition caused by hypertrophy and spasm of the


pyloric muscle, resulting in gastric outlet obstruction. It occurs in 6 to 8 per 1000 live
births, has a 5-to-1 male predominance, and is more common in rst-born children.
Infants with pyloric stenosis typically begin vomiting during the first weeks of life, but
onset may be delayed. The emesis becomes increasingly frequent and projectile and
forceful as time passes. The vomited material never contains bile, because the
gastric out- let obstruction is proximal to the duodenum. This feature differentiates
pyloric stenosis from most other obstructive lesions of early childhood. The stomach
becomes massively enlarged with retained food and secretions, and gastric
peristaltic waves are often visible in the left upper quadrant. A hypertrophied pylo-
rus (the “olive”) may be palpated.
Treatment of pyloric stenosis includes IV fluid and electrolyte resuscitation followed
by surgical pyloromyotomy. Before surgery, dehydration and hypochloremic
hypokalemic metabolic alkalosis must be corrected.

137- Ashmatic child his family stat that thy can not go out becouse the
nebuluzer machine need electicity, what u wll do to help them?
A. Tell them they are right.
B. Provide them with nebulizer with battery.
C. Clearly instruct the inhaler and tell them to stop the nebulizer.

138- Peadiatric pt with Testicular swelling translumination negative what u wll


do?
A. mish repair.
B. herniotomy.
C. hernigraphy.

139- peadiatric pte with bilatral testecular swlling increase when he is crrying
what to do?
A. herniotomy.
B. mish repair.
C. laproscopic mesh repair.

140- Olive.....pyloric stenosis Target sign....intusseption >>>


A- US -16-

141- Child with Hx of stem cell transplant few months back, brought to ER by
his father because his brother developed chicken box this morning. What are
you going to do?
A. Acyclovir
B. Varicella vaccine
C. Varicella immunoglobin
D. B+C

142- Asthmatic child, his family can not go out because nebulizer machine
need electricity, what you do to help them?
1- Tell them they are right
2- Provide them with nebulizer battery
4- Clearly instruct the inhaler and tell them to stop nebulizer

143-question about familial short stature


144-2 years child with WBC 21.low HB low platelet. Diagnosis

145-Di george syndrome.ask about the. Diagnosis

146-What vaccine giving in 1year ?


MMR

147-Child with egg sensitivity. What vaccine not to give ?


yallow

148-Child above 95th centile and look larger than other child in same age .
Investigation?
Initial >> IGF1Best >> OGTT

149-K/c of cervical incompitance came for antenatal care. When to do surgery?


12 to 14 weeks

150-Child with history of eczema in knee and flexure elbow. With maximum
cortisone dose. And the eczema extending. What to do?
Tacrolimus

151-Child with croup. Repeat Epinephrine


If mild >> dexa Moderate or severe >> epinephrine

152-Double bubble
Doudenal atresia

153-4 year Child with fever . Then convulsions treatment?


manage fever, paracetamol

154-Down syndrome patient. with fixed split S2. And pansystolic murmur in
left sternum. What cardiac abnormality?
AVSD
155-Child sleep with milk bottle in his mouth. He develop dental Carie's.
Diagnosis?
A-Babby bottles induced tooth decay.

N.B: Babby bottle induced tooth decay is caused by the frequent long term exposure
of the child's teeth to liquids containing sugars such as milk, fruit juice and other
sweetened drinks. The sugar in these liquids pool around teeth and gums, feeding
the bacteria that cause plaque and tooth decay.

156- Child with nephrotic syndrome , amount of protein in his diet ?


A- like any normal child
B- low protein
C- high protein

N.B: High protein diet isn't recommended for nephrotic syndrome cuz it is dangerous
and will damage the nephrons and causes renal insufficiency, so low protein intake
is recommended.

157- 11 years old , with hemochromatosis and hemolytic anemia , management


?
A- venipuncture
B- penicellamine
C- deforaxamine

N.B: Primary hemochromatosis ttt by Venisection ''phlebotomy'' ,,, secondary ttt by


Defroxamine.
N.B: Hemochromatosis in patient with chronic hemolytic anemia is secondary to
chronic reoeated blood transfusion.

158- Pt come to hospital complain of abdominal distension and vomiting and


nausea, examination there is abdominal mass?
A- intussption
B- mid valvular

N.B: Both can be presented with abdominal pain, distension and vomiting, but
presence of susage-shaped abdominal mass on palpation + currant red jelly like
stool is suggestibve of intussusception.

159- 15 month baby, he talk only 3 word, good development?


A- reassure the normal hearing
B- hearing test and refer to ENT

160- Pt pediatric need to exercise?


1-120min
2-60
3-30
4-15
161- child 1 year, brother died due immunodeficiency at same age. What
vaccine not give?
A. Varicella
B. inj polio
C. pneumococcal

162- Feel on the right side of his abdomen , mother noticed bulging what is the
diagnosis?
A. liver contusion
B. wilms
C. neuroblastoma

163- 3 years old dropped on his head, no LOC and is oriented but he vomited
twice and complaining of headache ,what will you do?
A. CT
B. MRI
C. Reassurance

164- Baby had teeth discoloring and the mother mentioned that she leaves the
bottle
On the baby’s mouth while sleeping?
A. Bottle related
B. syphilis

165- A6year old who never received vaccination except BCG at birth he is
hepB+ve ,what vaccines should he receive now?

166- Milestone of a baby can draw triangle but not square


A. 3 years

167- neonate complain diaphragmatic hernia not cry, what do to support him
A. chest tube
168- Child admitted in ER due to DKA, severely dehydrated Managed with fluid
and insulin, bad management complications can occurs
A. hyperkalemia
B. hypoglycemia
C. brainoedema
D. hyperglycemia

169- Child in Er DX as DKA managed by fluid and insulin ,serum electrolytes


will be imbalanced
A. can
B. k
C. na
D. mg

170-Kawasaki case diagnosis?


A- bilateral conjunctivitis without exudate

Kawasaki disease is necrotizing febrile vasculitis of medium-sized vessels that


primarily affects the large coronary blood vessels. It occurs in children. Look for
a child with more than 5 days fever and all 5 of the following criteria:
1. Rash
2. Mucositis
3. Edema or erythema of hands and feet
4. Cervical lymphadenopathy
5. Limbic-sparing bilateral conjunctivitis
Other symptoms suggestive of Kawasaki disease are elevated WBC and platelet
counts, transaminases, and acute phase reactants, as well as anemia and pyuria.

171 -awasaki case ( there was no IVIG result , asking about which one of the
things if presents whould mean that the CPR high)

172-Child with Cafe au late sopts , mother said that this is birth mark in her
family.?
A- send for more genetic education
B- send for genetic to health education about neurofibromatosis
C- counseling on neurofibromatosis type 1

173-Child come with status epilepticus When to say it’s sever ?


A- 2 times PO2 < 6.5
B- on time PO2 < 8
C- 2 times PO2 < 7.7
D- one time PO2 < 7.3

174-Child 3 years with symptoms of kwashiorkor (from rural area)


What’s the cause ?
A- deficiency in dietary protein

175--2 months baby died suddenly without any signs of abuse or fracture or
bruises, his mother is smoker and divorced
A- sudden infant death
B- maternity smoking ....

176-Researcher want to measure obesity in children he took BMI what else he


should take?
A-girth measurement
B-HDL/LDL level
C-dietary habits
D-forgot

177-Pediatric pt go with her mother to a party and suddenly he complain of


itching all over the body and S.O.B then came to the ER what is the important
question you need to ask about:
A/history of food
B/history of eczema

178-4years what can done


A-tell story and account to 4

179-child complain of cervical lymph node enlargement and had family history
of lymphoma in his brother and….
What is the important lymph node to be detected to diagnose lymphoma
A-supraclavicular lymph node
B-preauricular lymph node

180-Pediatric Pt came with hematuria after skin lesion.


A-HSP
B-ITP
C-POST streptococcal
181-Pt Pediatric go to dentist and complain with gum bleeding, dentist ask his
mother any hx of bleeding, she say is petechiae and bruise
A- hemophilia A
B- hemophilia b
C- ITP

182-pt Pediatric, conclusion of deficit


1-500

183-pt Pediatric, conclusion of maintenance


1-1000

184-mother came with her child she is complain, there is bruises in labia
majora
A- abuse

B- Foreign body

C- trauma

185-girl came to ER with child abuse, what the prove of abuse?


A- hymen injury at 6clock


186- A child with UTI treatment regimen with duration

187- Probability of turner syndrome in next pregnancy.


30
40,
50
60
187- A child every developmental is normal except speech,diagnosis?
Isolated expressive delay

188- baby with vomiting and diarrhea , looks ill cries with tears CR 3 sec
vitally stable tt?
A. Moderate dehydration give Oral fluid
B. Moderate dehydration give IV fluid
C. Sever dehydration give bolus fluid

189- A child with slightly increased Na and hypoglycemia, what fluid will you
give?
D5

190- Varicella vaccine 1dose an year ago, what to give now


Give now 2nd dose

191-Child with mingitis , papilledema , important complication to inform


parent is
A-Visual loss
B- Hearing loss

192- baby deliverd at home came with thigh bruise


A. Idiopathic thrombocytopenic purpra
B. Heamorrhagic dis of neoborne
C. hemophilia
D. factor X deficiency

193- pediatric on ventolin inher his mother notice asthmatic attack twice per
wk came for hospital one time in last month examination clear chest what to
do?
a)steroid inhaler
b)increase ventolin frequency

194- Patient e fractured humerus and ulna he cant extend his arm wrist fingers
which nerve injury
A.Ulnar
B.Median in cupital fussa
C.Medial lateral epicondil
D.Radial
195-Milestone of a baby can draw a triangle but not square :
4 years

196-Baby delivered at home presented 65 days later with rt thigh bruises other
exam unremarkable ( PT high , PTT high , other normal ) what is the dx?
A. hemorrhagic diseases of newborn
B. factor x deficiency

197-Baby delivered at home with umbilical 1-bleeding what the cause of this
bleeding?
A-factor Vlll
B- factor X
C- factor XIII
Note : if baby born in home and comes with bleeding its due to ( vit.K ) deficiency
Note: Vit k depended factors (2,7,9,10)
Note : bleeding from umbilical after clamping after birth due to ( factor XIII )
deficiency
Note : bleeding after circumcision due to ( factor VIII ) deficiency : Hemophilia A

198-Question directly related to Saudi vaccination guidelines, child aged


1 years, asked about what vaccine in this age?
MMR , PCV, MCV4,OPV-

199- Child aged 3 years old brought by his mother with episodes of
crying, fever, productive cough and drooling of saliva. 1-2 weeks ago
mother reported that her child was complaining of bilateral
conjunctivitis. Which of the following is the most causative organism?
A- Mycoplasmapneumonia
B- Adenovirus
C-Chlamydiatrichomonaspneumonitis

Viral infection from age 3 month to 5 years is the most common organism

200- Neoborn delivered at hospital, healthy, but mother reported that the
previous baby died due to immunodeficiency. What is the best action
regarding Saudi immunization program?
A-Don't give BCG
B- Give BCG with close observation
C- Referbaby
we cannot give baby BCG vaccine if there is history of immunodeficiency

201-A child has immunodeficiency. Which vaccine is contraindicated?


A- Varicella
B- Pneumococcus
C- Influenza
D- InjectablepolioIPV is inactive-

202-Mother came to hospital with her healthy normal weight 6 weeks old child,
she said that with each feeding the child passing informed stool. What to do?
A- Change Milk formula
B- Giverehydration solution

203- Child 1 years old on normal formula but passing bloodless diarrhea,
vomiting and abdominal pain since. What to do?
Change formula to cow free milk

204-Child 2 months, cry 1-2 days per week, not sleep well for 1 month. Normal
weight. Otherwise healthy. Dx?
Infantile colic
205- Child diagnosed as a case of nephrotic syndrome. Anyway, now the only
complaint is generalized edema. Management?
Frusemide

206-A 12 years old girl who is a known case of DM 1 diagnosed 2 years


ago. Now came to do routine checkup. She has no features of puberty.
What test to do annually?
A- Ophthalmology
B- GH assay!!!

207-When to do surgery for child with congenital atrial septal defect?


••depends on the size!!!
••depend on the site!!!

208-Case of male patient with joint and abdominal pain, splenomegaly,


not responding to NSAID. Dx?
Felty syndrome
-triad of RA , splenomegaly neutropenia-

209- Newborn of diabetic mother needs to take glucose infusion. Where to give
glucose through?
A. Central venous line
B. Peripheralvenous line
C. Nasogastric tube
.D Orogastrictube
210- Duringventose instrumental delivery there was an injured to
stylomastoid foreamin. What will happen to baby?
••Incomplete eye closure!!!
Bell’s palsy

211- Newborn of diabetic mother from where take sample


A-central venosuline
B-peripheral
C-nasogastric
D-orogastric

212- Which of the following is true about ASD in a 6 years old boy?
A-It will close by 5 years
B-Surgery should be done at 2 years

213- 3 weeks old infant with blood drops in diapers noticed 2 times by
mother when she change diapers. Dx?
A -Juvenile polyps
B-Mickels diverticulum

214- Child brought by his parents With meningitis. You need to do


emergency LP. What to do?
A••Do Lp without consent
B••Take consent from parents
C••Take consent from parents and child
D••Abstent from child, consent from parents
E••Abstent from parents, consent from child

215- Child is treated for eczema with topical steroid, comes to clinic with
itching and pastular lesions on top of his eczema, arrranged in grape like
pattern. What is the most likley organism that causes his superimposed
infection?
A••Herpes simplex
B••Staphylococcus aureus
C••Group A streptococcus

216- Child come with fever and headache, there is menengealirrtation sings on
examination, LP done with results. Glucose 2.9 (Normal 2.4-4.1), Protein 560
(Normal range 150-450). What ttt is best? ••Ampicellin + vancomycin!!!
A••Vancomycin + rifampin
B••Ampicill + somthing
C••Vancomycin + ceftriaxon
If bacterial, treatment according to age:

If viral : acyclovir
If TB: RIPS+ steroids 9months

217-Child come to clinic with cough, fever (38.6) for 3 days with rash started
on face then spread to trunk and white pustule on erythematous base on the
buccal mucosa (opposite the lower 1st & 2nd molars) Dx?
A••Measles (Koplik's spot)
B••Rubella
C••Scarlet fever

218-10 year old boy come with hx of diarrhea more than one month, there is
abdominal pain with blood somtimes and mucous sometimes, the child hasn't
been loosing weight, the child is reporting that he feels incomplete evacuation
even when there is nothing left in his bowel. What would be appropriate to try
first ?
••MetronidazoleCuz Amoeba infection
A••Gluten free diet
B••CT
C••Some other option

219-Child 5 years old diagnosed with tonsilitis, and asking about his brother (2
years). Prophylaxis?
A••I chose penicellin
B••Other cant remember

220-Child with abdominal pain which one reflect seriousness


A. Pain in day time
B. Pain in sleep time

221-Child has flu brought by his mother for vaccinations and mother wants to
postpone because he is sick, on ex mild flow, child is active what to do
A. Reschedule in 2 wks best answer is explain to the mother then give him
B. Ignore

222-What is the safe for baby?


A. mother HIV and direct breast feeding
B. mother HIV with expressed breast milk
C. mother TB direct breast feed.
D. mother TB with expressed breast milk

223- Vaccine contraindicated in egg allergy patients?


A. Yellow fever

224- Child can't collect 3 words in a sentence, can go upstairs. Age?


A. 2 years

225- Nocturnal enuresis occurs at which age?


A. 5 and above

226- Child with features of Nisseria meningitis (similar to HSP). Asking about
what to do with his child brother?
A. Ciprofloxacine
B. Ceftriaxone
C. Rifampicin
D. Vaccination

227- Case about measles. Dx?


A. Measles
228- Case of Croup. diagnosis?
A. Viral Croup

229- Case of Croup. Organism?


A. Parainfluenza

230- Case of child with BA. Mother worried about the disease in his child in the
future. What to tell her?
A. Bronchial asthma will continue in future
B. BA will improve in future
C. BA in children is not related to BA in adult

231-Infant crying abd pain,he pass jelly and bloodystool what is the most
appropriate invest to reach the diagnosis
A-Barium enema
B-Abdxray
C-AbdUS
D-AbdCT

232-Child abd pain asse headache physical exam normal


A-Abd migraine
B-Other choice Ididn't remember
233--2yrs old presented with fever and cough and resp distress on exam there
is bilateral crackles Xray showed both lung-infiltrate and something I forget
what is the dx?
A-Bronchopneumonia
B-Bronchoectasis
C-Cysticfibrosis
Fever + productive cough >> pneumonia

234- Child had fever for 5 days, erythemateous red lip and peeling of the palm
with conjuncivitis. Dx?
A- HSV.
B- Infectious mononucleosis.
C- Kawasaki disease.

235- Child had fever and dark urine. On exam there is mild tenderness in
abdomen, UA showed RBCs and +2 protein. TTT?
A- Frusimide.
B- Predinsolone.
C- Ceftriaxone.
236- 15 years old female had short stature and short neck. She didn't get her
cycle yet. Dx?
A- Turner.

237- Father brings his child due his brother died of immunodeficiency. What
vaccine should not be given in this child?
A- Influenza.
B- Inj polio.
C- Varicella.

238. developmental age of a child... can sit without support, friendly with
strangers but cannot play peek a boo....
a. 6-7 month
b. 7-8 months
c. 8-9 months
d. 9-10 months

239- Newborn with eye hemangioma that occludes the eye completely and
your concern to
not develop amblyopia when to do resection (surgery):

A. .A. one day


B. one week
C. one month
D. six month
240- New recommendation for sudden infantile death syndrome

A. Pacifier
B. Lying in prone position
C. Position support devices

242- short stature child. No family hx

A. Consisionl
B. genetic
C. hermonal...
D. question may not complete
244- Child presented with petechiae and his platelets is 15 , otherwise healthy.
What will you
do for him?

A. Splenectomy
B. IVIG
C. Observations....
D. steroid

245- 2 weeks neonate passed unformed stool. What will you do?

a. Prescribe formula milk.


b. Give oral rehydration solution.
c. Prescribe Lactose-free milk...

246- Baby thirsty , tachycardia , sunken eye , dehydration :

A. %9-4...
B. %0
C. < %2
D. %9>
247-child fell on outstretched hand pain in distal forearm what diagnosis...

A. A.bartons
B. B.something
C. C.smith...

Distal radius fracture. A Colles fracture.

248-Mumps complication in child ,?

A. Meningitis...
B. Enephalitis
C. Orchitis

249/ 4 years old child developed resistance or not respond to Chloramphenicol


in Salmonella infection, what you will give:

A. IM Ceftriaxone> children

250/ Pediatric case scenario indicate respiratory distress syndrome , ask about
the deficiency:

A. dipalmitoyl
B. phosphotidylcholine*

251- / Milestone baby can hold his head and when he looks at his flying hand
he laughs and coos?

A. Answer: 4m
Section3
Obgyn……
1. 48 years old female came with abnormal uterine bleeding. TVU shows sub
serous fibroid and endometrial thickness is 14 mm. Dx?
A. Fibrosarcoma
B. Myosarcoma
C. Fibromyoma

2. Case scenario of female patient gets menopause. Then she developed


dysfunctional uterine bleeding for the last 3 months. TVU showed multiple
endometriosis with thickness about 19mm. What to offer for this woman?
A. Ablation of endometriosis
B. Total abdominal hysterectomy- best management -
C. Laparoscopic hysterectomy
N.B If the next step, do endometrial biopsy.

3. 64 Years post-menopausal bleeding since 5 days HB 11 Pt ,PTTnormal,tvus


revealed normal except endometrial thickness 13mm, Best management?
A. Progesterone
B. Endometrial biopsy
C. Hysterectomy
D. Reassure
N.B: any women with postmenopausal bleeding, should be undergo TVU, then: if
endometrial thickness more than 5 mm so endometrial biopsy, if 5 mm or less so
repeat TVU after period of time.
N.B: Indications of endometrial biopsy: any abnormal uterine bleeding with one of the
following (age more than 40, DM, PCOS, family history of non-polyposis colorectal
cancer, obesity, nulliparity, failure of medical treatment of this AUB, significant
intermestrual bleeding, TVU showed endometrial thickness > 5 mm).
N.B: Methods of endometrial biopsy (office endometrial biopsy which is the best) or
(D&C) or (Hysteroscopy).

4. Pt had menopause at age of 40. She is at increased risk of?


A. increase risk for osteoporosis. √

5. Post-menopausal flushing mechanism?


A. Vasodilation
B. B-High FSH
C. Vasoconstriction
N.B: to confirm diagnosis of menopause (elderly with hot flushes + night sweats) =
FSH
N.B: pathophysiology of flushing in menopause = low estrogen leads to
vasodilatation (Consultant Obe/Gyn).
N.B: Regarding hormonal replacement therapy. We should know if she have uterus or
post hysterectomy to assess her for endometrial cancer risk (if post hysterectomy and
now need HRT so give Estrogen only HRT) (if intact uterus and now need HRT so give
Combined HRT)

6. Postmenopausal women come to take HRT. You said she didn't need it and
no evidence to take HRT. She insisted to take it. What to do?
A. Prescribe HRT to her
B. Don't give it /Refuse
C. Consult Hospital authority/ committee
D. Refer to another Gynecologist

7. What prevents uterus from prolapse?


A. Round
B. broad,
C. uterosacral ligament

N.B: The cardinal-uterosacral ligament complex provides apical attachment of the


uterus and vaginal vault to the bony sacrum. Uterine prolapse occurs when the
cardinal-uterosacral ligament complex breaks or is attenuated.
https://www.ncbi.nlm.nih.gov/pmc/articles/PMC2034734/

8. Female after uterus prolapse surgery have the urine from


vagina what is dx?
A. uretrovaginal fistula
B. -urethrovaginal fistula
C. vesicovaginal fistula

9. Urge incontinence principle management?


A. Medical
B. Surgical
C. Medical and surgical
D. Bladder training and physio

N.B: anticholinergic medication to relax detrusor MM.

10. Urge incontinence tx?


A. Kegel exercise
B. antibiotic incide,
C. surgery
11. chronic stress and urge- pelvic ms weakness

12. A 21-year-old woman presents to the gynecology clinic with a mass in the
left breast. She discovered this mass while showering. Her last menstrual
period was 10 days ago. There is no family history of breast cancer. On
physical exam, you palpate a 3 cm firm non-tender mass in the upper lateral
quadrant of the left breast. The mass is smooth, well circumscribed, and
mobile. There are no skin changes, nipple discharge, or axillary
lymphadenopathy. Which of the following is the most likely diagnosis in
this woman?
A. Fibroadenoma
B. Fibrocystic change
C. Intraducalpapailoma
D. Lobular carcinoma in

13. 30-year-old female pain with period and breast mass and disappear after
period, what is your next step?
Re. evaluation next period

14. A woman with a history of breast cancer. Which of the following is


medically contraindicated for this woman?
A. Condoms
B. Copper containing intrauterine device
C. Laparoscopic tubal ligation
D. Progestin-only contraceptive pills

15. Which of the following is the most common cause of hereditary breast and
ovarian cancers?
A. Alpha-fetoprotein
B. BRCA mutation ✅
C. CDH1
D. p53 mutation

16. Female patient diagnosed as a case of ovarian cancer. What is the most
important tumor marker for follow up?
A- C125
B- AFP

17. Pt with breast cancer and fibroid on tamoxifen develop vaginal bleeding US
show fibroid 8cm and hypo echogenic area what is diagnosis?
A. endometrial cancer.
B. lyomyosarcoma.
C. overian cancer.

18. Tamoxifen for breast cancer patient, she has metrorrhagia, US showed
thick endometrium, what is side effect of this drug?
A. decrease risk of DVT
B. decrease risk of pituitary adenoma
C. increase risk Endometrial cancer
D. increase risk of uterus tumor

N.B: Tamoxifen side effects: Increases risk of thrombosis and endometrial cancer

19. Female presented e scanty vaginal bleeding she noticed that postcoital
what is your action
A. Pelvic us
B. CBC
C. Assess the vagina and cervix

20. 65 years old female presented with vaginal bleeding and abd pain Ca125
very high what u will do?
A. biopsy.
B. antibiotic.
C. admit.

21. q about menopause had fibroid before 4 years 4x3 cm come now with
bleeding and in US the fibroid size 7.6 and endometrial thickness 6 (normal
<4) what is the cause?
A. Endometrial cancer

22. Case of endometrial cancer what is the mx


A. TAH + BSO

23. Screening of cervical CA. To which age group?

https://www.uspreventiveservicestaskforce.org/Page/Document/UpdateSumma
ryFinal/cervical-cancer-screening

24. Female with SCC on pap smear. What to do?


A. chemo
B. hysterectomy
C. radio
D. colposcopy
25. 56yrs postmenopausal her pap smear hx was normal except one month ago
revealed ASCUs, then given local estrogen for one month then pap smear.
Repeated revealed intra epithelia lesion, Next step is?
A. conization
B. hysterectomy
C. colposcopy

N.B: Any high risk lesion on pap smear (high vaginal swab) should be followed by
colposcopy

26. You are seeing a 78 yrs old lady in clinic. She looks sick had significant
weight loss and feels tired all the time. She has pelvic mass. What is the
most appropriate screening test to confirm the diagnosis:
A. US
B. Colposcopy
C. Ca-125
N.B: Colpo for cervical ca , Endometrial biopsy for endometrial ca , US then surgical
biopsy for Ovarian ca.

27. Abnormal pap smear insignificant result with colposcopy, next?


Cone biopsy
N.B: Cervical cancer is suspected based on Pap test results, colposcopy, and
cervical biopsy. A cone biopsy can determine the extent, depth, and severity of the
cancerous tissue and can guide treatment decisions.

28. what is the age recommended for screening by pap smear for married
women?
A. 20-24
B. 26-30
C. 30-35
D. 36-40

29. Pap smear screening is done every?


A. 2 years
B. 3 years
C. 5 years
N.B: Based on the age and HPV testing, but in general it is done every 3 years.
N.B: Pap smear if less than 21 years so no need.
N.B: Pap smear if between 21 and 29 years so pap test every 3 years.
N.B: Pap smear if between 30 and 65 years so pap test every 3 years or pap test +
HPV test every 5 years.
N.B: Pap smear if more than 65 years so no need if previous results are normal.
N.B: Pap smear in married women should be done after 3 years from marriage
regardless of the age.

30. Pt 40 years old, what to tell abt pap smear?


A. 5 years
B. 3 consecutive neg results no need for further

31. 27 years old female she did Pap smears before 3 years it was normal what
is your advice for her ?
A. reassurance
B. no pap test
C. pap test with cytology

32. How to do a Pap smear


From transformation Zone (endocervix)

33. Pt asymptomatic with 5 cm serous fibroid tx:


A. myomectomy
B. hysterectomy
C. follow up

34. Female patient with fibroid wants to keep fertility what is the tx?
A. laproscopic hestrectomy
B. laproscopic myeomectomy
C. laparotomy myeomectomy
D. laparotomy hestrectomy

35. Pregnant wants to quote smoking best way


A. CBT
B. Nicotine replacement
C. Buprine
N.B: smoking + pregnancy … ->IUGR

36. smoker with fetal demise cries


that her smoking killed her fetus
A-Tell her…
B-Say the cause not yet clear
37. Pregnant lady with late deceleration and
bleeding?
A. Placenta abruption

38. 36 weeks pregnant with history of smoking


came complaining of vaginal bleeding with
uterine tenderness. Diagnosis?
A. Abruptio placenta
B. Placentaprevia
C. Vasaprevia
D. Placenta accrete
N.B; VB+ tenderness + smoking history

39. Pregnant in her 37 week fainted and complain of severe abd pain, no
bleeding, pb 80/50, HR120, What is the diagnosis?
A. pulmonary empolism.
B. abruptio placentae

40. Pregnant complain of painful vaginal bleeding is it


A. Placenta abruptio
B. placenta previa

41. Pt had a previous C-S. Now pregnant at 36 weeks with abdominal pain. Vitals:
hypotensive, tachycardia. Dx?
A. Uterine rupture
B. Abruptio placenta.√
N.B: Hypotension + tachycardia are signs of abruption of the placenta.

42. 36 weeks pregnant lady has features of abruptio placenta, severe bleeding,
hypotensive, tachycardia and anemia. What to do to safe live?
A. ICU admission and
multiteam work
B. Insert 2 large bore
peripheral cannulas
and blood transfusion
C. Delivery room
N.B: The initial most important
step in Abrupto is immediate
resuscitation at ER to stabilize
patient.

43. Pt with placental abruption


BP 80/40 , what u will do?
A. us
B. admission for SC
C. maximum response team and multidisciplinary plan

44. placenta previa and she has contractions, what to do?


A. Cs

45. Pregnant has abruptio placenta, what is the acid base balance of fetus
Asphyxia?
A. Res acidosis..
B. Metabolic acidosis
N.B: Pethidine cause mixed acidosis, Placenta abruptio metabolic acidosis, Pre-
eclampsia respiratory acidosis.

46. Picture about variable


decelation in ctg what is the
cause, is it Abruption or cord
prolapse
Late de.= Placenta insufficient
Early de.= head
Virable= cord

“Umbilical cord prolapse occurs


when the umbilical cord delivers
through the cervix ahead of the
presenting fetal part. It is
associated with abnormal fetal
heart tracings, malpresentation,
and an unengaged fetal
presenting part.”

47. Management of
late deceleration?
A. change position.

48. picture of CTG


and ask about
diagnosis (there is
variable
deceleration)
A. cord
compression
B. placental
insufficiency
49. CTG (3 contraction
without
deceleration) what is
your dx ?
A. head compression
B. cord compression
C. uteruplacental
insufficiency
D. none of the above

50. CTG shows late


deceleration, Dx?
A. Head compression
B. Cord compression
C. Placental insufficiency

N.B: CTG, deceleration


Early -> head
compression
Late -> placenta
insufficiency
Variable -> cord
compression

51. Pregnant lady in


labor with active
regular
contractions every
3 mins and cervix
dilated 5 cm. After
4 hrs still the same,
and CTG sows
grade 1 heart
activity, What will u
do?
A. oxytocin
B. instrumental
delivery
C. CS

52. Pregnant during labor CTG


showing contractions and
deceleration of heart rate of fetus
she was given analgesia and
oxytocin what is causing the
deceleration?
A. Oxytocin
B. Anesthesia
Bc it causes tense uterine contraction and reduces blood transported to fetus.

53. Almost same question but different CTG showing no change in heart rate of
fetus during contraction, what’s the cause?
A. Oxytocin
B. Anesthesia??
C. Analgesia??
N.B: These 2 depend on CTG picture to determine type of deceleration

I had 3 ctgs
One early ...head compression can be in oxytocin.
One late ... anesthesia induced maternal hypotension lead to placentsl
hypoperfusion and fetal distress “Think abt it when the mother in epidural
anesthesia”
One was normal ctg no deceleration normal variability
If tendon reflex w hypotenstion and late deceleration mentioned = MgSO4 toxicity

54. Pt with multiple repeated contractions and sluggish fetal heart rate
response on CTG. Mother was given epidural analgesia, given oxytocin to
induce labor, given MgSO4 due preeclampsia. What is the cause of this
CTG?
A. Oxytocin
B. Analgesia
C. MgSO4

55. 34 weeks pregnant does not feel her baby movement, she goes to the
doctor and did CTG was reactive, what is your management?
A. Urgent CS
B. Bpp
C. Refer for admission
D. Reassure

56. Contraindications of ECV?


a. Placenta previa

57. Women pregnant 34+2 weeks had cesarean section past pregnancy due to
non-reassuring fetal monitoring, present now for external cephalic version,
US amniotic fluid index 14, responsive CTG. What would be absolute
contraindication for ecv?
A. Previous cs
B. CTG results
C. US findings
58. 43 year old she has 3 children the first child was by C -section the other 2
were vaginal birth, now she is pregnant (36 weeks) comes with Rupture of
membrane, by us placenta previa, AFI 4 cm,CX is 2 cm 50% effaced. What is
the Absolute CI for ECV ?
A. previous CS ??
B. her age
C. US result
N.B: C/I of ECV are US results of oligo or polyhydramnios + placenta previa +
previous C-S. N.B: CI of instrumental vaginal delivery is cephalopelvic disproportion.

59. Pregnant in her 38 week and 2 weeks ago she was admitted and ECV was
done for her. Currently the fetus position is in linear (or lateral not sure)
with amniotic fluid index 12. What’s the c/i for ECV in her condition?
A. Fetal position

60. 34 weeks pregnant came with breech presentation what is your next step?
A. Fu 36 weeks -ECV

61. 31-year-old pregnant (37 weeks) ROM in labor on exam breech


presentation, CTG reactive PV CX 2 cm, 50% effaced, mid anterior Cx.
Management?
A. CS
B. oxytocin
C. vagprostaglandin
D. IV abx
N.B: Breech ppt is one of the indication of CS.

62. Pregnant women has hard lump


at epigastrium , FHS is heard at
umbilicus what is Fetal
presentation?
A. Face
B. Brow
C. Breech
D. OP

63. US of pregnant shows breech


position, both legs and hip
flexed, asking about breech
Complete breech

64. Dizygotic twins


A. Monochrionic monoamniotic
B. Monochorionic Diaminotic
C. Dichromic monoamniotic
D. Dichromic diamniotic regardless of gender.

65. Best presentation for


NVD in case of twins?
A. breech cephalic
B. cephalic breech
C. cephalic cephalic
D. breech breech

The best is : cephalic


cephalic. Also it’s the most
common in twins
pregnancy

66. Smoking risk to


pregnant
A. low birth wt for fetus (IUGR)
B. respiratory problem of the baby
C. preterm baby

67. Pregnant lady, G4p3 ,35yo, 34GA came complaining of bleeding 2hours
ago, what question will you ask next?
A. Intercourse
B. Is she satisfied with her family?

68. 34 years G4P3 GA 32 she never came to ANC visit, what is the possiple
cause?
A. visit is expensive
B. Ignorance
C. scaring from visit

69. Pregnant woman with hepatitis, what is the type of prevention when you
treat her?
A. primary
B. secondary
C. tertiary
N.B: Primary prevention = vaccination and immunization.
N.B: Secondary prevention = Screening.
N.B: Tertiary prevention = Treatment and rehabilitation to avoid complications.

70. Lady came w her husband who started HBV


treatment, she is negative what to advice
about sexual relation?
A. Tell her to use condom
B. To have normal intercourse
71. 40 year female primigravida with Hx of DVT . RX?
A. Heparin
B. Enoxaparin

72. Female in productive age , she have symptoms of DVT , your investigation
?
A. Venous duplex
B. venography
C. plethro...
D. CT angio

73. A pregnant 32-year-old paraplegic woman underwent a cesarean sec􀆟on. A


week later she developed a deep venous thrombosis in her lower limb. If
the thrombus breaks, which of the following would first receive the
detached thrombus in its capillaries bed?
A. Left Kidney
B. Lungs
C. Heart
D. Brain

74. Pt at 22 wks has abdominal pain then


feel decrease of fetal movement and feel
SOB, …etc what is the dx?
A. Amniotic fluid embolism
B. Septic shock

75. 40 weeks of gestation primi gravida


presents with hypoxia, drowsiness and
agitation for 6 hours, LLedema (Long
scenario with blood tests). What does
she have?
A. Amniotic fluid embolism.
B. PE

76. A pt treating for inferior wall MI in the


hospital developed SOB due to pulmonary
thrombosis what to do?
A- give thrombolytic
B- give LMWH
C- Go for PCI

77. Post-partum has dvt started on enoxaparin then developed PE :?


A. Same
B. Give thrombolytic
**PE treated by anticoagulant Thrombolysis in PE has indications in unstable patient
with hypotension
78. Uterus Compress Inferior Vena cava will cause?
A. Hypotension
N.B: This is a case of IVC syndrome ''aortocaval compression syndrome'' in which
the utrerus in late pregnancy when pregnant women lies on her back will compress
IVC and aorta resulting in maternal hypotension.
N.B: Aortocaval compression is thought to be the cause of supine hypotensive
syndrome. Supine hypotensive syndrome is characterized
by pallor, tachycardia, sweating, nausea and hypotension and dizziness and occurs
when a pregnant woman lies on her back and resolves when she is turned on her
side. Medical management of supine hypotensive syndrome can include turning the
patient to the left recumbent position (so the uterus is not sitting on the IVC) and
administering IVF.

76. 10 weeks pregnant come with weight loss 2 kg +fatigue + dec appetite +
vomiting after each meal what is the cause of these symptoms?
A. H. Pylori
B. hyperemesis gravidarum

77. Which Ig can cross placenta?


Ig G

78. 12 weeks pregnant has gallbladder


stone since 5 years on medication
and now she is having pain when
to do cholecystectomy?
A. now
B. 2nd trimester belter laparoscopic
cholecystectomy √√
C. 3rd trimester
D. after delivery

79. pregnant lady with recurrent pain


known case of gallstone what will u
do?
A. open surgery in the second
trimester
B. belter laparoscopy in 2nd trimester.
C. surgery after delivery.
80. 10 wks pregnant with cholecystitis when to
do surgery
A- Now
B- 1st tri
C- 2nd
D- After delivery

81. Pregnant with mild systolic ejection


murmur along the left lower sternal border
radiate to carotid?
A. Physiological of pregnancy. (no radiated)
B. Tricuspid regurgitation.
C. Aortic stenosis

82. Pregnant 2nd trimester -> high BhCG, indicate what?


A. normal pregnancy
B. complete …?
C. Down

N.B: High BhCG in 1st trimester


= Molar pregnancy.
N.B: High BhCG in 2nd
trimester = Down syndrome

83. In what week of


gestation do alveoli form?
A. 12 weeks
B. 16 weeks
C. 24 weeks
D. 28 weeks

84. Patient in 3rd trimester have


high blood glucose despite
close observation, What is the
suspect cause?
A. neonate hyperglycemia
B. neonate hypoglycemia
C. mother hyper...
D. mother hypoglycemia
85. When you will do screening for GDM in weeks?
A-14.
B-24.
C-34.

86. Gestational diabetes is


A. Normal pregnancy associated with
diabetogenic condition due to A
progesterone
B. Same as above due to estrogen
C. Due to FSH

87. Save in pregnancy with DM ?


A. metformin
B. insulin

88. Women diabetic controlled developed dysuria and frequency urine analysis
nitrate, creatinine high, which drug is contraindicated>
A. Amoxicillin
B. Septrin (co-trimoxazole)
C. Ciprofloxacin
D. Pipracilin tazopactam

87. Pregnant lady with elevated


BP and significant edema
nothing was mentioned
about proteinuria, how to
manage?
A. Diuretics

88. Pregnant 36 weeks with


HTN and plasma protein is
high. Dx?
A. Preeclampsia
B. Eclampsia
C. Gestational HTN

89. Pregnant 37wks came to the clinic for follow up BP140/90 no headache,
invest on proteinuria what is the
dx?
A-Pre-eclampsia
B-Chronic HTN
C-Gestational HTN
90. Pregnant women at 15 weeks presented with nausea and headache. On
exam the BP was high "168/100", otherwise healthy. Dx?
A. Preeclampsia
B. Gestational HTN
C. Pregnancy induced HTN
D. chronic HTN

91. Pregnant women at 22 weeks presented with nausea and headache. On


exam the BP was high "168/100", otherwise healthy without protein in urine.
Dx?
A. Preeclampsia
B. Gestational HTN
C. Pregnancy induced HTN

92. 39 weeks pregnant her BP - 150/90, urine analysis normal, no headache, no


vision changes, no epigastric pain, her previous BP was below 110/70.
What is dx?
A. preeclampsia
B. eclampsia
C. Gestational hypertension
D. preeclampsia superimposed HTN
N.B: High BP after 20 weeks of gestation + proteinuria = Pre-eclampsia.
N.B: High BP after 20 weeks of gestation + no proteinuria = Gestational HTN

93. Hypertensive lady controlled in


pregnancy may complicated by?
A. Abnormal fetal presentation
B. Abnormal placenta
C. Preterm labor
N.B: Preeclampsia is a risk factor for
preterm labor, but here the scenario is
only controlled HTN so I think it is B
(Obe/Gyne consultant).

94. 24- pregnant women at 22 weeks


presented with nausea and
headache. On exam the BP was
high "168/100", otherwise healthy
without protein in urine. TTT?
A. Hydralazine
B. Methyl dopa
C. Nigedipine-
D. Nitrates

95. Female with severe preeclampsia. What to give?


A. methyldopa
B. nitropr
C. hydralazine
N.B: Mild > methyldopa, Severe > hydralazine.

96. What is the drug of choice for eclamptic seizure?


A. Phenytoin
B. Diazepam.
C. Magnesium Sulfate.

97. Pregnant BP 160/110 at the end of pregnancy, what is the next step ?
MgSO4 (Missing informations)
N.B: The drug of choice to treat and prevent eclampsia is magnesium sulfate
98. Pregnant with pre-eclampsia mild 140/40 with abdominal pain .platelet and ,
uricaerd what indicate severity?
Abdominal pain

99. Pregnant, now bp 140/90, platelets 90, hx of previously severe


preeclampsia. What indication here of severe preeclampsia?
A. bp
B. platelets
C. uterine contractions

100. how to diagnose recovery from preeclampsia


A. BP
B. Protinurea +3 test
C. Protinurea 5 test

101. Pregnant, 10 weeks with HTN never went to doctor before (chronic HTN)
what is the complication?
Pre-eclampsia ( if IUGR in the option , choese it )

102. HTN in pregnancy, what would you suspect? IUGR (smoking + pregnancy
… -> IUGR)

103. (Long Case)... IUGR , witch one of the following can be the cause?
Oligohydramnios

104. When to start education about breast feeding?


A. 2nd trimester
B. 3rd trimester C-after delivery
C. before pregnancy√
105. Breast feeding HIV and TB mother direct and expressed milk which is
safest?
Avoid breastfeeding in HIV and active TB

106. Pregnant have HCV with cracked nipple , which of the following is
contraindicated to breast feeding?
A. HCV
B. cracked nipple
N.B: HCV is not CI for breastfeeding except if there is eroded or cracked nipple.

107. Hep+ve mother asking about breast feeding?


Continue breastfeeding

108. Pregnant lady in her first trimester, she was exposed to rubella 3 days
ago, what you’re going to do:
A. Nothing
B. MMR
C. Terminate the pregnancy
D. Rubella Igg
109. Vaccine should be given
preconception?
A. Rubella
B. Varicella
N.B: Vaccines given in preconception
care are; HBV vaccine, MMR, tetanus
and Diphtheria.

110. 33Pregnant women, her child


is in school, she is afraid of
flaring, what next?
Take influenza

111. Contraindicated vaccine in


pregnancy?
A. HPV
B. Zoster
C. Pnemucocal
D. Influenza

112. Vaccines contraindicated in


pregnancy?
Varicella

113. Pregnant lady with no past history of chicken box .best to prefect her
from disease? .
A. avoid contact with patient
B. IVIG
C. acyclovir

114. 15 yrs female not menstruating yet at clinic by her parents, she is short
stature, short neck, hypertensive. Most appropriate diagnosis?
A. Turner
B. Down
C. TFS
D. Normal variant
N.B: HTN + short stature + webbed neck + primary amenorrhea = Turner.
N.B: Conditions associated with turner syndrome; hypothyroidism + Coarctation of
aorta.

115. 1st puberty sign in female?


A. Adrenarche.
B. Thelarche.
C. Pubarache.
D. Menarche.

116. Case 7 yrs has pubic hair no breast development: normal 8


premature adrenarche

117. Most common affect symptom in premenstrual dysmorphic syndrome ?


Irritability
IF…
Premenstrual syndrome most common is Mood liability

118. 42 years with 2ndry dysmenorrhea and heavy menses, on pv u found


symmetrical uterine enlargement. Dx?
A. endometritis
B. adenomyosis
C. endometrial cancer

119. A young female complaining of abdominal pain that increased with


cycles, stay for 5 days, first 2 days of cycle she missed school due severe
pain (dysmenorrhea). First next step in Tx?
NSAID is the best for primary dysmenorrhea

120. Clinical scenario about ovarian tube torsion?


Sudden severe lower abdominal pain + tenderness ^

121. Pt with sudden acute pain us showed mass in ovary what is the dx?
A. Ruptured of ovarian cyst.
B. Ovarian torsion.

122. Pt with irregular menses and acne and hair in face her wt 60kg :
A. Pcos
B. Hypothyroidsm

123. Metformin role in PCOS ? Decrease insulin sensitivity


Glory 2019 ..

124. Pt with irregular menses and acne and hair in face her wt 60kg :
C. Pcos
D. Hypothyroidsm

125. Metformin role in PCOS ?


Decrease insulin sensitivity
126. Women with PCOS, not planning to get pregnant soon. Tx?
A. Metformin
B. COCP
C. Progestrone only pills

127. Case of PCOS, normal FSH, high LH, testosterone high. What investigation
to do next?
A. Glucose and lipid
B. Thyroid function

128. 30 y old women comes with painless vaginal mass under urethral orifice
that bleeds when touched, picture was provided. What is the diagnosis?
A. Bartholin abscess (wrong option)
B. Other option can't remeber

129. Pic of vulva and papule ->


A. reassurance
B. More investigation
N.B: We should investigate for STDs.

130. pic or discerption -> cauliflower -> condyloma accuminata

131. Dendritic ulcer?


Herpes

132. A female patient came to the clinic complaining of a mass on a vagina she
has a history of repeated unprotected intercourse with multiple partners, upon
examination. she has a wart in the vagina, the causative agent is:
A. Herpes simples
B. Neisseria Gonorrhoea
C. Treponemma pallidum
D. Molluscum contagiosum

133. Female pt with small papule on genital area with central pitting, h/o
unprotected sexual intercourse with multiple partners,
A. Herpes simples
B. Neisseria Gonorrhoea

209
Glory 2019 ..

C. Treponemma pallidum
D. Molluscum contagiosum

134. Inhibition of which of the following is the primary action of oral


contraceptives?
A. Decrease estrogen to prevent the ovulation
B. Decrease GTRH spur at the mid cycle
C. Increase prolactin
D. Suppressing the release of gonadotropins
E. Spermatozoa and thickening cervical mucusa

135. The type of estrogen that abundant by humans...


A. Estriodol
B. Estriol
C. Estrogen
D. Estrone
? Estriol in pregnancy, Estrone in postmenopause., Estradiol in childbearing age

136. Beta HCG +ve but on US no


gestational sac seen?
Extra uterine pregnancy

- Questions about ectopic


pregnancy, diagnosis and
management
137. Multiple questions of female
with right lower quadrant pain
some came with +vepreganacy
and some with -ve what is the
diagnosis? Choices were the
same as I could remember.
A. Acute appendicitis
B. Ovarian torsion
C. Ectopic pregnancy

138. Question clearly states


ectopic pregnancy 6 weeks
amenorrhea what is tx?
A. Methotrexate
B. laparoscopic surgery
N.B: Depends on BHCG:
If < 5000 metho, If > 5000 lab. < than
3,5cm and pt is stable give metho

210
Glory 2019 ..

N.B: Laparoscopic is the best diagnostic and therapeutic.

139. Ectopic pregnancy 6 wks less than 3.5 cm


pt stable with mild pl mx?
A. Serial BHCG.
B. Methotrexate.
C. Laparoscopic.
D. Expectant mx

140. 24 yrs primigravida. Maried 6 months ago,


hx of iregular menses, at ER complaining
lower Abd pain, scanty vag. Bleeding, by us
uterus empety, rt adnexal mass 2,5cm, bhcg
2500, bp 120/70, P 78, RR 14 Mx?
A. Surgical
B. Medical
C. Medical plus surgical

141. Long scenario the conclusion is pregnant


( ectopic, life in rural area ) her husband travel in another country, she is
complain severe pain and her neighbor going with him to hospital, the vital
sign is normal, the mass 2cm BHcg Lower than 5000?
A. Methotrexate
B. Surgery because the distance
C. Vital sign

142. Ectopic pregnancy, 3cm no


cardiac activity minimal free fluid
in pouch, beta HCG:3500 BP 90/60
her home 80km from near hospital,
what is the contraindication of
methotrexate?
A. hcg and us finding
B. vital sign
C. distance of her home to the
hospital

143. Pt with ectopic bhcg 200+


treated with methotrexate came in
few weeks with severe pain and
bhcg 6000 what to do?
If stable = laparoscopy , unstable
= laparotomy

144. Ectopic pregnancy treated with salpingectomy, what is next?

211
Glory 2019 ..

A. No follow up needed
B. Follow up Bhcg after a week and till zero.

145. A female with abdominal pain and last menses 2months back, what is your
next step?
Pregnancy test

146. Pt with irregular menses LMP since 6week first test:


Pregnancy test
147. Bleeding after menorrhea – confirm pregnancy

148. child bearing age with acute abdomen- r/o pregnancy ]

149. 18 yrs primagrvida, married 6 months ago, Hx of irregular menses, at ER


complaining lower Abd pain, bloody vag discharge, us and urinalysis normal,
bp 120/70, P 78, RR14, Next step
A. pregnancy test
B. duplex us
C. ct angio
D. laparotomy

150. Pt with galctohrea and irregular menses first test :


A. Prolactine
B. LH
C. Progesteone

151. *hyperprolactinemia management (2questions, one of them, the wife did


MRI and there is tumor in pituitary gland, the other question ask about the
drugs to treat the patient with)
Bromocriptine (parolodel) then surgical resection through trans sphenoid

152. After how many months you will start call a couple infertile?
12months

153. couple seeking infertility clinic after 3 months, what is ur advise?


try more

154. Couple came to OBGYN clinic, need to check infertility, what to assess
first?
A. start abdominal ex
B. vaginal inspection
C. general appearance

212
Glory 2019 ..

D. PV
N.B: General appearance of the couple might help you to reach diagnosis, for example:
*Hirsutism on the female supports PCOS, Lymphadenopathy or parotid enlargement in
male supports mumps orchitis.

155. Very heavy periods and infertility.


Endometriosis
N.B: Endometriosis + infertility TX laparoscopy

156. Male with infertility, low testosterone, low FSH, low LH, and high prolactine?
Prolctinoma Is the diagnosis, treated by Bromocriptine (paroledl)

157. Male with infertility, Low Testosterone. Low FSH, LH and high prolactin.
What is the treatment?
Bromocriptine

158. Infertility and cysts in ovary what is the problem?


A. Ovulation.
B. Endometrial.
C. Cervix.

159. couples with infertility ask about surrogate pregnancy u are not sure wither
allowed in Saudi or not
A. Refer them to infertility clinic
B. Warn them might not allowed
C. Give appt after few wks and review bout the topic

160. couple ask about serogacy :


A-inform them it is illegal in Saudi arabia

161. First trimester miscarriage at 5 week with history of 2nd trimester abortion
twice duo to cervical incompetence, cause now:
Cervical incompetence->2nd
Chromosomal abnormality ->1st ->11week

162. Salpingiotomy done for tubal pregnancy. After 6 weeks B-hcg high, What's
to do?
If symptomatic - laparoscopic salpingectomy
If Asymptotic - methotrexate

163. Case about 21 years old primigravida with 2 years history of infertility now
came with spotting of blood (threatened abortion case). Nex step?

213
Glory 2019 ..

A. Confirm pregnancy by BhcG


B. Pelvic US
Ask about cause of infertility
N.B: Depend on real sentence in exam if not mention that she confirm pregnancy
before chose A
If mention that did urine test and was positive go for B

164. Woman come to clinic with history of multiple abortions, now she is 11
week pregnant, what is the best predictor of complication in pregnancy?
A. Abdominal pain
B. Vaginal bleeding
C. Other options

165. Case about pregnant in 13 weeks with vaginal bleeding. US picture given of
a uterus and a sac contains something. Closed internal os. What is the type of
abortion?
A. Unembryonic sac
B. Missed
C. Threatened

166. Pt c/o of bleeding and she at 8wk gestational age on ex os closed and there
is brownish color appear during pv, the abdomen is soft, not tender or
guarding and she denies passage of any tissue By u/s you found gestational
age 7wks what is diagnosis
A. Molar pregnancy
B. Threatened abortion
C. Ectopic pregnancy
N.B: Threatened abortion Products of conception intact, intrauterine bleeding, no
dilation of cervix,

214
Glory 2019 ..

167. 1st trimester , os closed , mild spotting. Type


is?
Threatened abortion. (tx is bed rest)

168. 1st trimester, Os closed and speculum


revealed heavy bleeding. Type of abortion?
Threatened abortion.

169. Pregnant came to ER in her 18 weeks,


comes with bleeding, cervix is open and
some products passed out?
A. Threatened
B. Missed
C. Incomplete

170. Pt pregnant come with severe bleeding


and component in cervix what next
management:
A. IV fluid and D and C
B. Expectant management
C. Explanation: this is a case of incomplete miscarriage.

171. Pregnant at 20
week complain of
vaginal bleeding os
closed ultra sound no
fetus
Complete abortion

172. Pregnant with


bleeding os open ...
Inevitable

215
Glory 2019 ..

173. jPregnant female 10 weeks came with vaginal blew + abdominal pain a
examination gestational age 11 -12 weeks , what is the cause ?
Molar pregnant
Incomplete abortion
Bc fundal height is more
than the actual ges age

174. Snow storm on


US?
Complete mole.

“A hydatidiform mole
presents with pain,
bleeding, and the
passage of vesicles. Ultrasound reveals a
heterogeneous uterine cystic mass with a
"snowstorm" appearance. B- hcg levels are
markedly elevated”

175. Questions about hyperemesis and


high BHCG (80 thousand) what is your
diagnosis?
A.partialmolar pregnancy

*clinical scenario about molar pregnancy, asking which type

216
Glory 2019 ..

176. Pt hadretained placenta and has


bleeding what type of PPH ?
A. primary√
B. secondary

177. SVD with 800ml blood loss visible to


eye?
A. Prim PPH
B. SEC PPH

178. Case of PPH. What to give?


A. oxytocin.(First drug to be given).√
B. ergot
C. carboprost

179. Pt female has history of recurrent PPH.


What to do ?
Active management of 3rd stage of labor.√

180. Pt 34 years developed PPH S.V.D they give Oxytocin and do massage but
not stopped what you will do?
A-Hysterectomy.
B-Ligation of uterine artery.
Treatment of PPH
Uterine massage
Nipple massage
-oxytocin ( increase contraction ) *para ventricular* and also vasopressin!!
IM 10 after 2-5 min
IV 20
IV. 40
-methergine 0.2 IM “every 2-4 hrs - contraindicated in HTN “
Carboprost! Prostaglandin f2 alpha 0.25 IM every 15 mins maximum is 8!!
“ Contraindicated in asthma “
Side effects = diarrhea
Mesoprastole = prostaglandin E1 per rectal = illegal abortion !! ( but different rout
sublingual )
Side effect = postparypyrxia
403 D
-Surgical
B linch stitch

217
Glory 2019 ..

Ligation ( uterine artery from anterior internal iliac artery )


Immobilization
Last hysterectomy

180. A delivery before how many weeks of gesta􀆟on called the Preterm birth
refers to a delivery?
37
40
34
28
“Preterm labor Is defined as contractions and cervical changes prior to 37 weeks
gestation. Pain associated with preterm labor is intermittent as the uterus relaxes
and softens between contractions.”

181. Preterm birth is highly suspected in symptoma􀆟c women if cervix length


by ultrasound scanning is less than which of the following?
20 mm
30 mm
10 mm
40 mm
“Cervical insufficiency refers to painless cervical dilation during the second trimester
that results in loss of pregnancy. The presence of lower abdominal pain (from
uterine contractions) excludes cervical insufficiency”

182. 22 weeks gestation , os dilated 1.5 cm. Dx?


A-cervical incompetence

183. Pregnant, 16 weeks, discovered to be RH –ve, what to do next?


A. Ultrasound
B. Amniocentesis
C. IV RH antibodies...

184. A 32 weeks gesta􀆟on, Para 2+0 woman, a􀆟ended the Emergency


Department complaining of lower abdominal and back pain that has increased
in its frequency and intensity over the last few hours. Abdominal examination
confirmed symphysis fundal height equal to 32 cm with longitudinal lie fetus
and cephalic presentation. Fetal heart was positive and cardio topography
was reactive with 2-3 uterine contractions per 10 minutes. Vaginal examination
confirmed 1 cm dilated cervix and 2 cm long, presenting part was cephalic at -
3 station. What would be best next action?
A. Inform neonatologist, give cor􀆟costeroids and strict bed rest
B. Inform neonatologist, give cor􀆟costeroids and hydrate the pa􀆟ent
C. Inform neonatologist, give tocoly􀆟cs and start intravenous an􀆟bio􀆟cs
D. Inform neonatologist, give intravenous an􀆟bio􀆟cs and strict bed rest

218
Glory 2019 ..

185. What is the more important thing to check before instrumental delivery?
A. Breach
B. placenta abruption
C. head pelvic disproportion

186. During-instrumental delivery, the doctor crushed the left (Ithink styelohioid)
what will be affected?
Left led closure,hearing loss,loss of taste anterior2/3, loss sensation of left sid

187. pt at 37 wks with sudden IUFD what to do


a. induction now
b. wait for spontaneous labour
c. let the pt to go home to make decion for next step

188. The mother refused the cs that will save the baby .....
a. Refer the pt
b. Repect her wish
c. Take the husband concent
d. Do vaginal delivery
e. Do cs against her wish

189. A 20 year old pregnant women refuses CS for complete placenta previa.
Fetus is full-term and healthy. Social worker spoke to mother (imp), next step
in management?
Doctor can go to the court and get permission for CS for the sake of the baby.

190. 24 week preterm delivered , mother she is a doctor and she asked not to
Resuscitate her baby , what you will do ?
Ignore the mother and Resuscitate

191. Pt in labor, dilatation 5cm since 2 hour , effacement 100 , station 0. Management?
A-observe√
B-give oxytocin

192. Pregnant lady with 6cm and 80 effacement , ruptured members and head
station+1, what is the labor stage?

219
Glory 2019 ..

2nd

193. 42 weeks + 3 days pregnant women


bishop score 8 CTG reactive, No CPD how
to manage?
A.expectant
B. CS
C. IOL
D. decide after one week
N.B: Bioshop above 4 = Go for induction
N.B: If she was 39 or 40 weeks the answer will be CS

194. protein +2, ketonuria, glucose high, pregnant 37 wks + 8 days, management ?
A. induction of labor
B. CS

195. Pregnant women in 38 weeks during her C.S surgeon noted bleeding came
from the upper abdomen, what the source of bleeding?
A. liver hemangioma
B. mesentericaneurysm
C. aorticaneurysm
D. perforated peptic ulcer
N.B: jaundice = CBD stone.

196. a pregnant lady was provided a pudendal nerve block as an analgesia,


which structure would be fully sensitive and not blocked by the analgesia
Rectum

197. Most common labor complication in newborn?


a. clavicle fracture...
b. shoulder dislocation
c. femur fracture

198. case female pregnant Still born at 38 wks every thing normal then
discharge but after 3 days come back with bleeding from every site injection
Which best investigation ? DIC
a. Hemoglobin electrophoresis
b. Fibrin product

220
Glory 2019 ..

199. Bleeding after cut of umbilical cord what's the cause?


Factor 13 deficiency

200. Baby brought by his mother after 7 days from delivery his weight was 3.5
and now 3.1 , why?
Normal

201. Pregnant women during vaginal delivery, what can make her has 4th
degree perineal tear ?
A. unrestrained legs and squatting position
B. unrestrained legs and sitting on chair
C. restrained legs and use of forceps and other metallic instrument

202. after delivery by 2 months, female came with urine leak from vagina during
urination. Dx?
urethrovaginal fistula
vesicovaginal fistula

203. case of a female came after 3 months of delivery, saying she used to suffer
with low mood , irritability during the first 2 weeks of delivery but symptoms
resolved now completely….what condition did she suffer from…?
a. postpartum psychosis
b. pp blues
c. pp depression
d. pp hallucinations

204. Female after delivered present with unable to breast fed her baby and no
milk at alL wtS diagnosis?
Sheehan syndrome

205. Case of Pregnancy ,did D&C , after 1 years got amenorrhea , what the
diagnosis ?
Asherman syndrome

206. PTs with Post delivery bleeding and she have asthma , what’s CI in her
case to stop bleeding?
a. Oxytocin
b. Misprostol( prostaglandins)
c. carboprost
Hemabate( carboprost) is PGF2 Alpha which can cause or aggravate bronchospasm
Note:Ergometrine is absolute contraindicated in HTN
Note: Ergometrine is relative contraindicated in asthma
Note: Carboprost (hemabate) is contraindicated in asthma

207. A woman presented with mastitis after recently giving birth to a boy, what’s
your next step in management?

221
Glory 2019 ..

a. Clean nipple with alcohol


b. Stop breast feeding
c. Surgical drainage
d. using the other breast and starting antibiotics

208. Preterm baby deliverd with difficult breathing .diagnosis?


A. Muconium aspiration
B. Respiratory distress syndrome
C. Transient tachypnea of newborn
N.B: Full term infant + CXR hyperinflation and fluids in fissures = TTN.
N.B: Preterm infant + problems during pregnancy/delivery such as asphyxia or fetal
distress or LBW or multiple births + CXR reticulogranular ''ground glass''
appearance, atelactesis, loss of heart borders and air bronchogram + symptoms
appear 2-6 hours after birth = RDS.
N.B: Postterm infant + CXR pneumonia and increased AP diameter and
hyperinflation and pneumothorax = MAS.

209. Mother came to clinic with her infant need to council about colicky pain:
A. reassure the mother 80% of infant with same complain
B. colicky pain with flatus in abdomen
C. reassure pain relief with the 6 weeks
N.B: Infantile colick = infantile spasm = infantile dyschezia = Crying for more than
three hours a day,, fore more than 3 days a week,, for three weeks.
*It will resolve by time, most of them in first 2-3-6 months of life with no long term
effects.

210. Case of pregnant women with UTI, culture was done and result was E.coli
sensitive for TMP/SMX and ceprofloxacin and nitrofurantoin, and asking what
is the best drug to give?
A. Cipro
B. TMP/SMX
C. Nitrofurantoin

211. female at 38 week with UTI what is ttt?


A. nitrufurtonin
B. cephalxin

212. acute urinary urge incontance- UTI Medical treatment with antichilonergic +
physiotherapy

213. 25 year old woman was seen in the gynaeological outpatient clinic with
excessive nad offensive vaginal discharge. What organism is the most likely
cause of her vaginal discharge?
trichomonas vaginalis

222
Glory 2019 ..

214. pt with vaginal itching and bad smell the microscopic is flatellaprazite
A. trichominosis
B. BV

215. Female come with grayish vaginal discharge + PH Dx, bacterial vaginosis
what is ttt ?
Oral metronidazole

trichomonas vaginalis

216. Yellowish greenish vaginal discharge increase with intercourse Dx?


A. trichomonasvaginitis.√
B. bacterial vaginosis.

217. TTT of trichomonas vaginalis (greenish vaginal discharge, flagellated


organism).
Metronidazole

218. pregnant lady complain foul smelling vaginal diacharg what u wll give?
A. Amoxicillin
B. cephalexin
C. Metronidazole.

219. Women e vaginal discharge offensive fishy odour revealed clue test d.d
A-Bacterial vaginosis
B-Trichomonas vaginosis

220. Female with vaginal discharge + nucleic acid amplification test (+ve) , Dx?
Gonorrhea

221. Fishy smell


bacterial vaginsois

222. Gray white discharge....


trichomanias

223. Case of Trichmonisis what’s the dx-


and what’s the treatment (metro)

224. Patients with sx of BV: What’s the


test:
gram stain

223
Glory 2019 ..

225. Pregnant women not complaining of anything, her urine analysis shows
nitrate, leucocytes, then culture revealed 50.000 organisms what is the dx?
A.cystitis
B.pyelonephritis
asymptomatic bacteriuria
N.B She must be treated.

226. Treatment of chlymedia


A- azithromycin
doxy cyclin
Azithromycin cuz it takes 1g once daily rather than doxycycline who taken twice daily

227. Multipara 7 with incontenance and vaginal pouch, what is dx?


Prolapse

228. Cervical incompetence


1. it is the most common cause of 2nd trimester miscarriage
2. ttt by cerclage 12-14 weeks and removed at 36 weeks

224
Glory 2019 ..

229. most common organism do tubal block?


Chlamydia

230. contraindication to EVC


Bicornate uterus

231. usual screening by mammogram every how many years? mammogram


screening
Starting from the age of 40
40-54 : annually
55 and older : every 2 years
^ American Cancer Society -Guidelines

232. Female came to routine visit and investigation , found high TSH + normal
T3 T4 what the diagnosis ?
a. Primary Hyperthyroidism
b. Primary hypothyroidism
c. Secondary hyperthyroidism
d. Subclinical hypothyroidism

233. Female came to clinic with hx of post coital bleeding , and you do
examination with speculum you found mass in cervix ( picture of mass ) what
the approach for her ?

225
Glory 2019 ..

a. Excesional the mass


b. Take biopsy
c. Check the Ca 125
d. Reassurance

234. Lady 22yrs with family HX of breast ca came to you with Right lower lateral
mass worsen ,with menstertion , you do biopsy found hyperplasia papillary
adenoma no pain , what you do ?
a. Tell the pt to follow up and came after 6 month with mammography
b. Excesional mass
c. Partial mastectomy
d. Total mastectomy

235. Pregnant lady and her kid started to go to shcoold, she is afraid that he will
affect her what vaccine she should take?
Influenza
236. Picture of anus with Cauliflower, for lady who do multiples unprotected sex
condylomata.

237. Female with fever , subrapupbic pain with vaginal discharge..


a. acute cervicitis.
b. acut salpingaitis
238. Female with fever and Supra public pain
with vaginal discharge ?
A. Acute Cervictiitis
B. Acute Salpingitis
C. Vaginal problem
D. Vulva problem
239. pregnant lady have Nuchal translucency
+ve > what complication the fetus will have ?
cardiac anomaly

240. female not controlled pass of urine during


cough or laugh ‫ﻛﺎن ﻓي ﺳﻧﺎرﯾو‬and has suprapubic
pulge
vasicocele

226
Glory 2019 ..

241. Couple come by infertility after investigation done show bilateral fallopian
tube obstruction what will do
A. Tell wife
B. Tell husband
C. Tell couple

242. Pregnant fear that her son may transfer an infection from his school Which
vaccine to give?
nfluenza

243. Preganat c/o sever abd pain and uterine cotractions she was given 6mg Mg
sulfate and the e contractions become normal then decrease the dose to 4mg
Then complained of shortness of breath What to do?
A. give her Ca gluconate
B. Stop Mg sulfate
C. measure serum Mg sulfate
D. fluid
** after stop Mg sulfate give the Ca gluconate

244. pregnant during ANC do 50gm glucose test and find high glucose level She
is more laible to which type if infection?
A. candida
B. trichomonus
C. bacterial vaginitis
D. atrophic vaginintis

245. pregnant c/o genital wart What to do?


A. cryo..
B. electro..
C. injection by......
**Cryotherapy (cryosurgery) destroys genital warts by freezing them with liquid nitrogen
and it’s safe for pregnancy

227
Glory 2019 ..

246. pregnant in early 2nd trimester c/o painful cx bleeding what to do?
A. pap smear
B. coloposcopy
C. biopsy
D. transvaginal US
** Threatend abortion, Transvaginal US

247. CTG shows cord compression?


Variable declaration

248. vaginal discharge, flagellate?


trichomonas

249. Trichomonas, and ask about ttt?


Metronidazole

250. fishy odor , dx?


Bact vaginosis

251. valval itchy for 1 year, dx?

252. post menopause Bleeding ?


cancer
253. tender uterus ?
adenomysis

254. female in reproductive age , come by an ambulance (RTA ) What 1st thing u
have to do?
check for pregnancy

255. Atrophic vaginitis diagnosis.


Clinically
** definition: dryness of the vagina that occur in postmenopausal women due to low
estrogen production

228
Glory 2019 ..

Features: vaginal spotting (mild bleeding every 4-5 days), vaginal soreness, burning,
dryness, redness, thin vagina with few or non fold, pH alkalosis > 5, periodic petechiae
Treatment: topical vaginal estrogen

256. Ectopic pregnancy 1- 3 cm and Bhcg 2600


medical treatment (methotrexate)

257. Ectopic pregnancy with peritonitis what is the the


next step
A. Laparotomy
B. Medical treatment

258. Patient with severe pre-eclampsia with proteinuria , low platelets ,


abdominal pain and high URAC acid What of the following indicate the poorer
out come :
A. high proteins
B. low playlets
C. Abdominal pain
D. High uric acid
259. patient with signs and symptoms of
polycystic ovary syndrome and asking about
diagnosis : but they mentioned the other
name which is Stein–Leventhal syndrome

260. what is the action of metformin in


treatment of polycystic ovarian syndrome:
reduce insulin resistance

261. The first stage breast milk ( colostrum) :


Has high percentage of protein
262. Pap smear :
Every 3 years
263. 55 years old with normal mammography when to repeat after :
A. 1 year
B. 2 years
C. 3 years

229
Glory 2019 ..

264. 28 years old female with normal pap smear 3 years old what to do :
A. Nothing
B. Pap smear with cytology

265. When does ovulation happen :


A. peak of LH
B. Peak of FSH
C. Peak of progesterone
D. 21 days after peak of deoxyprogetrone

266. What is the prominent estrogen during pregnancy:


A. Estradiol
B. Estriol
C. Estetrol
**estriol in pregnancy
Estrone in postmenopause
Estradiol in childbearing age

267. Patient with nervousness irritability and severe premenstrual symptoms


what to give :
A. intradermal progestogen patch
B. SSRIs
C. Progesterone only pills
D. Other contraceptive methods other than the OCP

268. What is the best predictor of glycemic control of prevent women :


A. OGTT
B. Hb A1c
** If they are asking about perdictor of glycemic control in general it would be B, If for
pregnant women and they’re referring to the screening test should b A

230
Glory 2019 ..

269. Quad test for Down syndrome in pregnancy:


High Bhcg , low estradiol, low alfa fetoprotein , high inhibin A

270. A 32-year-old woman sees her physician because she has noticed milk- like
discharge from her breasts the past 4 weeks. She also states that she has not
menstruated in 2 months. The examination reveals galactorrhea but is
otherwise normal.
Here our workup will be; initially measuring serum Prolactin, if high so next step is MRI
Pituitary to confirm diagnosis
** Always exclude states such as pregnancy, lactation, hypothyroidism and medications
before starting the workup of hyperprolactinemia. Prolactinomas may co-secrete growth
hormone (GH).

271. Many q about valvular lesion

272. Pregnant lady in week 13 GA, came with bleeding and suspected she have
an infection what test you do?
A. Pap smear
B. Biopsy
C. Colposcopy
D. invasive

231
Glory 2019 ..

Section 4
Surgery …..

232
Glory 2019 ..

1. .29 Years old male did sleeve gastrectomy 6 days ago, came to the ER
complaining of persistent vomiting since operation, on exam nothing
significant, ABG normal ( there were numbers ) , US normal. Best
management?
A.laparoscopy
B.Admissions
C.laparotomy
D.reassure

N.B: Dumping syndrome: postprandial symptoms like epigastric fullness, pain, vomiting.
It's treatment is only small multiple low charbohydrate, low fat and high protein meals +
avoiding liquids with meals.
N.B: If there is persisting vomiting for several months aftergastric surgery so think of
narrowing or stricture and next is gastroscopy.

2. 21 year old male complains of pain around umbilicus then traveled to rt iliac
quadrant tenderness ,reboundtenderness,positiverovsing sign Vomiting,
constipation. What is the DX?
A.polyposis
B.Appendicitis
C.intestinal obstruction
D.intussusception

N.B: Most characteristic feature of appendicitis is beginning of the pain around


umbilicus then its migration to right iliac fossa.

3.Someone has gallbladder stones came complaining of yellowish discoloration


and epigastric pain , his labs ALT, AT , ALP , bilirubin , amylase are high. What
are next step ?
A.US
B.CT
C.ERCP

N.B: Pt is a known case of gallstones so no need to do US.


N.B: The case is CBD stone, next step is ERCP cuz both diagnostic and therapeutic.

233
Glory 2019 ..

4.Case Scenario for pt with Mitral Valve prolapse, the question is WOTF is true
regarding the surgery ?
Don’t give the Ab before surgery

5.TOF

6.Man with blunt trauma , what site of aorta is injured ?


A- aorticarch
B- distal part of aorta in subclavian site
C- proximal part of aorta

7.Man in ER with pain in the right upper abdomen with vomiting ultrasound
shows thick wall of gallbladder and fluid accumulation around management
A - lap cholecystectomy after 12 weeks
B - percutaneous drain
C - open cholecystectomy

N.B: it is acute calcular Cholecystitis.


N.B: RUQ pain = Biliary colick.
N.B: RUQ pain + Fever = Acute cholecystitis. N.B: RUQ pain + jaundice = CBD stone.
N.B: RUQ pain + Fever + jaundice = Acute cholangitis.

8.Appendicitis without pus management


A- lap appendectomy after 12 weeks
B- open appendectomy after 12 weeks
C - immediate appendectomy

N.B:Acute appendicitis = immediate open appendectomy


N.B:Appendicular abscess = ABx and percutaneous drainage
N.B:Appendicular mass = conservative Oshler scheren regimen

9.Symptom confirm diagnosis of eosinophilic esophagitis


A- dysphagia
B stucking of food in esophagus
C - chest pain centrally located not respond to antacid
D - confirmation by biopsy and endoscopy

If asking about what clinical feature is highly related to EoE so C


If about how to confirm Dx, so D-

10.Q about 2 nodules in thyroid management

234
Glory 2019 ..

11.Patient with fever right upper abdominal pain +ve serology of echinococcus (
hydatid disease) **picture management
A- albendazole
B- cyst
C- Laparoscopiccystectomy
D - percutaneous drainage

12. Pt with appendectomy what is the most common infection post op?
A- fragilisbacterial
B- staph
C- shigella
D- pseudomonas

The organisms most frequently isolated from the wound after appendectomy are
Bacteroides (especially B. fragilis)

13. pt will do thyroidectomy suffer from MR give?


A- cefazoline for wound infection
B.Amoxicillin for endocarditis
C- amoxicillin & cefazoline together
D- Give nothing

N.B: Thyroid surgery is a clean procedure so no need for prophylactic ABx N.B: Valvular
disease is not indicated for endocarditits ABx prophylaxis.

14.pt with constipation for 2 days suffer from sever anal pain ex at 6 & 12 o’clock
A- not laceration
B- internal sphincter abscess
C- anal fissure
D- fistula

N.B: Most common site of anal fissure is 6 (posterior) then 12 o'clock N.B: TTT of anal
fissure by lateral internal sphicterotomy

15. 40 yr parathyroid adenoma, when we do parathyroidectomy


A- age > 50
B- osteoporosis

16. IV contrast , precaution?


A-Avoid Analgesia and Metformin and Diuretic (before )
B-Give fluid Nacl (before and after contrast)

Both are correct according to which is given in choices.

235
Glory 2019 ..

17.swelling med line under thyroid bone?


motile thyroglossal cyst

18. Neck mass ; biopsied showing (thyroid folicullar cells) ?


ectopic thyroid

19.thyroid mass cytology -> medullary thyroid carcinoma, rt lobe ‫ﯿﻢ‬


A- total
B- subtotal
C-radiotherapy
D-chemotherapy

total thyroidectomy to avoid the recurrence in other lobe

20.lesion in upper thigh ( irregular margin )


A- excision biopsy
B- punch biopsy

21. laparoscopic cholecystectomy come with discharge from middle management


A.daily dressing
B- wound inspect
C- exploration

22.tension pneumothorax, best ttt ?


A- thoracocnthesis
B- tube thoracotomy

N.B: Initial step in tension pneumothorax = needle decompression


N.B: Best step in tension pneumothorax = Tube insertion

23.pic of open fracture -> debridement and surgery

24.hematemesis ->endoscope ->resuscitation Before endoscope what you’ll give


A- Octoreotide

25- Mallory weisssyndrome


-No hx of peptic ulcer rule out the erosive gastritis

26.Anal itching and pain after defecation ?


A- anal fistula

236
Glory 2019 ..

B- anal fissure.

N.B: features of anal fissure are: visible tear around anus, skin tag or lump near tear,
sharp anal pain related to defecation, streaks of blood on stools, burning or itching in
anal area.

27.Inguinal hernia reducable, TTT?


A-simple repair
B-Elective open hernioplasty with mesh repair.
C-laprascopic
D-observe

N.B: Best for hernia is open surgery except: female for cosmetic reason + bilateral +
obese so here it is better for lap surgery.

28. Hemorrhoidectomy is a treatment for?


A-interalpiles.
B-external lateral hemorrhoids.
C-fissure.

29-Pt has multiple multiple liver lacertion what to do ?


A-prehepatic packing
B-rthepatectomy
C-vesselsligation

30-Pt had rt appeindeceal abscess what pathophysiology ?


A-Avasoconstriction
B-redistribution of blood vessels

31-Pt had chest trauma nad 3⁄4 rib fracture , paradoxical chest movement ?
A-chest tube
B-needle aspiration

32- charcoats triad ?


A-acute cholangitis
B-acute cholecystitis
C-acute appendicitis

33. duodenal perforation treatment


A-simple closure with omental patch “Graham omental patch”

34-anal fissure treatment 2 mcqs related to this ?

237
Glory 2019 ..

A- Lateral internal sphinterectomy


B-externalsphincterectomy

35- pt with history of Ocp use wat is associated with it?


A anemia
B history of fatigue
C history of DVT

36-third and fourth degree perineal tears due to ?


A unrestrained legs and squatting position
B unrestrained legs and sitting on chair
C restrained legs and use of forceps and other metallic instruments

37-a pic of ultrasound showing chocolate cyst (endometrioma) ?


A associated with increased risk of infertility

38-multiple draining sinuses ?


A carbuncle

39-a pt rescued from a burning building ....he is conscious with singed nasal
hairs
A- intubation
B- respiratory support

40. contraindication of liver transplant


A acute hepatitis
B chronic
C liver cirrhosis
D end stage liver disease

41-Patient admitted after surgery then developed inferior MI what management ?


A- thromolytic
B- angioplasty
C- heparin

-Thrombolytic is contraindicated post surgery

42-Trauma to knee can’t fix tibia to femur with severe pain and swelling which
ligament affected tibial lig
A-ACL
B- PCL

238
Glory 2019 ..

43-pt with fever and upper right pain with positive echinococus
A- Albendazol
B- Cystderoofing
C- Laprascopiccystectomy
D- Perctunouse drain

44-man inthe ER with pain in right lower quadrant of abdomen, nausea and
vomiting. U/S show thick wall gall bladder with fluid surrounding. Management?
A.Lap cholecystectomy after 12 weeks
B.Percutaneous drain
C.Open cholecystectomy

45-Man with appendicular mass without pus, what is the management?


A. lap appendectomy after 12 wks
B.open appendectomy after 12 wks
C.immediately appendectomy

Ttx observation and Ochsner-Sherren regimen first

46- Man with HTN, labs show normal adrenal gland, but CT shows mass with lipid
change. Management?
A- adrenalectomy
B- MRI adrenal
C- observation

47-Alcoholic man, sever epigastric pain, radiated to back, abdomen is


tenderness, amylase is very high, BP 90/60, tachycardia, next?
A.IV fluid
B.Analgesia
C.Antibiotic
D.Surgical operation

48-Before adrenal gland operation, what should you give the patient?
Hydrocortisone

49-case of long bone fracture ttt

Analgesia and fluids Tetanus if open Xray


Reduction Cast or fixation

50-ttt of pseudocysts 4/6cm


51-Pancreatic pseudocyst ttt :

239
Glory 2019 ..

-less than 6 cm and 6 week> observation


-more than 6 cm and 6 week> drainage:
A- if there is sign of infection as fever and leukocytosis do external drainage
B- if sterile do internal drainage

53-Asd

54-carpal tunnalsyndromttt after operation


A.opiate
B.amytriptilian
C.NSAID

55- Increase intracranial pressure.

56- Cases about post surgicalinfection , fever and complications?

57- Lead pipe picture << ulcerative colitis

58- Elective surgery to pt ,pt don’t want to know details but anesthesiologist
insist to tell her. What to do?
A. Tell pt anesthesiologist decision
B. Reschedule surgery until see another anesthesiologist

59- thyroid-Single nodule on one lobe of the thyroid. What is the surgical
treatment?
A. total thyroidectomy
B. subtotal
C. left lobe thyroidectomy

Depends
If toxic nodule , do Thyroid scan then : total if cold, hemi if hot
If non toxic nodule, do FNA then, if malignant( medullary ) so total

60.Multiple nodular in iodine uptake scan: almost same choices as above

61- epigastric pain, tenderness, pain radiated to back, x ray show air under
diaphragm, what the most likely diagnosis?
A. Perforated duodenal ulcer
B. Pancreatitis
C. Gastric ulcer

240
Glory 2019 ..

62-Patient fell from 50 feet exam decreased breath sounds on left side CXR
picture of pleural effusion on left side what is the treatment?
A- chest tube -hemothorax-
B- thoracotomy
& I dont remember.

63-pt did laparoscopic cholecystectomy and you found a large mass in the
stomach, what you will do :
A-cholycystectomy
B-stop the procedure
C-resection of mass
D- cholysytoctomy and resection of mass

64- After cholecystectomy pt developed pain at angle of mandible,temp 38 wha to


do
A-Paracetamol.
B-Abs
C-Xray.
D-CT

Explanation: case of anesthesia mumps , benign , just needs pain relief .

65- most place injured of blunt thoracic


A-aortic arch(distal to subclavian)

66- pt with one leg pain progress over month what you will ask in hx:
A-hx of truma
B-family hx of same complain

67- pt smoker with cavity in rt lobe


A-scc
B-small cell carcinoma

68- consultant at OR and ask to prepare right kidney for surgery but the intern
read in notes it is left kidney what is the most appropriate action by intern?
A- inform the consultant
B-leave the OR
C-go out to re read the note

69- pt with wt loss and loss of appetite , what the next:


A-upeer GI endoscopy
B-abd US
C -CT

241
Glory 2019 ..

Question not clear , lacks informations

70- pt with pruritus and jundice and abd pain he k/c of Ulcerative colitis what is
the dx?
A-primary biliary cholangitis
B-primary scleorsing cholangitis

71- (55 years old) female her last mammogram was normal when she will do the
next after?
A-1 year
B-2year
C-3-year
D-4 year

72- 23 y female with symptoms of IBS and she had concer because his uncle had
colon cancer at 60 y what dx?
A-IBS

73- old pt with DM and Ischemic heart disease and gas severe abd pain what dx?
A-mesentric ischemia
B-appendicitis

74-pt with rt knee pain he has epigastric burning what you will give ?
A-ibuprofen
B-paracetamol

Explanation : NSAIDs are contraindicated in cases of PUD.

75- pt came to the ER can’t pass stool since 2 days and has sever pain in exam
you can’t do PR bc of pain ,pt did hemorrhoidectomy 12 months ago what is the
cause ??
A-anal stenosis
B-anal fissure

Explanation: Anal stenosis is a rare but serious complication of anorectal surgery, most
commonly seen after hemorrhoidectomy

76- indiction for parathyroidoctomy ?


A-osteoporosis
B-age more than 50 y

77- pt post op or was with significant pnemounia what should add


Also has elevated INR

242
Glory 2019 ..

A-Vit K was one of the options

78- pt k/c of crohon presented with anal sinus


Which medication protect from that
NO choices
- Infliximab

79- absolute indication for parathyroidectomy


No choices
- Osteoporosis

80- Pt with with otitis external after swimming in pool which Abx to give?
Explanation
The most common cause of acute otitis external is pseudomonas bacteria infection,
fungal candidiasis by candida albicans is usually resulted from over treatment with
topical antibiotic, TTT of bacterial otitis external is by antipseudomonal like
Ciprofloxacine or Ceftazidime otic drops while TTT of fungal otitis external is by
repeated debridement and topical anti-fungal agents like Gentian violet or Mycostatin
powder)
^ Toronto

81- Pt with hx of trauma to abdomen came after several days with purulent
discharge what to do?
A- Ctabdomen.
B-Lapratomy

If stable >> CT
Unstable >>lapratomy

82- Pt with chest trauma asymptomatic CXR showed 2 cm pneumothorax what


will u do?
A. Aspiration needle then follow up chest X-ray

83- Pt with gun shout to chest, underwent chest tube which drained 2L of blood.
What is the next step?
A- Pleural tab.
B- Embolization.
C- Emergency Department Thoracotomy!!

84- Dx pt with painful progressing swelling with discharge at his neck what is the
dx?
A- Cellulitis.
B- Abscess.

243
Glory 2019 ..

85- Pancreatic pseudocyst treatment?


A- Internal drainage.
B- B- Percutaneous drainage.
Explanation:
Pancreatic pseudocyst is one of the complications acute pancreatitis, it is an
encapsulated fluid collection that occur 1-3 weeks following acute pancreatitis
Management
1. if size < 5-6 cm or less than 6 weeks = observation
2. if > 5-6 cm or more than 6 weeks = drainage is either internal drainage "if no
fever or high WBCs", or external percutaneous under radiological guidance "if
fever or high WBCs"
^ Toronto + Step up to medicine

86- Alcoholic patient with sever epigastric pain radiated to back, nausea and
vomiting, elevated lipase and amylase. And obstructive pattern of LFT. Dx?
A. Pancreatitis

87- Case of benign prostatic hyperplasia what is the management?


Alpha blocker , prazosin

88- Pt with upper breast cancer next step?


A- PET scan.

Explanation :for staging , breast cancer Mets ( Bone , lung ,liver, brain )

89- Case of painless freely mobile breast lump not related to menstruated
changes. What is the diagnosis?
A- Fibroadenosis.
B- Fibroadenoma.
C- Fat necrosis.
D- Breast cancer.

90- pt with hernia came with acute swelling , redness , tender , not reducible
Which complication of hernia is this
Iirdeuceable
Incarcerated
Strangulated

91- heaveysmoker Cxray pleural effution , trachea shifted


- Obstructing lung tumer
- Ruptured esophagus

92- adult with epigastria pain us showed several narrowing of biliary duct
LFT obstructive pattern what is the dx
Primary sclerosing cholangitis

244
Glory 2019 ..

93- pt presented with appendicitis ssx , us should obesity in the pelvis what to do
next
ct abdomen

94- mid age man with appendoceal after conservative ttt recovered and discharge
what to do next
- - no thing
- Laproscopic appendectomy in 12 wks
- Open appendectomy in 12 wks

95- pt after appendectomy stble with mild pain, should minmal collection
- Conservative mx
- - re open
- - aspirate

96- after thyroidectomy swelling and un able to breath


- Open wound in word
- Trachiotomy

97- after mesh repair of hernia pt with radiculopathy radiate to leg


- Open mesh stable
- Nerve block

98- case of bowel obstruction imaging showed y shape shadow and enlarge
colon what is thenext step ?
A- Enema
B - CT abdomen

- CT is the best to confirm the diagnosis

99- pt with stab wound to chest , SOB, I think low o2 saturation and ssx of
pneumothorax next step ?
- If dullness on percussion so hemo so chest tube
- If hyperresonant so tension pneumothorax so needle then tube

100-post brain operation Diabetes insipidus management ?


- DDAVP ( Desmopreson )

101- pituitary adenoma management ?


Trans sphenoidal adenectomy

245
Glory 2019 ..

102- what is the most common GIT malignant associated with peutz-
jegherssyndrome ?
A- stomach
B- pancreas
C- colon
D- liver

103- x-ray of a pneumothorax, pt after truauma developed difficulty in breathing,


asking best initial management?
a. needleaspiration
b. chesttubeinsertion
c. intubation
d. emergentsurgeryreferral

104- patient comes with neck swelling, moving with deglutition located hyoid
area, what’s?
a. Thyroglossalcyst
b. Thyroidcancer
c. CysticHygroma

105- 21 years old women presents to the gynaecology Clinic with a mass in the
left breast, she discovered this mass while showering, her last menstrual peroid
was 10 days ago, there’s no family history of breast cancer.
On physical examination, you palpate a 3 cm, rm, non tender mass in the upper
lateral quadrant of the left breast, the mass is smooth, well circumscribed and
mobile there’s no skin changes or nipple discharge or axillary lymphadenopathy.
Which of the following is the most likely diagnosis in this woman?
a. Fibroadenoma
b. Fibrocysticchange
c. Intraducalpapailoma
d. Lobularcarcinomainsitu

106- lady with a mass in left upper quadrant of the breast, aspiration was yellow
uid without masses, this mass completely disappears with aspiration, what’s the
diagnosis?
a. Phyllod
b. breastcancer
c. broadenoma
d. fibrocysticdisease

107-case of appendicular mass ttt


a. do open surgerya fter 3 month

246
Glory 2019 ..

b. do laparoscope surgery after 3 month


c. Noneedforsurgery

108-which of the following investigation should be done for early detection


of bilateral breast cancer?
A. FNA
B. biopsy
C. mammogram
D. tumor marker

109- A 37-year old women presented to the doctor with intermittent bloody
nipple discharge from the left breast. On examination there is no palpable
mass. Which of the following is the most likely diagnosis?
A. Fibroadenoma
B. Fibrocystic changes
C. Intraductal papilloma
D. Mastitis

110-A 32- year- old female presented to her physician with feel small lump
in her right breast , On Examination and investigation normal, what is the
best asdvise can tell her to do self-breast exam every?
A. 1 week
B. 2 month
C. 3 month
D. month

247
Glory 2019 ..

111- 36-year-old woman complains of a 3-month history of bloody


discharge from the nipple. At examination, a small nodule is found, deep to
the areola. Careful palpation of the nipple-areolar complex results in blood
arrears at the 3 O’clock position. Mammogram findings are normal. What is
the likeliest diagnosis?
A. Breast cyst
B. Carcinoma in situ
C. Intraductal carcinoma
D. Intraductal papilloma

112- patient lady noticed lump for three months the mass freely mobile no
discharge not related to menstrual, what is the diagnosis?
A. cystic
B. duct ectesia
C. ductal papilloma
D. Fibroadenoma

113- Which of the following is the best treatment for a lung abscess?
a. Ciprooxacin
b. Clindamycin
c. Macrolides
d. Piperacillin

114- Female work as an actress, present with mass that increases before
menses, after taking FNA the result is yellow stain with no refilling again
what is the diagnosis?
A. Fibrocystic breast disease
B. follicular cancer
C. intraductal papilloma
D. phyllodes tumor

115-Patient with bloody stained nipple discharge, you’re suspecting


intraductal papilloma. What’s the next step?
A. Excision
B. FNA
C. Mammogram
D. discharge culture
Explanation:
To confirm the Dx in histopathology

248
Glory 2019 ..

116- A 40-year-old lady presented with bloody nipple discharge. Breast


examination showed normal skin and no areolar fullness. There were no
palpable axillary lymph nodes. Which of the following is the most likely
diagnosis?
A. Breast abscess
B. Fibroadenoma
C. Fibrocystic disease
D. Galactocele
E. Intraductal papilloma

117- patient presents to the hospital with bloody nipple discharge, what is
the most likely cause?
A. Intraductal papilloma
B. ductal
C. lobular carcinoma in situ
D. papillary

118- After intraductal papilloma, unilateral bloody nipple discharge from


one duct orifice is most commonly caused by which of the following
pathologic conditions ?
A. Infammatory carcinoma
B. Intraductal carcinoma
C. Paget's disease of the nipple
D. Subareolar mastitis

119- patient noticed lump for three the mass freely mobile, no discharge
not related to menstrual , what is the diagnosis ?
A. cystic
B. duct ectesia
C. ductal papilloma
D. Fibroadenoma

120- t e thoraccentesis site of needle


A-4th
B-5th
C-6and7
D-7and8

Explanation:
Needle :2
Chest tube : 5

249
Glory 2019 ..

Liver biopsy :7
Thoracosentesis : 8-10

121- After cholecystectomy pt developed pain at angle of mandible,temp38 wh to


do
A-Paracetamol
B-Abs
C-Xray
D-CT

122- Pt developed appendicular abscess drainage was done wh is the


pathophysiology
A-Redistribution of bld supply
B-Cardiac index
C-Bradycardia
D-Peripheral vaso constriction

123- Female mass at lt upper breast,painless,movable not related to menstrual


cycles wh is the d.d
A-Fibrocystic
B-Fibroadenoma
C-Ductalpapilloma

124- Patient with breast cancer, she is having bleeding from breast with redness
,on routine check 2x3 cm then became 3x6cm.What do you suspect?
A. bad prognosis
B. inammatory process
C. malignant cancerous change
D. metastasis precess

125- In patient post mastectomy, they do for her reconstruction from the rectus
muscle. what is the vessels maybe injured or effected?
A. Inferior epigastric artery
B. Superior epigastric artery
C. intercostal artery
D. internal thoracic artery

250
Glory 2019 ..

126-A 70 year old patient is complaining of face 􀆟ushing and head heaviness in
the morning
that's relieved during the day. He has been smoking for 30 year.s CXR shows
bilateral hilar masses. What is the diagnosis?
A. COPD
B. Hodgkin's lymphoma
C. Lung cancer
D. TB

127- A male patient who is a smoker, developed haemoptysis and shortness of


breath on exertion. Investigations shows a high serum calcium and a solitary
central lung nodule. What is the most likely diagnosis?
A. Adenocarcinoma
B. Hamartoma
C. Small cell cancer
D. Squamous Cell Carcinoma

128-What type of cancer is most commonly found in the distal esophagus?


A. Adenocarcinoma
B. Metastatic cancer
C. Small cell carcinoma
D. Squamous cell carcinoma

129-What is the most common type of cancer in the middle of the


oesophagus?
Scc
Upper 2/3 > SCC
Lower 1/3 > adenocarcinoma

130-30 years old female dancer with breast mass that disappeared by
aspiration, What is your suspect?
A. cystic –fibroadenosis-
B. duct ectesia
C. Fibroadenoma
D. phylloid

131-which of the following investigation should be done for early detection


of bilateral breast cancer?
A. FNA
B. biopsy
C. mammogram
D. tumor marker

251
Glory 2019 ..

132-A 37-year old women presented to the doctor with intermittent bloody
nipple discharge from the left breast. On examination there is no palpable
mass. Which of the following is the most likely diagnosis?
A. Fibroadenoma
B. Fibrocystic changes
C. Intraductal papilloma
D. Mastitis

133-A 32- year- old female presented to her physician with feel small lump
in her right breast , On Examination and investigation normal, what is the
best asdvise can tell her to do self-breast exam every?
A. 1 week
B. 2 month
C. 3 month
D. month

134-36-year-old woman complains of a 3-month history of bloody


discharge from the nipple. At examination, a small nodule is found, deep to
the areola. Careful palpation of the nipple-areolar complex results in blood
arrears at the 3 O’clock position. Mammogram findings are normal. What is
the likeliest diagnosis?
A. Breast cyst
B. Carcinoma in situ
C. Intraductal carcinoma
D. Intraductal papilloma

135-patient lady noticed lump for three months the mass freely mobile no
discharge not related to menstrual, what is the diagnosis?
A. cystic
B. duct ectesia
C. ductal papilloma
D. Fibroadenoma

For 6 weeks

136-Female work as an actress, present with mass that increases before


menses, after taking FNA the result is yellow stain with no refilling again
what is the diagnosis?
A. Fibrocystic breast disease
B. follicular cancer
C. intraductal papilloma
D. phyllodes tumor

252
Glory 2019 ..

137-Patient with bloody stained nipple discharge, you’re suspecting


intraductal papilloma. What’s the next step?
A. Excision
B. FNA
C. Mammogram
D. discharge culture

Confirmation by histo pathology


excisional biopsy of affected duct
must rule out intraductal papillary carcinoma

138-A 40-year-old lady presented with bloody nipple discharge. Breast


examination showed normal skin and no areolar fullness. There were no
palpable axillary lymph nodes. Which of the following is the most likely
diagnosis?
A. Breast abscess
B. Fibroadenoma
C. Fibrocystic disease
D. Galactocele
E. Intraductal papilloma

139-After intraductal papilloma, unilateral bloody nipple discharge from


one duct orifice is most commonly caused by which of the following
pathologic conditions ?
A. Infammatory carcinoma
B. Intraductal carcinoma
C. Paget's disease of the nipple
D. Subareolar mastitis

140-40- t e thoraccentesis site of needle


A-4th
B-5th
C-6and7
D-7and8

Needle :2
Chest tube : 5
Liver biopsy :7
Thoracosentesis : 8-10

253
Glory 2019 ..

141- After cholecystectomy pt developed pain at angle of mandible,temp38 wh to


do
A-Paracetamol
B-Abs
C-Xray
D-CT
Anesthesia mumps

142-Pt developed appendicular abscess drainage was done wh is the


pathophysiology
A-Redistribution of bld supply
B-Cardiac index
C-Bradycardia
D-Peripheral vaso constriction

143- Unmarried young lady , 20 yr old, with bilateral abdominal pain, she is a
waitress at a resort, very vague sexual history, no bleeding, no significant and
examination….no fever, missed last month periods….diagnosis.
a. ovarian torsion
b. salphingitis
c. ruptured ectopic pregnancy
d. endometritis and adnexal masses

Bilateral = generalized = rupture ectopic causes perotinitis

144- indirect inguinal hernia mechanism ??

145- overall deaths percentage due to post part haemorrhage.


a. 15%
b. 25%
c. 20%
d. 10%

254
Glory 2019 ..

146- 58 yr old woman, h/o epigastric pain, for 2 months, no rebound tenderness,
no abdominal mass, no bruit heard over abdominal area, history of mild
discomfort over chest since 6 months ECG of ST elevation in 2, 3, AvF given….
a. MI > inferior
b. peptic ulcer disease
c. abdominal aortic aneurysm
d. coarctation of aorta.

147- pt with thyroid function test… TSH 400 T3 normal T4 normal, diagnosis…
a. sub clinical thyroiditis
b. primary hyperthyroidism
c. primary hypothyroidism
d. goitrogenic cancer

148- pt after colectomy due to colorectal ca follow up?


A. 3month
B. 6month
C. annually
D. 2yrs

149- pt with scaling skin lesion in the extensor surface of the forearm ( with
picture), your advice:
a- avoid trauma.
b- avoid sunlight
c. apply sunscreen
d. green tinged make up

150- 55year old patient with dysphagia for solids with a previous history of reflux
A. Carcinoma oesophagus
B. Stricture
C. Scleroderma
D. Achalasia

255
Glory 2019 ..

151- x ray of pneumothorax, pt after trauma developed difficulty in breathing,


asking best initial management
a. needle aspiration
b. chest tube insertion
c. intubation
d. emergent surgery referral

152- Patient came for assessment after colle’s fracture by falling on outstretched
arm with minimal trauma, what is the appropriate test to check for bone density?
A- VIT D
B- Calcium level
C-X RAY hip and pelvis
D-Dual energy x ray absorptiometry

256
Glory 2019 ..

153- Patient complain of light-headedness, tachycardia, diarrhea, relieve by lying


down, history of gastrointestinal surgery before 2 month, what is your provisional
diagnosis
a) IDS
b) Dumping syndrome
c) Villous adenoma
d) Crohn’s disease

154- about hernia advise in asymptomatic


surgery is required

True except if the pt younger than 5 y/o

156- Patient comes with neck swelling, moving with deglutition located hyoid
area. What is diagnosis?
a) multi nodular goiter
b) Thyroglossal cyst
c) Thyroid cancer
d) Cystic Hygroma

157- pt with enlarged parotid gland with secretions increased over time, best
initial investigation
A. USG
B. CT
C. X-ray
D. MRI
Best initial is US
Confirm is MRI

158-Q about Slipped capital femoral epiphysis Clear scenario and X ray
Obese In boys more than female txx:surgical

257
Glory 2019 ..

159-Which nerve carries the referred painof the parotid to the ear?
A. Vagus
B. Facial
C. Auriculotemporal
D. Trigeminal

Auriculotemporal innervates auricle and parotid gland. It mediates referred otalgia.

160-Q. Patient came to ER, complaining of hypotension, tachycardia and


hypercapnia, ECG showed arrhythmia, what is your immediate action?
A. Needle decompression
B. Pericardiocentesis
C. FAST ultrasound
D. Thoracotomy

161-ase scenario about a patient who had an injury in his right knee, the doctor
applied a valgus stress to examine which ligament
A- Anterior cruciate ligament
B- Posterior cruciate ligament
C- Fibular collateral
D- Tibial collateral

Tibial collateral = medial collateral

162-which true for breast screening?


Women (50-70) mammogram each 2 years (grade B recom)

258
Glory 2019 ..

163-Pt on ttt for hyperthyroidism came with [ neutrophils, monocyt , basophil ]


agranulocytosis NL RBC +PLT what is ttt?
Methimazole

164-Knee or leg pain decrease with passive stretch , which mnsd ?


Gastrocnemius

165-Soldier with pain at medial scle of foot with Hx of walking for long distance,
which ligament is affected?
Spring Ligament

166-Mutiorgan failure after trauma lab [ high phosphate] which organ can cause
this ?
Kidney

167-Can't elevate hand drop wrist?


Radial nerve injury (in spiral grove of humans)

168-Pt with head fracture what is the first step ?


Airway

169-17 year old runner (Athlete) when preue of knee cab kid pain Dx ?
osgood Schlatter

170-Bilatral inguinal swelling + painful in genitalia +cervical lymphadenopathy


Dx?
lymphogranuloma venereum

171-Neck pain radiated to shoulder


disk prolapse Cervical

172-Pt with dysphagia pain + enlargement in one tonsils, Dx?


Quinsy (peritonsillar abscess)

173-How to effect of intathecal anesthesia?


By tremblenburg position
if decreses effect anti trenderberg
if increse trenderberg

259
Glory 2019 ..

174-Witch of the following characteristics for cholecystitis ?


RVQ pain

175-Female with problem in chest go for mastectomy we will take from abdominal
muscle for
graft witch artery is response for it ?
A. superior epigastric
B. Inferior epigastric

176-Pt after surgery loss of sensation of upper lip


infraorbital nerve

177- Pt with hyperaldosteronism [normal renin -BP -aldosterone] what's the cause
in the adrenal cortex for this?
Glomerulosa.
Aldosterone, the main mineralocorticoid hormone, is a steroid hormone produced by the
zona glomerulosa of the adrenal cortex in the adrenal gland.

178- Case of cardiac tamponade, hypotensive, heart voice not clear, trauma,
distend
venous neck, next step?
Pericardiocentesis.
Triad of cardiac tamponade are muffled distant heart sounds + distended neck veins +
hypotension.

179- Pt with cholecystectomy develop pain in the angle of the mouth what u wll
do?
Paracetamol.

180- U did appendictomy and you found it not inflamed you remove it and closed
what the next?
A. Tell the pt.
B. Tell the ethics.
C. Tell the surgery head.

181- You are doing appendictomy u found bus and fluid in the RIF. What u will
do?
A. Remove the fluid and appendix.
B. Remove the appendix.

260
Glory 2019 ..

182- Pt with AAA 4.5 cm, have cholecystectomy, vascular surgery councled what
he wll say?
A. proceed the surgery and folw up with US.
B. PROCEED THE SURGRY FOLOW WITH CT.
C. CT THEN DO THE SURGERY.

183- Pt with recurent abdominal pain us show dilated intrahepac duct T 38 pain
increase with morphin, LFT Normal ALP high what is diagnosis?
A. cholelithiasis.
B. choleangitis.
C. cholecystitis.

184- Pt have epigastric pain after vomiting CXR show some pleural effusion what
is diagnosis ?
rupture esophagus.

185- Pt with Abdominal mass after lefting heavy object mass not change with
cough
what is diagnosis?
A. rectus muscl heamatoma.
B. hernia.

186- Pt with testicular pain and vomiting scrotum is red what u wll do?
A. US.
B. urgent surgry.

187- Patient fall in out stretched hand ...pic of open colles fracture what you
will.do next
A. Open reduction
B. close reduction
C. debridement with open reduction

188- Meickle diverticulum


A. end of ileum 6cm

rule of 2's; 2% of the population, within 2 feet of the ileocecal valve, 2 inches in length,

189- Pt with right ilac fossa pain with bcg negetive.....


A. acute appendicites

261
Glory 2019 ..

190-Clostridium perfengis after surgery - Infection


A.gases Gangrene

191- Patient with breast Ca, polyuria and increase thirst, she has also
Hypercalcemia: what will u give her
A. Normal saline
B. Furosemide
C. Amlodipine

192- Patient with unilateral knee pain and urethral discharge of pus. Negative
culture of the pus:
A. Riter syndrome

> triad
- Since cultures are negative it’s chlamydia

262
Glory 2019 ..

193- Parklond formula, how much fluid are given in first 8hrs for anterior trunk
and circumferenal le= upper and le= lower limbs: 6.3

15 ×4× body surface area burned %

194- patient with Hight parathyroid hormone and hypercalcemia, what u will do
next:
Look for parathyroid adenoma
TSH, T3 and T4

the most common cause for 1 hyperparathyroidism is Adenoma so


initially US
Diagnostic sestamibi scan

195- best prophylaxis for thrombosis post surgery is


A - LMWH

196- case of otitis media and asking about the diagnosis->


A. pain

197- patient with case of otitis media and erytham on mastoid bone, what’s the
treatment:
A- Amoxicillin
B- Cefoxaim

263
Glory 2019 ..

198- 19 yrs old lady with Diarrhea for 10 months, wt loss, postprandial
periumbilical pain. Diarrhea sometimes bloody
A- Crohn’s
B- Peptic ulcer
C- Chronic Pancreatic

199- Pt with cholecystectomy develop pain in the angle of the mouth what you
will do?
Paracetamol

200- U did appendectomy and you found it not inflamed you remove it and closed
what the next
• Tel the pt
• Tel the ethics
• Tell the surgery head

201- Your doing appendectomy u found bus and fluid in the RIF what u wl do
• Remove the fluid and appendix
• Remove the appendix

202- PT WITH persistent abdominal pain US show dilated intrahepatic duct temp
38 pain increase with morphing LFT Normal ALP high what is diagnosis
• cholelithiasis
• choleangitis
• cholecystitis

Fever / abdominal pain / jaundice = cholangitis

203- Pt with AAA 4,5 cm, have cholecystectomy, vascular surgeon consult, what
he will say?
• Proceed the surgery and follow up by US
• Proceed the surgery and follow up with CT
• Ct the do the surgery

204-managenet of papilary breast cancer?


Surgical excision

205-management of ductal breast cancer ?


Surgical excision

206-management of cholelithiasis

264
Glory 2019 ..

Elective Cholysystectomy

207-question about mammogram breast screen


Every 2 years

208-man with both lower limbs burn, ask about fluid replacement

209-parathyroid tumor management?


Surgical ( adenoma total excision)

210-clinical scenarios about epigastric pain, pancreatic, gastric, esophagus


you should know how to diferentiate
Radiat to back >> pancreatic

265
Glory 2019 ..

Associated with food >> gastric


With dysphagia and odynophagia >> esophageal
1082-question about diagnosis of trichomoniasis

211-acute pancreatitis complicated by pseudocyst what is the management


Pancreatic pseudocyst ttt:
- less than 6 cm and 6 week > observation
- more than 6 cm and 6 week > drainage:
A- if there is sign of infection as fever and leukocytosis do external drainage
B- if sterile do internal drainage

212-Patient with complex reagonal pain after surgery to carpal tunnel . Treatment?
NSAID

213-Patint fall from 4 meter. Just has pain in feet.investigation?


X ray feet

214-Hemoptysis with couch . X ray show rt PE massive Plural effusion. Trachea


and heart not displaced. Diagnosis?
A-Rupture esophagus
B-TB
C-Heart faliure
D-Mallory wees

215-CA colon screening age?


colon 50

216- Male with Gun shots. Inter lateral to left nipple .and exit under the scapula.
The patient collapsed in ER. What to do?
A-Iv.fluid
B-Pericardiocentesis

217-Cholysistitis with dilate duct and stone. INV?


A-Ercp
B-US

218-Polyps in colon. With history of blood .diagnosis?


Hereditary polyp Isis

219-Prevent dvt in presurgical patient?


LMWH

266
Glory 2019 ..

220-Elderly with hem a you're a.cystoscopy done and show in the uppermost of
bladder mass? .
A-Transitional cell carcinoma
B-squamous cell carcinoma

221-Anterior resection of colon . Supply from which artery?


A-Superior mesenteric artery

222-Patient with constipation and painfull. Bugging .?


A-Anal Fissure
B-Hemorrhoid thrombus

223- 26 yrs male did appendectomy then was discharged next day as he was wel,8
hours later he came back to hospital with mild pain at site of incision, US
reaveled colection 2cm x2cm ,HB 11 TLC 7.9 bp 110/70 p.76 RR 16
How would you do
A.Exploration
B.observation

N.B: Wound exploration is the best ttt of any postoperative surgical site infection.

224- 35 years old male, I don't remember his complain, on colonoscopy traveled
multiple polyps, your diagnosis?
A- familial polyposis
By cancer

225- Nasogastric tube revealed greenish secretion, next step?


A- bariam meal
B- endoscopy

226- 32 years old male , his left leg is pallor and cold , no hair , your diagnosis ?
A- atrial embolism
B- atrial thrombosis
C- DVT

N.B: Trophic changes like hair loss support atherosclerosis and thrombosis

227- Diabetic ulcer in lateral malleolus ttt?


A- antibiotic
B- chemotherapy
C- surgical debridement

228- Most common site of rodent ulcer (Basal cell carcinoma of the skin)?

267
Glory 2019 ..

A- foot
B- arm
C- face
D- shoulder

229- Anal pain , discharge underwear , in palpation there is tenderness above


sphincter , your diagnosis ?
A- perianal fistula
B- inter sphincter abscess

230- Pt c/o right upper abdominal pain in the right side colicky in nature, radiate to
back, in CT found stones (no write the diameter ) in gall bladder, with normal
Galbladder?
1- Elective Cholestectomy
2- Ursodeoxy colic acid

231- Patient was playing football and then suddenly he heardapop, he is not able
to walk on his tip toe ,what ligament is injured?
A. Quadriceps
B. Calcaneal tendon

232- Posterior hip dislocation ,position of the leg ?


A. Adducted and internal rotation

233- Patient feel scalf pain while waking with loss of hair and the index is 0.8,
what is the diagnosis?
A. Limb ischemia
B. DVT
C. Venous insufficiency

234- Pudendal nerve block all the following except?


B. Rectum

235- How to palpate the dorsalis pedis?


A. Lateral to the extensors halicus longus

And Medial to extensor digitourm longus

236- Function of the anterior compartment of the forearm?


A. Flex both rest and fingers

237- DM pt came to ER with DIC on heparin, past hx of claudication, atrial fib After
DIC what to stop?
A. heparin

268
Glory 2019 ..

B. insulin
C. furosmide

238- colon cancer what to give


A. Vit k
B. Vit c
C. Fibrate

Fibriate + vit D + folic acid

239- 26 yrs male did appendectomy then was discharged next day as he was well ,
8hours later he came back to hospital with mild pain at site of incision, US
reaveled collection 2cm x2cm , HB 11 TLC 7.9 bp 110/70 p. 76 RR 16 , How would
you do
A. Exploration
B. observation
C. CT
D. Need le suction

240- Where do you put the thoracotomy needle?


A. 2nd intercostal mid clavicular
B. 3rd intercostal mid clavicular

241-Elderly pt came with volvulus (Pic)


A- rectum
B- cecum
C- transverse colon
D- sigmoid

242-Pt did surgery in colon, came after 1 week with low grade fever, what you’ll do
?
A- MRI
B- CT
C- exploration

Us is best intial imaging

243--Breast mass after biopsy (sarcoma phyllodes)


A- mastectomy
B- excision with wide margin

269
Glory 2019 ..

C- excision and radiotherapy


surgical approach should include a wide local excision with histologic margins negative
for malignant cells

244-Pt came with lipoma in neck then discover thyroid nodule (suspicious)
What you’ll do ?
A- excision nodule
B- hemi thyroidectomy

245-Child with umbilical hernia, what you’ll do ?


A- Reassurance

246-Case of pneumothorax, what will you do?

247-Same case but with intact air entry & percussion dull
A- cardiac tamponade

248-Ulcer in medial malleolus ( HTN, not DM )


First step !
A- Duplex
B- angiography
C- distal pulse
Ulcer over medial malleolus most probably venous ulcer so initial test duplex us

249--24 yrs with vomiting, endoscopy done and he has stricture


A- Crohn’s

250--Female came with abdominal pain + distention + fever, hx of laparoscopic


sleeve 6 yrs ago
What’s the cause ?
A- internal hernia
B- incisional hernia
C- intussusception

251-Female compline of lower outer quadrant lumb and it's (hyperplasia


something like that) and she had strong family history of breast canser what is
the management
A/ total mastectomy
B/ Excision of tumor
C/ reassurance

252-Pt came to the ER complain of sever epigastric pain and he is hypotensive


and had history of cholecystitis what is the diagnosis

270
Glory 2019 ..

A/ perforated peptic ulcer

253-Same scenario of previous question and he mention the diagnosis (peptic


ulcer) and ask about operation
A/ iliojejunestomy
B/repair of injury by graft from
C/gastrectomy

254-Pt c/o of S.O.B on ex there is muffled heart sound and dilated ventricle what
is the diagnosis
A/core pulmonale
B/pulmonary edema
C/Cardiac tamponade

255-Pt c/o thyroid enlargement (multi nodular or diffuse l not remember) and there
is two lumps One is smal1×2 and other larger 3×4 what to do
A/FNA from largest one
B/FNA from both lumps
C/Excisional biopsy

256-Ask about diagnosis of fibroadenoma


- case of cervical carcinoma and ask about the investigation ? (read about
endometrial
carcinoma and cervical carcinoma)

257- 70 years old complain of fresh blood from rectum and when to do endoscopy
you found
polyps what is the management :
A/ Reassurance
B/ tell him to come for follow up after 6 wks
C/ do resection of polyp
D/ resection of rectum ( something like that )

271
Glory 2019 ..

258-pt came to the ER and complained of burn in his tow lower limbs
his wt 70 kg what is the amount of fluid must you give him
A/ ringer lactate 2500 ml at 1 st 8 hrs and 2500 ml at second 1 6 hrs
B/ NS 5000 ml at 1 st 8 hrs

259-Pt her age 55 when do breast screen (mammogram)


A- after one year
B- 2yrs
C- 3yrs

260-thyroid nodule, what the intimation investigation


A- thyroid scan if toxic do this
B- FNA

261-Pt with enlargement of Thyroid, large after in thyroid scan, what the second
option

262- Where do you put the thoracotomy needle?


2nd intercostal mid clavicular
3rdintercostal midclavicular

263- Patient with bag of worms in the scrotum, what will you do ?
Dopler

264-Scrotal mass which is basically hernia what to do?


Refer to surgery-

265- The reason of unhealed ulcer in diabetic patients?


Decrease phagocytosis
decrease immune response
sugar is a good media

272
Glory 2019 ..

266- You found 1cm neck mass, next step?


Biopsy
US
Neck CT

267- Contraindication for circumcision


hypospadias

268- Pancreatitis case with high ggt->alcohol

269- scaphoid fracture


AVN

270- Spinal stenosis case. Relieve after leaning forward pain decreases

271- Post vagatomy->esophagus

272- Radical mastectomy....


Noticed wigned scapula injury to LTA

273- post operative of appendicectomy came with vague Abd pain o/x mild
tenderness CT shows retrocecal collection 2×2 what to do?

274- -post lap cholecystectomy developed high grade fever with shills CT
subhepatic collection what to do
Depends on the size if small abx and observe, If drainage add abx

275- with lower abd pain o/px mass, lower abdomen tenderness mainly RIF with
fullness, pregnancy test negative?
a)acute cystitis
b)ovarian torsion
c)rupture cyst
d)appendicitis

276- pt c/o thyroid enlargment (multi noduler or difuse l not remember) and there
is two lumps ones small 1×2 and other larger 3×4 what to do?
A/ FNA from largest one
B/ FNA from both lumps
C/Excisional biopsy

273
Glory 2019 ..

277- One case of pain on the thenar aspect of the hand after a Surgery n the
patient been , seen by orthopaedic.
A.give analgesics.
B.opiot.
C.physioyherapy. ??
D.codeine

278-After 12 hours of gastric slap surgury, pt Came with upper abdominal pain,
every thing normal?
Increase analgesia
Laporotomy

279- Ant resection of colon. Supply from which artery?


Superior mesmeric artery

280- Patient with metastatic breast ca present with SOB , distended neck vein,
apex beat not palpable. distal heart sounds , BP 70/20. The best step to conrm the
diagnosis?
A. IV fuid & urgent echo
B. IV furosemide
C. Pericardiocentesis D. chest tube

281- Patient with the breast mass, which of the following indicates that the mass
is cystic (or no need for reevaluation)?
A. Bloody nipple discharge
B. Clear fluid with reappearance of the mass
C. FNA showing clear fluid on aspiration and disappearance of the mass after
D. fatty tissue appearance

282- 41-year-old Lady has breast mass since 2 weeks, upon examination it hard
and fixed, what is "most " accurate diagnostic modality?
A. Lactography
B. MRI Breast
C. Mammography
D. US

274
Glory 2019 ..

192-
-

1497-

275

You might also like